| NCERT Exemplar Solutions | ||||||
|---|---|---|---|---|---|---|
| 6th | 7th | 8th | 9th | 10th | 11th | 12th |
Chapter 3 Trigonometric Functions
Welcome to this comprehensive resource dedicated to providing detailed solutions and thorough explanations for the Class 11 NCERT Exemplar problems focused on Trigonometric Functions. This crucial chapter builds significantly upon earlier introductions, and the Exemplar questions are specifically designed to push your understanding far beyond standard formula application. They demand a deeper mastery of intricate trigonometric identities, sophisticated equation-solving techniques, insightful graphical analysis, and the ability to apply these concepts in complex scenarios, often requiring intricate algebraic manipulation and robust conceptual clarity. Successfully navigating these challenges is vital for building the strong trigonometric foundation required for calculus and other advanced mathematical fields.
Our solutions begin by reinforcing fundamental concepts, but within the challenging context set by the Exemplar. This includes conversions between degree and radian measures (understanding that $\pi$ radians $= 180^\circ$) in non-trivial problems, applying the arc length formula $l=r\theta$ (where $\theta$ is in radians), and solidifying the understanding of trigonometric functions defined using the unit circle for any angle $\theta$. This unit circle perspective is key to grasping the signs of trigonometric functions in different quadrants, their respective domains and ranges (e.g., range of $\sin x$ and $\cos x$ is $[-1, 1]$), and their inherent periodicity (e.g., $\sin(x+2n\pi) = \sin x$, $\tan(x+n\pi) = \tan x$ for integer $n$).
A primary focus of the Exemplar, and thus these solutions, is the rigorous proving of complex trigonometric identities. Success here requires more than just knowing the basic formulas; it demands strategic thinking and skillful manipulation. The solutions demonstrate the application of a wide arsenal of identities, including:
- Fundamental Pythagorean identities (e.g., $\sin^2 A + \cos^2 A = 1$, $1 + \tan^2 A = \sec^2 A$).
- Sum and difference formulas (like $\sin(A \pm B)$, $\cos(A \pm B)$, $\tan(A \pm B)$).
- Multiple angle formulas (such as $\sin 2A$, $\cos 2A = \cos^2 A - \sin^2 A$, $\tan 2A$, $\sin 3A$, $\cos 3A$).
- Sub-multiple angle formulas (derived from multiple angle formulas).
- Sum-to-product and product-to-sum transformations.
Exemplar proofs often involve multiple steps, clever substitutions, conversion to $\sin$ and $\cos$, finding common denominators, or rationalization. The solutions provide clear, step-by-step pathways, outlining effective strategies for tackling these demanding proofs.
Solving trigonometric equations constitutes another critical area. The solutions guide you through finding not only the principal solutions (typically within the interval $[0, 2\pi)$) but also, more importantly, the general solutions, which express all possible solutions using an integer parameter $n$ (e.g., if $\sin x = \sin \alpha$, then $x = n\pi + (-1)^n \alpha$, $n \in \mathbb{Z}$). Exemplar problems often require simplifying complex equations involving multiple functions or angles down to basic forms before applying the general solution formulas, sometimes involving factorization or squaring. Conditional identities, particularly those relevant to the angles of a triangle ($A+B+C=\pi$), are also addressed. Furthermore, problems requiring the determination of maximum or minimum values of trigonometric expressions, often solved using identities or the bounded nature of sine and cosine, are included. While graphical understanding is conceptually important, the focus remains largely on analytical manipulation and solving. These solutions cover all typical Exemplar formats (MCQs, Fill-in-the-Blanks, True/False, Short/Long Answer) and are indispensable for developing the advanced trigonometric skills essential for future mathematical endeavors.
Solved Examples
Example 1 to 8 (Short Answer Type Questions)
Example 1: A circular wire of radius 3 cm is cut and bent so as to lie along the circumference of a hoop whose radius is 48 cm. Find the angle in degrees which is subtended at the centre of hoop.
Answer:
Given:
Radius of the circular wire, $r_{wire} = 3$ cm.
Radius of the hoop, $r_{hoop} = 48$ cm.
To Find:
The angle (in degrees) subtended at the centre of the hoop.
Solution:
When the circular wire is cut and bent, its length becomes the arc length on the circumference of the hoop.
The length of the circular wire is equal to its circumference.
Circumference of the wire, $C = 2 \pi r_{wire}$.
$C = 2 \pi (3)$ cm
$C = 6\pi$ cm.
This length acts as the arc length, $s$, on the hoop. So, $s = 6\pi$ cm.
The radius of the hoop is $r_{hoop} = 48$ cm.
The formula relating arc length ($s$), radius ($r$), and the angle subtended at the centre ($\theta$) in radians is $s = r\theta$.
Substituting the values, we get:
$6\pi = 48 \times \theta$
Solving for $\theta$ in radians:
$\theta = \frac{6\pi}{48}$
$\theta = \frac{\pi}{8}$ radians.
To convert radians to degrees, we use the relation $1 \text{ radian} = \frac{180^\circ}{\pi}$.
Angle in degrees $= \theta \times \frac{180^\circ}{\pi}$
Angle in degrees $= \frac{\pi}{8} \times \frac{180^\circ}{\pi}$
Angle in degrees $= \frac{180}{8}$ degrees
Angle in degrees $= \frac{45}{2}$ degrees
Angle in degrees $= 22.5^\circ$.
Therefore, the angle subtended at the centre of the hoop is $22.5^\circ$.
Example 2: If A = cos2 θ + sin4 θ for all values of θ, then prove that $\frac{3}{4}$ ≤ A ≤ 1.
Answer:
Given:
$A = \cos^2 \theta + \sin^4 \theta$ for all values of $\theta$.
To Prove:
$\frac{3}{4} \leq A \leq 1$.
Proof:
We are given the expression $A = \cos^2 \theta + \sin^4 \theta$.
We know the identity $\cos^2 \theta = 1 - \sin^2 \theta$. Substitute this into the expression for A:
$A = (1 - \sin^2 \theta) + \sin^4 \theta$
$A = 1 - \sin^2 \theta + \sin^4 \theta$
Let $x = \sin^2 \theta$. Since $-1 \leq \sin \theta \leq 1$, we have $0 \leq \sin^2 \theta \leq 1$. Thus, $0 \leq x \leq 1$.
The expression for A becomes a quadratic in $x$:
$A = x^2 - x + 1$
We need to find the range of this quadratic function for $x \in [0, 1]$.
Consider the function $f(x) = x^2 - x + 1$. This is a parabola opening upwards.
The vertex of the parabola occurs at $x = -\frac{(-1)}{2(1)} = \frac{1}{2}$.
Since the vertex is within the interval $[0, 1]$, the minimum value of $f(x)$ occurs at the vertex.
Minimum value of $A = f(\frac{1}{2}) = (\frac{1}{2})^2 - \frac{1}{2} + 1 = \frac{1}{4} - \frac{1}{2} + 1 = \frac{1 - 2 + 4}{4} = \frac{3}{4}$.
The maximum value of $f(x)$ on the interval $[0, 1]$ occurs at one of the endpoints.
At $x=0$ (which corresponds to $\sin^2 \theta = 0$, e.g., $\theta = 0$):
$A = f(0) = 0^2 - 0 + 1 = 1$.
At $x=1$ (which corresponds to $\sin^2 \theta = 1$, e.g., $\theta = \frac{\pi}{2}$):
$A = f(1) = 1^2 - 1 + 1 = 1$.
Thus, the maximum value of A is 1.
Combining the minimum and maximum values, we get:
$\frac{3}{4} \leq A \leq 1$.
This proves the required inequality.
Example 3: Find the value of $\sqrt{3}$ cosec 20° – sec 20°
Answer:
We need to find the value of the expression $\sqrt{3} \text{ cosec } 20^\circ - \sec 20^\circ$.
Rewrite the expression in terms of sine and cosine:
$\sqrt{3} \text{ cosec } 20^\circ - \sec 20^\circ = \sqrt{3} \left(\frac{1}{\sin 20^\circ}\right) - \left(\frac{1}{\cos 20^\circ}\right)$
Combine the terms into a single fraction:
$= \frac{\sqrt{3} \cos 20^\circ - \sin 20^\circ}{\sin 20^\circ \cos 20^\circ}$
Consider the numerator: $\sqrt{3} \cos 20^\circ - \sin 20^\circ$.
We can write this in the form $R \sin(A-B)$ or $R \cos(A+B)$.
Let's multiply and divide the numerator by 2:
$2 \left(\frac{\sqrt{3}}{2} \cos 20^\circ - \frac{1}{2} \sin 20^\circ\right)$
We know that $\sin 60^\circ = \frac{\sqrt{3}}{2}$ and $\cos 60^\circ = \frac{1}{2}$.
So, the numerator becomes:
$2 (\sin 60^\circ \cos 20^\circ - \cos 60^\circ \sin 20^\circ)$
Using the identity $\sin(A-B) = \sin A \cos B - \cos A \sin B$, with $A = 60^\circ$ and $B = 20^\circ$:
Numerator $= 2 \sin(60^\circ - 20^\circ) = 2 \sin 40^\circ$.
Consider the denominator: $\sin 20^\circ \cos 20^\circ$.
We know the double angle identity $\sin 2\theta = 2 \sin \theta \cos \theta$.
So, $\sin \theta \cos \theta = \frac{1}{2} \sin 2\theta$.
Using this identity with $\theta = 20^\circ$:
Denominator $= \frac{1}{2} \sin(2 \times 20^\circ) = \frac{1}{2} \sin 40^\circ$.
Now, substitute the simplified numerator and denominator back into the expression:
$\frac{\text{Numerator}}{\text{Denominator}} = \frac{2 \sin 40^\circ}{\frac{1}{2} \sin 40^\circ}$
Cancel out the $\sin 40^\circ$ terms (assuming $\sin 40^\circ \neq 0$, which is true):
$= \frac{2}{\frac{1}{2}}$
$= 2 \times 2$
$= 4$.
Thus, the value of $\sqrt{3}$ cosec 20° – sec 20° is 4.
Example 4: If θ lies in the second quadrant, then show that
$\sqrt{\frac{1\;-\;\sin\theta}{1\;+\;\sin\theta}} + \sqrt{\frac{1\;+\;\sin\theta}{1\;-\;\sin\theta}} = - 2sec \;θ$
Answer:
Given:
The expression $\sqrt{\frac{1\;-\;\sin\theta}{1\;+\;\sin\theta}} + \sqrt{\frac{1\;+\;\sin\theta}{1\;-\;\sin\theta}}$.
$\theta$ lies in the second quadrant.
To Prove:
$\sqrt{\frac{1\;-\;\sin\theta}{1\;+\;\sin\theta}} + \sqrt{\frac{1\;+\;\sin\theta}{1\;-\;\sin\theta}} = - 2\sec \theta$.
Proof:
Consider the Left Hand Side (LHS) of the equation:
LHS $= \sqrt{\frac{1\;-\;\sin\theta}{1\;+\;\sin\theta}} + \sqrt{\frac{1\;+\;\sin\theta}{1\;-\;\sin\theta}}$
Rationalize the denominators of the terms inside the square roots:
First term: $\sqrt{\frac{1\;-\;\sin\theta}{1\;+\;\sin\theta}} = \sqrt{\frac{(1\;-\;\sin\theta)(1\;-\;\sin\theta)}{(1\;+\;\sin\theta)(1\;-\;\sin\theta)}}$
$= \sqrt{\frac{(1\;-\;\sin\theta)^2}{1\;-\;\sin^2\theta}} = \sqrt{\frac{(1\;-\;\sin\theta)^2}{\cos^2\theta}}$
Second term: $\sqrt{\frac{1\;+\;\sin\theta}{1\;-\;\sin\theta}} = \sqrt{\frac{(1\;+\;\sin\theta)(1\;+\;\sin\theta)}{(1\;-\;\sin\theta)(1\;+\;\sin\theta)}}$
$= \sqrt{\frac{(1\;+\;\sin\theta)^2}{1\;-\;\sin^2\theta}} = \sqrt{\frac{(1\;+\;\sin\theta)^2}{\cos^2\theta}}$
So, LHS $= \sqrt{\frac{(1\;-\;\sin\theta)^2}{\cos^2\theta}} + \sqrt{\frac{(1\;+\;\sin\theta)^2}{\cos^2\theta}}$
Using the property $\sqrt{a^2} = |a|$, we get:
LHS $= \frac{|1\;-\;\sin\theta|}{|\cos\theta|} + \frac{|1\;+\;\sin\theta|}{|\cos\theta|}$
We are given that $\theta$ lies in the second quadrant.
In the second quadrant:
$\sin \theta > 0$. Therefore, $1 - \sin \theta$ can be positive or negative. However, since $-1 \le \sin\theta \le 1$, $1-\sin\theta \ge 0$. So $|1 - \sin \theta| = 1 - \sin \theta$.
$\sin \theta > 0$. Therefore, $1 + \sin \theta > 0$. So $|1 + \sin \theta| = 1 + \sin \theta$.
$\cos \theta < 0$. Therefore, $|\cos \theta| = -\cos \theta$.
Substitute these absolute values back into the expression for LHS:
LHS $= \frac{1\;-\;\sin\theta}{-\cos\theta} + \frac{1\;+\;\sin\theta}{-\cos\theta}$
Combine the fractions since they have a common denominator:
LHS $= \frac{(1\;-\;\sin\theta) + (1\;+\;\sin\theta)}{-\cos\theta}$
LHS $= \frac{1\;-\;\sin\theta + 1\;+\;\sin\theta}{-\cos\theta}$
LHS $= \frac{2}{-\cos\theta}$
LHS $= -\frac{2}{\cos\theta}$
LHS $= -2 \left(\frac{1}{\cos\theta}\right)$
Using the reciprocal identity $\sec \theta = \frac{1}{\cos \theta}$:
LHS $= -2 \sec \theta$
This is equal to the Right Hand Side (RHS).
Thus, $\sqrt{\frac{1\;-\;\sin\theta}{1\;+\;\sin\theta}} + \sqrt{\frac{1\;+\;\sin\theta}{1\;-\;\sin\theta}} = - 2\sec \theta$.
Hence Proved.
Example 5: Find the value of tan 9° – tan 27° – tan 63° + tan 81°
Answer:
We need to find the value of $\tan 9^\circ - \tan 27^\circ - \tan 63^\circ + \tan 81^\circ$.
We can use the complementary angle identities: $\tan(90^\circ - \theta) = \cot \theta$.
$\tan 81^\circ = \tan(90^\circ - 9^\circ) = \cot 9^\circ$
$\tan 63^\circ = \tan(90^\circ - 27^\circ) = \cot 27^\circ$
Substitute these into the expression:
Expression $= \tan 9^\circ - \tan 27^\circ - \cot 27^\circ + \cot 9^\circ$
Rearrange the terms:
Expression $= (\tan 9^\circ + \cot 9^\circ) - (\tan 27^\circ + \cot 27^\circ)$
Recall the identity: $\tan \theta + \cot \theta = \frac{\sin \theta}{\cos \theta} + \frac{\cos \theta}{\sin \theta} = \frac{\sin^2 \theta + \cos^2 \theta}{\sin \theta \cos \theta} = \frac{1}{\sin \theta \cos \theta}$.
Also, using the double angle identity $\sin 2\theta = 2 \sin \theta \cos \theta$, we have $\sin \theta \cos \theta = \frac{1}{2} \sin 2\theta$.
So, $\tan \theta + \cot \theta = \frac{1}{\frac{1}{2} \sin 2\theta} = \frac{2}{\sin 2\theta}$.
Apply this identity to our expression:
$\tan 9^\circ + \cot 9^\circ = \frac{2}{\sin (2 \times 9^\circ)} = \frac{2}{\sin 18^\circ}$
$\tan 27^\circ + \cot 27^\circ = \frac{2}{\sin (2 \times 27^\circ)} = \frac{2}{\sin 54^\circ}$
The expression becomes: $\frac{2}{\sin 18^\circ} - \frac{2}{\sin 54^\circ}$
We know the exact values:
$\sin 18^\circ = \frac{\sqrt{5}-1}{4}$
$\sin 54^\circ = \cos (90^\circ - 54^\circ) = \cos 36^\circ = \frac{\sqrt{5}+1}{4}$
Substitute these values into the expression:
Expression $= \frac{2}{\frac{\sqrt{5}-1}{4}} - \frac{2}{\frac{\sqrt{5}+1}{4}}$
Expression $= \frac{8}{\sqrt{5}-1} - \frac{8}{\sqrt{5}+1}$
Combine the fractions by finding a common denominator:
Common denominator $= (\sqrt{5}-1)(\sqrt{5}+1) = (\sqrt{5})^2 - 1^2 = 5 - 1 = 4$.
Expression $= \frac{8(\sqrt{5}+1)}{(\sqrt{5}-1)(\sqrt{5}+1)} - \frac{8(\sqrt{5}-1)}{(\sqrt{5}+1)(\sqrt{5}-1)}$
Expression $= \frac{8(\sqrt{5}+1) - 8(\sqrt{5}-1)}{4}$
Expression $= \frac{8\sqrt{5} + 8 - 8\sqrt{5} + 8}{4}$
Expression $= \frac{16}{4}$
Expression $= 4$
The value of $\tan 9^\circ - \tan 27^\circ - \tan 63^\circ + \tan 81^\circ$ is 4.
Example 6: Prove that $\frac{sec \;8 θ \;-\; 1}{sec \;4 θ \;-\; 1} = \frac{tan \;8 θ}{tan \;2 θ}$
Answer:
To Prove:
$\frac{\sec 8\theta - 1}{\sec 4\theta - 1} = \frac{\tan 8\theta}{\tan 2\theta}$
Proof:
Consider the Left Hand Side (LHS):
LHS $= \frac{\sec 8\theta - 1}{\sec 4\theta - 1}$
Rewrite sec in terms of cos:
LHS $= \frac{\frac{1}{\cos 8\theta} - 1}{\frac{1}{\cos 4\theta} - 1}$
Combine the terms in the numerator and denominator:
LHS $= \frac{\frac{1 - \cos 8\theta}{\cos 8\theta}}{\frac{1 - \cos 4\theta}{\cos 4\theta}}$
LHS $= \frac{1 - \cos 8\theta}{\cos 8\theta} \times \frac{\cos 4\theta}{1 - \cos 4\theta}$
Using the identity $1 - \cos 2x = 2 \sin^2 x$:
$1 - \cos 8\theta = 2 \sin^2 4\theta$
$1 - \cos 4\theta = 2 \sin^2 2\theta$
Substitute these into the LHS expression:
LHS $= \frac{2 \sin^2 4\theta}{\cos 8\theta} \times \frac{\cos 4\theta}{2 \sin^2 2\theta}$
LHS $= \frac{\sin^2 4\theta \cos 4\theta}{\cos 8\theta \sin^2 2\theta}$
Now consider the Right Hand Side (RHS):
RHS $= \frac{\tan 8\theta}{\tan 2\theta}$
Rewrite tan in terms of sin and cos:
RHS $= \frac{\frac{\sin 8\theta}{\cos 8\theta}}{\frac{\sin 2\theta}{\cos 2\theta}}$
RHS $= \frac{\sin 8\theta}{\cos 8\theta} \times \frac{\cos 2\theta}{\sin 2\theta}$
Using the double angle identity $\sin 2x = 2 \sin x \cos x$:
$\sin 8\theta = 2 \sin 4\theta \cos 4\theta$
$\sin 4\theta = 2 \sin 2\theta \cos 2\theta$
Substitute $\sin 8\theta$ into the RHS expression:
RHS $= \frac{2 \sin 4\theta \cos 4\theta}{\cos 8\theta} \times \frac{\cos 2\theta}{\sin 2\theta}$
Substitute $\sin 4\theta$ into the RHS expression:
RHS $= \frac{2 (2 \sin 2\theta \cos 2\theta) \cos 4\theta}{\cos 8\theta} \times \frac{\cos 2\theta}{\sin 2\theta}$
RHS $= \frac{4 \sin 2\theta \cos 2\theta \cos 4\theta \cos 2\theta}{\cos 8\theta \sin 2\theta}$
Assuming $\sin 2\theta \neq 0$, we can cancel $\sin 2\theta$ from numerator and denominator:
RHS $= \frac{4 \cos 2\theta \cos 4\theta \cos 2\theta}{\cos 8\theta}$
RHS $= \frac{4 \cos^2 2\theta \cos 4\theta}{\cos 8\theta}$
Let's go back to the simplified LHS:
LHS $= \frac{\sin^2 4\theta \cos 4\theta}{\cos 8\theta \sin^2 2\theta}$
Substitute $\sin 4\theta = 2 \sin 2\theta \cos 2\theta$ into the LHS:
LHS $= \frac{(2 \sin 2\theta \cos 2\theta)^2 \cos 4\theta}{\cos 8\theta \sin^2 2\theta}$
LHS $= \frac{4 \sin^2 2\theta \cos^2 2\theta \cos 4\theta}{\cos 8\theta \sin^2 2\theta}$
Assuming $\sin 2\theta \neq 0$, we can cancel $\sin^2 2\theta$ from numerator and denominator:
LHS $= \frac{4 \cos^2 2\theta \cos 4\theta}{\cos 8\theta}$
We see that LHS $= \frac{4 \cos^2 2\theta \cos 4\theta}{\cos 8\theta}$ and RHS $= \frac{4 \cos^2 2\theta \cos 4\theta}{\cos 8\theta}$.
Therefore, LHS = RHS.
The identity is proven, provided that $\cos 8\theta \neq 0$, $\cos 4\theta \neq 0$, and $\sin 2\theta \neq 0$ (which implies $\cos 2\theta \neq 0$ for $\tan 2\theta$ to be defined in the first place).
Hence Proved.
Example 7: Solve the equation sin θ + sin 3θ + sin 5θ = 0
Answer:
Solution:
We are asked to solve the equation $\sin \theta + \sin 3\theta + \sin 5\theta = 0$.
Rearrange the terms to group $\sin 5\theta$ and $\sin \theta$:
$(\sin 5\theta + \sin \theta) + \sin 3\theta = 0$
Use the sum-to-product formula: $\sin A + \sin B = 2 \sin \left(\frac{A+B}{2}\right) \cos \left(\frac{A-B}{2}\right)$.
Let $A = 5\theta$ and $B = \theta$.
$\sin 5\theta + \sin \theta = 2 \sin \left(\frac{5\theta+\theta}{2}\right) \cos \left(\frac{5\theta-\theta}{2}\right) = 2 \sin \left(\frac{6\theta}{2}\right) \cos \left(\frac{4\theta}{2}\right) = 2 \sin 3\theta \cos 2\theta$.
Substitute this back into the equation:
$2 \sin 3\theta \cos 2\theta + \sin 3\theta = 0$
Factor out the common term $\sin 3\theta$:
$\sin 3\theta (2 \cos 2\theta + 1) = 0$
This equation holds true if either $\sin 3\theta = 0$ or $2 \cos 2\theta + 1 = 0$.
Case 1: $\sin 3\theta = 0$
The general solution for $\sin x = 0$ is $x = n\pi$, where $n$ is an integer ($n \in \mathbb{Z}$).
So, $3\theta = n\pi$
$\theta = \frac{n\pi}{3}$, where $n \in \mathbb{Z}$.
Case 2: $2 \cos 2\theta + 1 = 0$
$2 \cos 2\theta = -1$
$\cos 2\theta = -\frac{1}{2}$
The principal value $\alpha$ for which $\cos \alpha = -\frac{1}{2}$ is $\alpha = \frac{2\pi}{3}$ (since $\cos \frac{2\pi}{3} = -\frac{1}{2}$).
The general solution for $\cos x = \cos \alpha$ is $x = 2k\pi \pm \alpha$, where $k$ is an integer ($k \in \mathbb{Z}$).
So, $2\theta = 2k\pi \pm \frac{2\pi}{3}$
Divide by 2:
$\theta = k\pi \pm \frac{\pi}{3}$, where $k \in \mathbb{Z}$.
Combining the solutions from both cases, the general solution to the equation $\sin \theta + \sin 3\theta + \sin 5\theta = 0$ is:
$\theta = \frac{n\pi}{3}$ or $\theta = k\pi \pm \frac{\pi}{3}$, where $n, k \in \mathbb{Z}$.
Example 8: Solve 2 tan2 x + sec2 x = 2 for 0 ≤ x ≤ 2π
Answer:
Given:
The equation $2 \tan^2 x + \sec^2 x = 2$.
The interval $0 \leq x \leq 2\pi$.
To Solve:
Solve the given equation for $x$ in the interval $[0, 2\pi]$.
Solution:
The given equation is:
$2 \tan^2 x + \sec^2 x = 2$
We use the fundamental trigonometric identity relating tangent and secant:
$\sec^2 x = 1 + \tan^2 x$
Substitute this identity into the given equation:
$2 \tan^2 x + (1 + \tan^2 x) = 2$
Combine like terms:
$3 \tan^2 x + 1 = 2$
Subtract 1 from both sides:
$3 \tan^2 x = 2 - 1$
$3 \tan^2 x = 1$
Divide by 3:
$\tan^2 x = \frac{1}{3}$
Take the square root of both sides:
$\tan x = \pm \sqrt{\frac{1}{3}}$
$\tan x = \pm \frac{1}{\sqrt{3}}$
We have two cases to consider:
Case 1: $\tan x = \frac{1}{\sqrt{3}}$
The general solution for $\tan x = a$ is $x = n\pi + \alpha$, where $n$ is an integer ($n \in \mathbb{Z}$) and $\tan \alpha = a$.
For $\tan x = \frac{1}{\sqrt{3}}$, the principal value is $\alpha = \frac{\pi}{6}$ (since $\tan \frac{\pi}{6} = \frac{1}{\sqrt{3}}$).
The general solution is $x = n\pi + \frac{\pi}{6}$, where $n \in \mathbb{Z}$.
We need to find the values of $x$ in the interval $0 \leq x \leq 2\pi$.
For $n=0$: $x = 0\pi + \frac{\pi}{6} = \frac{\pi}{6}$. (This is in the interval $[0, 2\pi]$)
For $n=1$: $x = 1\pi + \frac{\pi}{6} = \frac{6\pi + \pi}{6} = \frac{7\pi}{6}$. (This is in the interval $[0, 2\pi]$)
For $n=2$: $x = 2\pi + \frac{\pi}{6} = \frac{13\pi}{6}$. (This is outside the interval $[0, 2\pi]$)
For $n=-1$: $x = -1\pi + \frac{\pi}{6} = -\frac{5\pi}{6}$. (This is outside the interval $[0, 2\pi]$)
Case 2: $\tan x = -\frac{1}{\sqrt{3}}$
For $\tan x = -\frac{1}{\sqrt{3}}$, we need to find a value $\alpha$ such that $\tan \alpha = -\frac{1}{\sqrt{3}}$. A convenient value in the range $[0, \pi)$ is $\alpha = \pi - \frac{\pi}{6} = \frac{5\pi}{6}$ (since tangent is negative in the second quadrant).
The general solution is $x = k\pi + \frac{5\pi}{6}$, where $k$ is an integer ($k \in \mathbb{Z}$).
We need to find the values of $x$ in the interval $0 \leq x \leq 2\pi$.
For $k=0$: $x = 0\pi + \frac{5\pi}{6} = \frac{5\pi}{6}$. (This is in the interval $[0, 2\pi]$)
For $k=1$: $x = 1\pi + \frac{5\pi}{6} = \frac{6\pi + 5\pi}{6} = \frac{11\pi}{6}$. (This is in the interval $[0, 2\pi]$)
For $k=2$: $x = 2\pi + \frac{5\pi}{6} = \frac{17\pi}{6}$. (This is outside the interval $[0, 2\pi]$)
For $k=-1$: $x = -1\pi + \frac{5\pi}{6} = -\frac{\pi}{6}$. (This is outside the interval $[0, 2\pi]$)
The solutions for $x$ in the interval $0 \leq x \leq 2\pi$ are the values found in Case 1 and Case 2 that lie within this interval.
The solutions are $\frac{\pi}{6}$, $\frac{7\pi}{6}$ (from Case 1) and $\frac{5\pi}{6}$, $\frac{11\pi}{6}$ (from Case 2).
The set of solutions is $\left\{\frac{\pi}{6}, \frac{5\pi}{6}, \frac{7\pi}{6}, \frac{11\pi}{6}\right\}$.
Example 9 to 14 (Long Answer Type Questions)
Example 9: Find the value of $\left( 1+cos\frac{\pi}{8} \right)\left( 1+cos\frac{3\pi}{8} \right)+\left( 1+cos\frac{5\pi}{8} \right)\left( 1+cos\frac{7\pi}{8} \right)$
Answer:
Let the given expression be $E$.
We have the angles $\frac{\pi}{8}, \frac{3\pi}{8}, \frac{5\pi}{8}, \frac{7\pi}{8}$.
Observe the relationships between the angles:
$\frac{5\pi}{8} = \pi - \frac{3\pi}{8}$
$\frac{7\pi}{8} = \pi - \frac{\pi}{8}$
Using the trigonometric identity $\cos(\pi - x) = -\cos x$:
$\cos\frac{5\pi}{8} = \cos(\pi - \frac{3\pi}{8}) = -\cos\frac{3\pi}{8}$
$\cos\frac{7\pi}{8} = \cos(\pi - \frac{\pi}{8}) = -\cos\frac{\pi}{8}$
Substitute these into the given expression $E$:
$E = \left( 1+\cos\frac{\pi}{8} \right)\left( 1+\cos\frac{3\pi}{8} \right)+\left( 1-\cos\frac{3\pi}{8} \right)\left( 1-\cos\frac{\pi}{8} \right)$
Let $A = \cos\frac{\pi}{8}$ and $B = \cos\frac{3\pi}{8}$. The expression becomes:
$E = (1+A)(1+B) + (1-B)(1-A)$
$E = (1 + A + B + AB) + (1 - A - B + AB)$
$E = 1 + A + B + AB + 1 - A - B + AB$
$E = 2 + 2AB$
Substitute $A = \cos\frac{\pi}{8}$ and $B = \cos\frac{3\pi}{8}$ back:
$E = 2 + 2\cos\frac{\pi}{8}\cos\frac{3\pi}{8}$
Using the product-to-sum formula $2\cos x \cos y = \cos(x+y) + \cos(x-y)$:
$2\cos\frac{\pi}{8}\cos\frac{3\pi}{8} = \cos\left(\frac{\pi}{8}+\frac{3\pi}{8}\right) + \cos\left(\frac{\pi}{8}-\frac{3\pi}{8}\right)$
$= \cos\left(\frac{4\pi}{8}\right) + \cos\left(-\frac{2\pi}{8}\right)$
$= \cos\left(\frac{\pi}{2}\right) + \cos\left(-\frac{\pi}{4}\right)$
Using the values $\cos\left(\frac{\pi}{2}\right) = 0$ and $\cos(-x) = \cos x$, so $\cos\left(-\frac{\pi}{4}\right) = \cos\left(\frac{\pi}{4}\right) = \frac{1}{\sqrt{2}}$:
$2\cos\frac{\pi}{8}\cos\frac{3\pi}{8} = 0 + \frac{1}{\sqrt{2}} = \frac{1}{\sqrt{2}}$
Substitute this back into the expression for $E$:
$E = 2 + \frac{1}{\sqrt{2}}$
We can also write $\frac{1}{\sqrt{2}}$ as $\frac{\sqrt{2}}{2}$:
$E = 2 + \frac{\sqrt{2}}{2} = \frac{4 + \sqrt{2}}{2}$
The final value of the expression is $\mathbf{2 + \frac{1}{\sqrt{2}}}$ or $\mathbf{2 + \frac{\sqrt{2}}{2}}$.
Example 10: If x cos θ = y cos $\left( \theta+\frac{2\pi}{3} \right)$ = z $\left( \theta+\frac{4\pi}{3} \right)$ then find the value of xy + yz + zx.
Answer:
Solution:
Given that $x \cos \theta = y \cos \left( \theta+\frac{2\pi}{3} \right) = z \cos \left( \theta+\frac{4\pi}{3} \right)$.
Let this common value be equal to $k$.
$x \cos \theta = k \implies x = \frac{k}{\cos \theta}$
$y \cos \left( \theta+\frac{2\pi}{3} \right) = k \implies y = \frac{k}{\cos \left( \theta+\frac{2\pi}{3} \right)}$
$z \cos \left( \theta+\frac{4\pi}{3} \right) = k \implies z = \frac{k}{\cos \left( \theta+\frac{4\pi}{3} \right)}$
We need to find the value of $xy + yz + zx$.
Substitute the expressions for $x, y, z$:
$xy + yz + zx = \left( \frac{k}{\cos \theta} \right) \left( \frac{k}{\cos \left( \theta+\frac{2\pi}{3} \right)} \right) + \left( \frac{k}{\cos \left( \theta+\frac{2\pi}{3} \right)} \right) \left( \frac{k}{\cos \left( \theta+\frac{4\pi}{3} \right)} \right) + \left( \frac{k}{\cos \left( \theta+\frac{4\pi}{3} \right)} \right) \left( \frac{k}{\cos \theta} \right)$
$xy + yz + zx = k^2 \left[ \frac{1}{\cos \theta \cos \left( \theta+\frac{2\pi}{3} \right)} + \frac{1}{\cos \left( \theta+\frac{2\pi}{3} \right) \cos \left( \theta+\frac{4\pi}{3} \right)} + \frac{1}{\cos \left( \theta+\frac{4\pi}{3} \right) \cos \theta} \right]$
Find a common denominator for the terms inside the bracket:
Common denominator is $\cos \theta \cos \left( \theta+\frac{2\pi}{3} \right) \cos \left( \theta+\frac{4\pi}{3} \right)$.
The expression inside the bracket becomes:
$\frac{\cos \left( \theta+\frac{4\pi}{3} \right) + \cos \theta + \cos \left( \theta+\frac{2\pi}{3} \right)}{\cos \theta \cos \left( \theta+\frac{2\pi}{3} \right) \cos \left( \theta+\frac{4\pi}{3} \right)}$
Consider the sum of the cosines in the numerator:
$S = \cos \theta + \cos \left( \theta+\frac{2\pi}{3} \right) + \cos \left( \theta+\frac{4\pi}{3} \right)$
Using the formula $\cos A + \cos B = 2 \cos \frac{A+B}{2} \cos \frac{A-B}{2}$:
$S = \cos \theta + \left[ \cos \left( \theta+\frac{2\pi}{3} \right) + \cos \left( \theta+\frac{4\pi}{3} \right) \right]$
$S = \cos \theta + 2 \cos \left( \frac{\theta+\frac{2\pi}{3} + \theta+\frac{4\pi}{3}}{2} \right) \cos \left( \frac{\theta+\frac{2\pi}{3} - (\theta+\frac{4\pi}{3})}{2} \right)$
$S = \cos \theta + 2 \cos \left( \frac{2\theta+\frac{6\pi}{3}}{2} \right) \cos \left( \frac{-\frac{2\pi}{3}}{2} \right)$
$S = \cos \theta + 2 \cos \left( \frac{2\theta+2\pi}{2} \right) \cos \left( -\frac{\pi}{3} \right)$
$S = \cos \theta + 2 \cos \left( \theta+\pi \right) \cos \left( \frac{\pi}{3} \right)$ $\quad$ (since $\cos(-x) = \cos x$)
Using $\cos(\theta+\pi) = -\cos\theta$ and $\cos(\pi/3) = 1/2$:
$S = \cos \theta + 2 (-\cos \theta) \left( \frac{1}{2} \right)$
$S = \cos \theta - \cos \theta$
$S = 0$
So, the numerator of the bracketed expression is 0.
$xy + yz + zx = k^2 \left[ \frac{0}{\cos \theta \cos \left( \theta+\frac{2\pi}{3} \right) \cos \left( \theta+\frac{4\pi}{3} \right)} \right]$
Provided that the denominator is not zero, which implies $\cos \theta \neq 0$, $\cos(\theta+2\pi/3) \neq 0$, and $\cos(\theta+4\pi/3) \neq 0$. If the denominator is zero, then $x, y,$ or $z$ would be undefined, meaning the initial equality would not hold for a finite non-zero $k$. For a finite $x, y, z$, the denominator must be non-zero.
Therefore, $xy + yz + zx = k^2 \times 0 = 0$.
The value of $xy + yz + zx$ is $\mathbf{0}$.
Example 11: If $α$ and $β$ are the solutions of the equation a tan θ + b sec θ = c, then show that $tan (α + β) = \frac{2ac}{a^{2}-c^{2}}$.
Answer:
Solution:
The given equation is $a \tan \theta + b \sec \theta = c$.
Rearrange the equation to isolate the term with $\tan \theta$:
$a \tan \theta = c - b \sec \theta$
Square both sides of the equation:
$(a \tan \theta)^2 = (c - b \sec \theta)^2$
$a^2 \tan^2 \theta = c^2 - 2bc \sec \theta + b^2 \sec^2 \theta$
Use the identity $\tan^2 \theta = \sec^2 \theta - 1$:
$a^2 (\sec^2 \theta - 1) = c^2 - 2bc \sec \theta + b^2 \sec^2 \theta$
$a^2 \sec^2 \theta - a^2 = c^2 - 2bc \sec \theta + b^2 \sec^2 \theta$
Rearrange the terms to form a quadratic equation in $\sec \theta$:
$a^2 \sec^2 \theta - b^2 \sec^2 \theta + 2bc \sec \theta - a^2 - c^2 = 0$
$(a^2 - b^2) \sec^2 \theta + (2bc) \sec \theta - (a^2 + c^2) = 0$
This is a quadratic equation in the variable $\sec \theta$. Let $u = \sec \theta$. The equation is $(a^2 - b^2) u^2 + (2bc) u - (a^2 + c^2) = 0$.
Since $\alpha$ and $\beta$ are the solutions to the original equation $a \tan \theta + b \sec \theta = c$, it means that $\sec \alpha$ and $\sec \beta$ are the roots of this quadratic equation, provided $\cos \alpha \neq 0$ and $\cos \beta \neq 0$.
By Vieta's formulas, for a quadratic equation $Ax^2 + Bx + C = 0$ with roots $x_1, x_2$:
Sum of roots: $x_1 + x_2 = -\frac{B}{A}$
Product of roots: $x_1 x_2 = \frac{C}{A}$
Applying Vieta's formulas to the quadratic in $\sec \theta$:
Sum of roots: $\sec \alpha + \sec \beta = - \frac{2bc}{a^2 - b^2}$
Product of roots: $\sec \alpha \sec \beta = \frac{-(a^2 + c^2)}{a^2 - b^2} = \frac{a^2 + c^2}{b^2 - a^2}$
From the original equation, for $\theta = \alpha$ and $\theta = \beta$:
$a \tan \alpha + b \sec \alpha = c \implies \tan \alpha = \frac{c - b \sec \alpha}{a}$
$a \tan \beta + b \sec \beta = c \implies \tan \beta = \frac{c - b \sec \beta}{a}$
Now, calculate the sum $\tan \alpha + \tan \beta$:
$\tan \alpha + \tan \beta = \frac{c - b \sec \alpha}{a} + \frac{c - b \sec \beta}{a}$
$\tan \alpha + \tan \beta = \frac{2c - b (\sec \alpha + \sec \beta)}{a}$
Substitute the sum of secants:
$\tan \alpha + \tan \beta = \frac{2c - b \left( -\frac{2bc}{a^2 - b^2} \right)}{a} = \frac{2c + \frac{2b^2c}{a^2 - b^2}}{a}$
$\tan \alpha + \tan \beta = \frac{\frac{2c(a^2 - b^2) + 2b^2c}{a^2 - b^2}}{a} = \frac{\frac{2a^2c - 2b^2c + 2b^2c}{a^2 - b^2}}{a} = \frac{\frac{2a^2c}{a^2 - b^2}}{a}$
$\tan \alpha + \tan \beta = \frac{2a^2c}{a(a^2 - b^2)} = \frac{2ac}{a^2 - b^2}$
Next, calculate the product $\tan \alpha \tan \beta$:
$\tan \alpha \tan \beta = \left( \frac{c - b \sec \alpha}{a} \right) \left( \frac{c - b \sec \beta}{a} \right)$
$\tan \alpha \tan \beta = \frac{c^2 - bc (\sec \alpha + \sec \beta) + b^2 \sec \alpha \sec \beta}{a^2}$
Substitute the sum and product of secants:
$\tan \alpha \tan \beta = \frac{c^2 - bc \left( -\frac{2bc}{a^2 - b^2} \right) + b^2 \left( \frac{a^2 + c^2}{b^2 - a^2} \right)}{a^2}$
$\tan \alpha \tan \beta = \frac{c^2 + \frac{2b^2c^2}{a^2 - b^2} - \frac{b^2(a^2 + c^2)}{a^2 - b^2}}{a^2}$ $\quad$ (since $b^2 - a^2 = -(a^2 - b^2)$)
$\tan \alpha \tan \beta = \frac{\frac{c^2(a^2 - b^2) + 2b^2c^2 - b^2(a^2 + c^2)}{a^2 - b^2}}{a^2}$
$\tan \alpha \tan \beta = \frac{a^2c^2 - b^2c^2 + 2b^2c^2 - a^2b^2 - b^2c^2}{a^2(a^2 - b^2)}$
$\tan \alpha \tan \beta = \frac{a^2c^2 - a^2b^2}{a^2(a^2 - b^2)} = \frac{a^2(c^2 - b^2)}{a^2(a^2 - b^2)} = \frac{c^2 - b^2}{a^2 - b^2}$
Now, use the tangent addition formula:
$\tan (\alpha + \beta) = \frac{\tan \alpha + \tan \beta}{1 - \tan \alpha \tan \beta}$
Substitute the calculated values for the sum and product of tangents:
$\tan (\alpha + \beta) = \frac{\frac{2ac}{a^2 - b^2}}{1 - \frac{c^2 - b^2}{a^2 - b^2}}$
$\tan (\alpha + \beta) = \frac{\frac{2ac}{a^2 - b^2}}{\frac{(a^2 - b^2) - (c^2 - b^2)}{a^2 - b^2}}$
$\tan (\alpha + \beta) = \frac{\frac{2ac}{a^2 - b^2}}{\frac{a^2 - b^2 - c^2 + b^2}{a^2 - b^2}}$
$\tan (\alpha + \beta) = \frac{\frac{2ac}{a^2 - b^2}}{\frac{a^2 - c^2}{a^2 - b^2}}$
$\tan (\alpha + \beta) = \frac{2ac}{a^2 - b^2} \times \frac{a^2 - b^2}{a^2 - c^2}$
$\tan (\alpha + \beta) = \frac{2ac}{a^2 - c^2}$
Thus, we have shown that $\tan (\alpha + \beta) = \frac{2ac}{a^{2}-c^{2}}$.
Example 12: Show that 2 sin2 $β$ + 4 cos ($α$ + $β$) sin $α$ sin $β$ + cos 2 ($α$ + $β$) = cos 2$α$
Answer:
Solution:
To Prove: $2 \sin^2 \beta + 4 \cos (\alpha + \beta) \sin \alpha \sin \beta + \cos 2 (\alpha + \beta) = \cos 2\alpha$
Consider the Left Hand Side (LHS):
LHS = $2 \sin^2 \beta + 4 \cos (\alpha + \beta) \sin \alpha \sin \beta + \cos 2 (\alpha + \beta)$
Use the product-to-sum identity: $2 \sin A \sin B = \cos(A-B) - \cos(A+B)$.
So, $4 \sin \alpha \sin \beta = 2 (2 \sin \alpha \sin \beta) = 2 (\cos(\alpha - \beta) - \cos(\alpha + \beta))$.
Substitute this into the LHS:
LHS = $2 \sin^2 \beta + \cos (\alpha + \beta) \times [2 (\cos(\alpha - \beta) - \cos(\alpha + \beta))] + \cos 2 (\alpha + \beta)$
LHS = $2 \sin^2 \beta + 2 \cos (\alpha + \beta) \cos(\alpha - \beta) - 2 \cos^2 (\alpha + \beta) + \cos 2 (\alpha + \beta)$
Use the product-to-sum identity: $2 \cos A \cos B = \cos(A+B) + \cos(A-B)$.
$2 \cos (\alpha + \beta) \cos(\alpha - \beta) = \cos ((\alpha + \beta) + (\alpha - \beta)) + \cos ((\alpha + \beta) - (\alpha - \beta))$
$= \cos (2\alpha) + \cos (2\beta)$
Substitute this into the expression for LHS:
LHS = $2 \sin^2 \beta + [\cos (2\alpha) + \cos (2\beta)] - 2 \cos^2 (\alpha + \beta) + \cos 2 (\alpha + \beta)$
Use the double angle identity: $\cos 2x = 2 \cos^2 x - 1$.
Rearrange this to get $2 \cos^2 x = \cos 2x + 1$.
So, $2 \cos^2 (\alpha + \beta) = \cos (2(\alpha + \beta)) + 1$.
Substitute this into the LHS:
LHS = $2 \sin^2 \beta + \cos (2\alpha) + \cos (2\beta) - [\cos (2(\alpha + \beta)) + 1] + \cos 2 (\alpha + \beta)$
LHS = $2 \sin^2 \beta + \cos (2\alpha) + \cos (2\beta) - \cos (2(\alpha + \beta)) - 1 + \cos 2 (\alpha + \beta)$
The terms $-\cos (2(\alpha + \beta))$ and $+\cos 2 (\alpha + \beta)$ cancel out.
LHS = $2 \sin^2 \beta + \cos (2\alpha) + \cos (2\beta) - 1$
Use the double angle identity: $\cos 2x = 1 - 2 \sin^2 x$.
Rearrange this to get $2 \sin^2 x = 1 - \cos 2x$.
So, $2 \sin^2 \beta = 1 - \cos 2\beta$.
Substitute this into the expression for LHS:
LHS = $(1 - \cos 2\beta) + \cos (2\alpha) + \cos (2\beta) - 1$
LHS = $1 - \cos 2\beta + \cos 2\alpha + \cos 2\beta - 1$
The terms $1$ and $-1$ cancel out, and $-\cos 2\beta$ and $+\cos 2\beta$ cancel out.
LHS = $\cos 2\alpha$
This is equal to the Right Hand Side (RHS).
Therefore, $2 \sin^2 \beta + 4 \cos (\alpha + \beta) \sin \alpha \sin \beta + \cos 2 (\alpha + \beta) = \cos 2\alpha$.
Hence proved.
Example 13: If angle θ is divided into two parts such that the tangent of one part is k times the tangent of other, and φ is their difference, then show that
$sin \; θ = \frac{k\;+\;1}{k\;-\;1} \;\sin\phi$
Answer:
Solution:
Let the angle $\theta$ be divided into two parts, say $A$ and $B$.
Then, we have:
$A + B = \theta$
The difference between the two parts is $\phi$. Assuming $A \ge B$, we have:
$A - B = \phi$
(If $B > A$, then $B - A = \phi$, which means $A - B = -\phi$. Since $\sin(-\phi) = -\sin\phi$ and the factor $\frac{k+1}{k-1}$ can be negative, the result will hold regardless of which part is larger).
The tangent of one part is $k$ times the tangent of the other part. Let's assume $\tan A = k \tan B$.
Given: $\tan A = k \tan B$
We need to show that $\sin \theta = \frac{k+1}{k-1} \sin \phi$.
Let's express $\sin \theta$ and $\sin \phi$ in terms of $A$ and $B$:
$\sin \theta = \sin(A + B) = \sin A \cos B + \cos A \sin B$
$\sin \phi = \sin(A - B) = \sin A \cos B - \cos A \sin B$
From the given relation $\tan A = k \tan B$, we can write:
$\frac{\sin A}{\cos A} = k \frac{\sin B}{\cos B}$
Cross-multiplying gives:
$\sin A \cos B = k \cos A \sin B$
... (i)
Substitute equation (i) into the expressions for $\sin \theta$ and $\sin \phi$.
For $\sin \theta$:
$\sin \theta = \sin A \cos B + \cos A \sin B = (k \cos A \sin B) + \cos A \sin B$
$\sin \theta = (k + 1) \cos A \sin B$
... (ii)
For $\sin \phi$:
$\sin \phi = \sin A \cos B - \cos A \sin B = (k \cos A \sin B) - \cos A \sin B$
$\sin \phi = (k - 1) \cos A \sin B$
... (iii)
Now, divide equation (ii) by equation (iii):
$\frac{\sin \theta}{\sin \phi} = \frac{(k + 1) \cos A \sin B}{(k - 1) \cos A \sin B}$
Assuming $\cos A \sin B \neq 0$ (i.e., $A \neq m\pi + \pi/2$ and $B \neq n\pi$ for integers $m, n$, which implies $\tan A$ and $\tan B$ are well-defined and $A \neq B$), we can cancel the term $\cos A \sin B$ from the numerator and denominator.
$\frac{\sin \theta}{\sin \phi} = \frac{k + 1}{k - 1}$
Multiply both sides by $\sin \phi$ (assuming $\sin \phi \neq 0$, which means $\phi \neq n\pi$, so $A \neq B$):
$\sin \theta = \frac{k + 1}{k - 1} \sin \phi$
This is the required result.
Alternate Approach (using Componendo and Dividendo):
Given $\tan A = k \tan B$. This can be written as:
$\frac{\tan A}{\tan B} = k = \frac{k}{1}$
Apply Componendo and Dividendo:
$\frac{\tan A + \tan B}{\tan A - \tan B} = \frac{k + 1}{k - 1}$
Consider the ratio of $\sin(A+B)$ and $\sin(A-B)$:
$\frac{\sin(A+B)}{\sin(A-B)} = \frac{\sin A \cos B + \cos A \sin B}{\sin A \cos B - \cos A \sin B}$
Divide the numerator and denominator by $\cos A \cos B$ (assuming $\cos A \neq 0$ and $\cos B \neq 0$):
$\frac{\frac{\sin A \cos B}{\cos A \cos B} + \frac{\cos A \sin B}{\cos A \cos B}}{\frac{\sin A \cos B}{\cos A \cos B} - \frac{\cos A \sin B}{\cos A \cos B}} = \frac{\frac{\sin A}{\cos A} + \frac{\sin B}{\cos B}}{\frac{\sin A}{\cos A} - \frac{\sin B}{\cos B}} = \frac{\tan A + \tan B}{\tan A - \tan B}$
So, we have:
$\frac{\sin(A+B)}{\sin(A-B)} = \frac{\tan A + \tan B}{\tan A - \tan B}$
Substitute the result from applying Componendo and Dividendo to $\frac{\tan A}{\tan B}$:
$\frac{\sin(A+B)}{\sin(A-B)} = \frac{k+1}{k-1}$
Given $A+B = \theta$ and $A-B = \phi$. Substitute these into the equation:
$\frac{\sin \theta}{\sin \phi} = \frac{k+1}{k-1}$
Multiply by $\sin \phi$:
$\sin \theta = \frac{k+1}{k-1} \sin \phi$
This confirms the result.
Hence showed.
Example 14: Solve $\sqrt{3}$ cos θ + sin θ = $\sqrt{2}$
Answer:
Solution:
The given equation is $\sqrt{3} \cos \theta + \sin \theta = \sqrt{2}$.
This equation is in the form $a \cos \theta + b \sin \theta = c$, where $a = \sqrt{3}$, $b = 1$, and $c = \sqrt{2}$.
To solve this, we can convert the left-hand side into the form $R \cos (\theta - \alpha)$ or $R \sin (\theta + \alpha)$.
Let's use $R \cos (\theta - \alpha) = R (\cos \theta \cos \alpha + \sin \theta \sin \alpha)$.
Comparing this with $\sqrt{3} \cos \theta + 1 \sin \theta$, we need:
$R \cos \alpha = \sqrt{3}$
$R \sin \alpha = 1$
Square and add the two equations to find $R$:
$(R \cos \alpha)^2 + (R \sin \alpha)^2 = (\sqrt{3})^2 + 1^2$
$R^2 \cos^2 \alpha + R^2 \sin^2 \alpha = 3 + 1$
$R^2 (\cos^2 \alpha + \sin^2 \alpha) = 4$
$R^2 (1) = 4$
$R = \sqrt{4} = 2$ $\quad$ (Since $R$ is a magnitude, we take the positive root)
Now find $\alpha$ using $R \cos \alpha = \sqrt{3}$ and $R \sin \alpha = 1$:
$2 \cos \alpha = \sqrt{3} \implies \cos \alpha = \frac{\sqrt{3}}{2}$
$2 \sin \alpha = 1 \implies \sin \alpha = \frac{1}{2}$
Since both $\cos \alpha$ and $\sin \alpha$ are positive, $\alpha$ is in the first quadrant. The angle $\alpha$ that satisfies these conditions is $\frac{\pi}{6}$ (or $30^\circ$).
So, $\alpha = \frac{\pi}{6}$.
The equation $\sqrt{3} \cos \theta + \sin \theta = \sqrt{2}$ can be written as:
$2 \left( \frac{\sqrt{3}}{2} \cos \theta + \frac{1}{2} \sin \theta \right) = \sqrt{2}$
$2 \left( \cos \frac{\pi}{6} \cos \theta + \sin \frac{\pi}{6} \sin \theta \right) = \sqrt{2}$
Using the formula $\cos A \cos B + \sin A \sin B = \cos (A - B)$:
$2 \cos \left( \theta - \frac{\pi}{6} \right) = \sqrt{2}$
Divide by 2:
$\cos \left( \theta - \frac{\pi}{6} \right) = \frac{\sqrt{2}}{2} = \frac{1}{\sqrt{2}}$
We know that $\cos \left( \frac{\pi}{4} \right) = \frac{1}{\sqrt{2}}$.
The general solution for $\cos x = \cos y$ is $x = 2n\pi \pm y$, where $n$ is an integer.
So, $\theta - \frac{\pi}{6} = 2n\pi \pm \frac{\pi}{4}$, where $n \in \mathbb{Z}$.
We have two cases:
Case 1: $\theta - \frac{\pi}{6} = 2n\pi + \frac{\pi}{4}$
$\theta = 2n\pi + \frac{\pi}{4} + \frac{\pi}{6}$
$\theta = 2n\pi + \frac{3\pi + 2\pi}{12}$
$\theta = 2n\pi + \frac{5\pi}{12}$
Case 2: $\theta - \frac{\pi}{6} = 2n\pi - \frac{\pi}{4}$
$\theta = 2n\pi - \frac{\pi}{4} + \frac{\pi}{6}$
$\theta = 2n\pi + \frac{-3\pi + 2\pi}{12}$
$\theta = 2n\pi - \frac{\pi}{12}$
The general solution for the equation is $\theta = 2n\pi + \frac{5\pi}{12}$ or $\theta = 2n\pi - \frac{\pi}{12}$, where $n$ is any integer.
We can write the solution concisely as:
$\theta = 2n\pi + \frac{5\pi}{12} \quad \text{or} \quad \theta = 2n\pi - \frac{\pi}{12}, \quad n \in \mathbb{Z}$
Example 15 to 19 (Multiple Choice Questions)
Choose the correct answer from the given four options against each of the Examples 15 to 19
Example 15: If tan θ = $\frac{-4}{3}$, then sin θ is
(A) $\frac{-4}{5}$ but not $\frac{4}{5}$
(B) $\frac{-4}{5}$ or $\frac{4}{5}$
(C) $\frac{4}{5}$ but not - $\frac{4}{5}$
(D) None of these
Answer:
Solution:
Given that $\tan \theta = \frac{-4}{3}$.
The tangent of an angle is negative in the second quadrant (Q2) and the fourth quadrant (Q4).
We know that $\tan \theta = \frac{\sin \theta}{\cos \theta}$.
Consider a right-angled triangle with opposite side $4$ and adjacent side $3$. The hypotenuse is $\sqrt{4^2 + 3^2} = \sqrt{16 + 9} = \sqrt{25} = 5$.
The sign of $\sin \theta$ depends on the quadrant.
Case 1: $\theta$ is in the second quadrant (Q2).
In Q2, $\sin \theta$ is positive and $\cos \theta$ is negative.
$\tan \theta = \frac{\text{positive}}{\text{negative}} = \text{negative}$. This matches the given $\tan \theta = \frac{-4}{3}$.
In this quadrant, $\sin \theta = \frac{\text{opposite}}{\text{hypotenuse}} = \frac{4}{5}$.
Case 2: $\theta$ is in the fourth quadrant (Q4).
In Q4, $\sin \theta$ is negative and $\cos \theta$ is positive.
$\tan \theta = \frac{\text{negative}}{\text{positive}} = \text{negative}$. This matches the given $\tan \theta = \frac{-4}{3}$.
In this quadrant, $\sin \theta = \frac{\text{opposite}}{\text{hypotenuse}} = \frac{-4}{5}$.
Since $\tan \theta = \frac{-4}{3}$ implies that $\theta$ can be in either the second or the fourth quadrant, $\sin \theta$ can take two possible values: $\frac{4}{5}$ (from Q2) or $\frac{-4}{5}$ (from Q4).
Therefore, $\sin \theta$ is $\frac{-4}{5}$ or $\frac{4}{5}$.
Comparing with the given options:
(A) $\frac{-4}{5}$ but not $\frac{4}{5}$ - Incorrect.
(B) $\frac{-4}{5}$ or $\frac{4}{5}$ - Correct.
(C) $\frac{4}{5}$ but not - $\frac{4}{5}$ - Incorrect.
(D) None of these - Incorrect.
The correct option is (B).
Example 16: If sin θ and cos θ are the roots of the equation ax2 – bx + c = 0, then a, b and c satisfy the relation.
(A) a2 + b2 + 2ac = 0
(B) a2 – b2 + 2ac = 0
(C) a2 + c2 + 2ab = 0
(D) a2 – b2 – 2ac = 0
Answer:
Solution:
The given quadratic equation is $ax^2 - bx + c = 0$.
The roots of this equation are $\sin \theta$ and $\cos \theta$.
Using Vieta's formulas, the sum of the roots is given by $\frac{-(-b)}{a} = \frac{b}{a}$.
$\sin \theta + \cos \theta = \frac{b}{a}$
... (i)
The product of the roots is given by $\frac{c}{a}$.
$\sin \theta \cos \theta = \frac{c}{a}$
... (ii)
We know the fundamental trigonometric identity:
$\sin^2 \theta + \cos^2 \theta = 1$
... (iii)
Square both sides of equation (i):
$(\sin \theta + \cos \theta)^2 = \left( \frac{b}{a} \right)^2$
$\sin^2 \theta + \cos^2 \theta + 2 \sin \theta \cos \theta = \frac{b^2}{a^2}$
Substitute the values from equations (ii) and (iii) into the squared equation:
$1 + 2 \left( \frac{c}{a} \right) = \frac{b^2}{a^2}$
$1 + \frac{2c}{a} = \frac{b^2}{a^2}$
Multiply the entire equation by $a^2$ to clear the denominators:
$a^2 \times 1 + a^2 \times \frac{2c}{a} = a^2 \times \frac{b^2}{a^2}$
$a^2 + 2ac = b^2$
Rearrange the terms to match the options:
$a^2 - b^2 + 2ac = 0$
This is the required relation between $a, b,$ and $c$.
Compare the derived relation with the given options:
(A) $a^2 + b^2 + 2ac = 0$ - Incorrect
(B) $a^2 – b^2 + 2ac = 0$ - Correct
(C) $a^2 + c^2 + 2ab = 0$ - Incorrect
(D) $a^2 – b^2 – 2ac = 0$ - Incorrect
The correct answer is (B).
Example 17: The greatest value of sin x cos x is
(A) 1
(B) 2
(C) $\sqrt{2}$
(D) $\frac{1}{2}$
Answer:
Solution:
We want to find the greatest value of the expression $\sin x \cos x$.
We can use the double angle identity for sine, which states:
$\sin 2x = 2 \sin x \cos x$
From this identity, we can express $\sin x \cos x$ as:
$\sin x \cos x = \frac{1}{2} \sin 2x$
The sine function, $\sin \theta$, has a maximum value of $1$ and a minimum value of $-1$ for any real angle $\theta$.
So, the range of $\sin 2x$ is $[-1, 1]$.
$-1 \leq \sin 2x \leq 1$
To find the range of $\frac{1}{2} \sin 2x$, multiply the inequality by $\frac{1}{2}$:
$\frac{1}{2} \times (-1) \leq \frac{1}{2} \sin 2x \leq \frac{1}{2} \times 1$
$-\frac{1}{2} \leq \sin x \cos x \leq \frac{1}{2}$
The maximum value of $\sin x \cos x$ is the upper bound of this inequality, which is $\frac{1}{2}$.
The greatest value is $\mathbf{\frac{1}{2}}$.
Comparing with the given options:
(A) 1 - Incorrect
(B) 2 - Incorrect
(C) $\sqrt{2}$ - Incorrect
(D) $\frac{1}{2}$ - Correct
The correct answer is (D).
Eaxmple 18: The value of sin 20° sin 40° sin 60° sin 80° is
(A) $\frac{-3}{16}$
(B) $\frac{5}{16}$
(C) $\frac{3}{16}$
(D) $\frac{1}{16}$
Answer:
Solution:
We need to find the value of the expression $\sin 20^\circ \sin 40^\circ \sin 60^\circ \sin 80^\circ$.
We know the exact value of $\sin 60^\circ = \frac{\sqrt{3}}{2}$.
The expression becomes $\sin 20^\circ \sin 40^\circ \left( \frac{\sqrt{3}}{2} \right) \sin 80^\circ$.
Rearranging the terms:
Expression $= \frac{\sqrt{3}}{2} (\sin 20^\circ \sin 40^\circ \sin 80^\circ)$
We can use the product-to-sum formula $2 \sin A \sin B = \cos(A-B) - \cos(A+B)$ or a specific identity for the product of sines involving angles in an arithmetic progression or related to $60^\circ$.
Consider the identity: $\sin A \sin(60^\circ - A) \sin(60^\circ + A) = \frac{1}{4} \sin 3A$.
Let $A = 20^\circ$. Then:
$60^\circ - A = 60^\circ - 20^\circ = 40^\circ$
$60^\circ + A = 60^\circ + 20^\circ = 80^\circ$
So, the product $\sin 20^\circ \sin 40^\circ \sin 80^\circ$ fits the pattern $\sin A \sin(60^\circ - A) \sin(60^\circ + A)$ with $A = 20^\circ$.
Applying the identity:
$\sin 20^\circ \sin 40^\circ \sin 80^\circ = \frac{1}{4} \sin (3 \times 20^\circ) = \frac{1}{4} \sin 60^\circ$
Substitute this back into the original expression:
Expression $= \frac{\sqrt{3}}{2} \times (\sin 20^\circ \sin 40^\circ \sin 80^\circ)$
Expression $= \frac{\sqrt{3}}{2} \times \left( \frac{1}{4} \sin 60^\circ \right)$
Expression $= \frac{\sqrt{3}}{8} \sin 60^\circ$
Substitute the value $\sin 60^\circ = \frac{\sqrt{3}}{2}$:
Expression $= \frac{\sqrt{3}}{8} \times \frac{\sqrt{3}}{2}$
Expression $= \frac{\sqrt{3} \times \sqrt{3}}{8 \times 2} = \frac{3}{16}$
The value of $\sin 20^\circ \sin 40^\circ \sin 60^\circ \sin 80^\circ$ is $\frac{3}{16}$.
Compare with the given options:
(A) $\frac{-3}{16}$ - Incorrect
(B) $\frac{5}{16}$ - Incorrect
(C) $\frac{3}{16}$ - Correct
(D) $\frac{1}{16}$ - Incorrect
The correct answer is (C).
Example 19: The value of cos $\frac{\pi}{5}$ cos $\frac{2\pi}{5}$ cos $\frac{4\pi}{5}$ cos $\frac{8\pi}{5}$
(A) $\frac{1}{16}$
(B) 0
(C) $\frac{-1}{8}$
(D) $\frac{-1}{16}$
Answer:
Solution:
We need to find the value of the expression $\cos \frac{\pi}{5} \cos \frac{2\pi}{5} \cos \frac{4\pi}{5} \cos \frac{8\pi}{5}$.
We use the general formula for the product of cosines:
$\cos x \cos 2x \cos 4x \dots \cos 2^{n-1} x = \frac{\sin (2^n x)}{2^n \sin x}$
In this problem, $x = \frac{\pi}{5}$ and there are $n=4$ terms in the product.
The expression is $\cos x \cos 2x \cos 2^2 x \cos 2^3 x$.
Applying the formula with $x = \frac{\pi}{5}$ and $n=4$:
Value $= \frac{\sin \left( 2^4 \times \frac{\pi}{5} \right)}{2^4 \sin \frac{\pi}{5}}$
Value $= \frac{\sin \left( \frac{16\pi}{5} \right)}{16 \sin \frac{\pi}{5}}$
Now, let's simplify the term $\sin \left( \frac{16\pi}{5} \right)$:
$\frac{16\pi}{5} = \frac{10\pi + 6\pi}{5} = 2\pi + \frac{6\pi}{5}$
Using the periodicity of the sine function, $\sin(2\pi + \theta) = \sin \theta$:
$\sin \left( \frac{16\pi}{5} \right) = \sin \left( 2\pi + \frac{6\pi}{5} \right) = \sin \frac{6\pi}{5}$
Now, simplify $\sin \frac{6\pi}{5}$:
$\frac{6\pi}{5} = \pi + \frac{\pi}{5}$
Using the identity $\sin(\pi + \theta) = -\sin \theta$:
$\sin \frac{6\pi}{5} = \sin \left( \pi + \frac{\pi}{5} \right) = -\sin \frac{\pi}{5}$
Substitute this back into the expression for the value:
Value $= \frac{-\sin \frac{\pi}{5}}{16 \sin \frac{\pi}{5}}$
Assuming $\sin \frac{\pi}{5} \neq 0$, which is true since $\frac{\pi}{5}$ is not a multiple of $\pi$:
Value $= \frac{-1}{16}$
The value of the expression is $\mathbf{\frac{-1}{16}}$.
Comparing with the given options:
(A) $\frac{1}{16}$ - Incorrect
(B) 0 - Incorrect
(C) $\frac{-1}{8}$ - Incorrect
(D) $\frac{-1}{16}$ - Correct
The correct answer is (D).
Example 20 (Fill in the Blanks)
Example 20: If 3 tan (θ – 15°) = tan (θ + 15°), 0° < θ < 90°, then θ = _________
Answer:
Solution:
The given equation is $3 \tan (\theta - 15^\circ) = \tan (\theta + 15^\circ)$.
Rearrange the equation to form a ratio:
$\frac{\tan (\theta + 15^\circ)}{\tan (\theta - 15^\circ)} = \frac{3}{1}$
Apply the Componendo and Dividendo rule, which states that if $\frac{a}{b} = \frac{c}{d}$, then $\frac{a+b}{a-b} = \frac{c+d}{c-d}$.
$\frac{\tan (\theta + 15^\circ) + \tan (\theta - 15^\circ)}{\tan (\theta + 15^\circ) - \tan (\theta - 15^\circ)} = \frac{3 + 1}{3 - 1}$
$\frac{\tan (\theta + 15^\circ) + \tan (\theta - 15^\circ)}{\tan (\theta + 15^\circ) - \tan (\theta - 15^\circ)} = \frac{4}{2} = 2$
Use the identity $\frac{\tan A + \tan B}{\tan A - \tan B} = \frac{\sin(A+B)}{\sin(A-B)}$.
Let $A = \theta + 15^\circ$ and $B = \theta - 15^\circ$.
$A + B = (\theta + 15^\circ) + (\theta - 15^\circ) = 2\theta$
$A - B = (\theta + 15^\circ) - (\theta - 15^\circ) = 30^\circ$
Substitute these into the equation:
$\frac{\sin(2\theta)}{\sin(30^\circ)} = 2$
We know that $\sin(30^\circ) = \frac{1}{2}$.
$\frac{\sin(2\theta)}{\frac{1}{2}} = 2$
$2 \sin(2\theta) = 2$
$\sin(2\theta) = 1$
The general solution for $\sin x = 1$ is $x = 2n\pi + \frac{\pi}{2}$ for $n \in \mathbb{Z}$, or in degrees, $x = 360^\circ n + 90^\circ$.
So, $2\theta = 360^\circ n + 90^\circ$.
Divide by 2:
$\theta = 180^\circ n + 45^\circ$
We are given the condition $0^\circ < \theta < 90^\circ$.
For $n=0$:
$\theta = 180^\circ(0) + 45^\circ = 45^\circ$
This value $45^\circ$ lies within the range $0^\circ < \theta < 90^\circ$.
For $n=1$:
$\theta = 180^\circ(1) + 45^\circ = 225^\circ$
This value is outside the range.
For $n=-1$:
$\theta = 180^\circ(-1) + 45^\circ = -135^\circ$
This value is outside the range.
The only solution in the given range is $\theta = 45^\circ$.
The value of $\theta$ is $\mathbf{45^\circ}$.
Example 21 (True or False)
Example 21: “The inequality $2^{\sin\theta} + 2^{\cos\theta} \geq 2^{1-\frac{1}{\sqrt{2}}}$ holds for all real values of θ”
Answer:
Solution:
The given inequality is $2^{\sin\theta} + 2^{\cos\theta} \geq 2^{1-\frac{1}{\sqrt{2}}}$.
We need to determine if this inequality holds for all real values of $\theta$.
Consider the expression on the left-hand side: $2^{\sin\theta} + 2^{\cos\theta}$.
We know that the range of $\sin\theta$ and $\cos\theta$ is $[-1, 1]$.
Consider the sum $\sin\theta + \cos\theta$. We can write this as:
$\sin\theta + \cos\theta = \sqrt{1^2 + 1^2} \left( \frac{1}{\sqrt{2}} \sin\theta + \frac{1}{\sqrt{2}} \cos\theta \right)$
$= \sqrt{2} \left( \cos \frac{\pi}{4} \sin\theta + \sin \frac{\pi}{4} \cos\theta \right)$
$= \sqrt{2} \sin \left( \theta + \frac{\pi}{4} \right)$
The range of $\sin \left( \theta + \frac{\pi}{4} \right)$ is $[-1, 1]$.
So, the range of $\sin\theta + \cos\theta$ is $[-\sqrt{2}, \sqrt{2}]$.
The minimum value of $\sin\theta + \cos\theta$ is $-\sqrt{2}$, which occurs when $\sin \left( \theta + \frac{\pi}{4} \right) = -1$, i.e., $\theta + \frac{\pi}{4} = \frac{3\pi}{2} + 2n\pi$, or $\theta = \frac{5\pi}{4} + 2n\pi$.
When $\theta = \frac{5\pi}{4}$, $\sin\theta = -\frac{1}{\sqrt{2}}$ and $\cos\theta = -\frac{1}{\sqrt{2}}$.
Consider the function $f(t) = 2^t$. This is a convex function because $f''(t) = (\ln 2)^2 2^t > 0$.
By AM-GM inequality, for non-negative numbers $a, b$: $\frac{a+b}{2} \geq \sqrt{ab}$.
Let $a = 2^{\sin\theta}$ and $b = 2^{\cos\theta}$. Both are positive.
$\frac{2^{\sin\theta} + 2^{\cos\theta}}{2} \geq \sqrt{2^{\sin\theta} \cdot 2^{\cos\theta}}$
$\frac{2^{\sin\theta} + 2^{\cos\theta}}{2} \geq \sqrt{2^{\sin\theta + \cos\theta}}$
$2^{\sin\theta} + 2^{\cos\theta} \geq 2 \cdot 2^{\frac{\sin\theta + \cos\theta}{2}}$
$2^{\sin\theta} + 2^{\cos\theta} \geq 2^{1 + \frac{\sin\theta + \cos\theta}{2}}$
The minimum value of the exponent $1 + \frac{\sin\theta + \cos\theta}{2}$ occurs when $\sin\theta + \cos\theta$ is minimum, which is $-\sqrt{2}$.
Minimum exponent $= 1 + \frac{-\sqrt{2}}{2} = 1 - \frac{\sqrt{2}}{2} = 1 - \frac{1}{\sqrt{2}}$.
So, the minimum value of the lower bound given by AM-GM is $2^{1 - \frac{1}{\sqrt{2}}}$.
The AM-GM inequality tells us that $2^{\sin\theta} + 2^{\cos\theta} \geq 2^{1 + \frac{\sin\theta + \cos\theta}{2}}$.
Since $1 + \frac{\sin\theta + \cos\theta}{2} \geq 1 - \frac{1}{\sqrt{2}}$ for all $\theta$, and the base $2$ is greater than 1, we have:
$2^{1 + \frac{\sin\theta + \cos\theta}{2}} \geq 2^{1 - \frac{1}{\sqrt{2}}}$
Combining the AM-GM result with this minimum:
$2^{\sin\theta} + 2^{\cos\theta} \geq 2^{1 + \frac{\sin\theta + \cos\theta}{2}} \geq 2^{1 - \frac{1}{\sqrt{2}}}$
Thus, $2^{\sin\theta} + 2^{\cos\theta} \geq 2^{1 - \frac{1}{\sqrt{2}}}$ holds for all real values of $\theta$.
The equality holds when $\sin\theta = \cos\theta$ AND the exponent $1 + \frac{\sin\theta + \cos\theta}{2}$ is at its minimum. This occurs when $\sin\theta = \cos\theta = -\frac{1}{\sqrt{2}}$, i.e., $\theta = \frac{5\pi}{4} + 2n\pi$.
At $\theta = \frac{5\pi}{4}$, LHS $= 2^{-1/\sqrt{2}} + 2^{-1/\sqrt{2}} = 2 \cdot 2^{-1/\sqrt{2}} = 2^{1 - 1/\sqrt{2}}$, which equals the RHS.
For any other value of $\theta$, $2^{\sin\theta} + 2^{\cos\theta}$ will be greater than the minimum value $2^{1-\frac{1}{\sqrt{2}}}$.
The statement is True.
Example 22 (Match the Following)
Match each item given under the column C1 to its correct answer given under column C2
Example 22:
$C_1$
(a) $\frac{1 - \cos x}{\sin x}$
(b) $\frac{1 + \cos x}{1 - \cos x}$
(c) $\frac{1 + \cos x}{\sin x}$
(d) $\sqrt{1 + \sin 2x}$
$C_2$
(i) $2 \cot^2 \frac{x}{2}$
(ii) $\cot \frac{x}{2}$
(iii) $| \cos x + \sin x |$
(iv) $\tan \frac{x}{2}$
Answer:
Solution:
Let's simplify each expression from Column C1 using standard trigonometric identities.
(a) $\frac{1 - \cos x}{\sin x}$
Using the half-angle identities $1 - \cos x = 2 \sin^2 \frac{x}{2}$ and $\sin x = 2 \sin \frac{x}{2} \cos \frac{x}{2}$:
$\frac{1 - \cos x}{\sin x} = \frac{2 \sin^2 \frac{x}{2}}{2 \sin \frac{x}{2} \cos \frac{x}{2}}$
Assuming $\sin \frac{x}{2} \neq 0$:
$= \frac{\sin \frac{x}{2}}{\cos \frac{x}{2}} = \tan \frac{x}{2}$
This matches item (iv) in Column C2.
(b) $\frac{1 + \cos x}{1 - \cos x}$
Using the half-angle identities $1 + \cos x = 2 \cos^2 \frac{x}{2}$ and $1 - \cos x = 2 \sin^2 \frac{x}{2}$:
$\frac{1 + \cos x}{1 - \cos x} = \frac{2 \cos^2 \frac{x}{2}}{2 \sin^2 \frac{x}{2}}$
Assuming $\sin \frac{x}{2} \neq 0$:
$= \frac{\cos^2 \frac{x}{2}}{\sin^2 \frac{x}{2}} = \cot^2 \frac{x}{2}$
This matches item (i) in Column C2 (assuming a potential typo in the coefficient in C2(i) as $\cot^2 \frac{x}{2}$ is the standard simplification). Based on the provided options and other matches, C2(i) is the intended correspondence.
(c) $\frac{1 + \cos x}{\sin x}$
Using the half-angle identities $1 + \cos x = 2 \cos^2 \frac{x}{2}$ and $\sin x = 2 \sin \frac{x}{2} \cos \frac{x}{2}$:
$\frac{1 + \cos x}{\sin x} = \frac{2 \cos^2 \frac{x}{2}}{2 \sin \frac{x}{2} \cos \frac{x}{2}}$
Assuming $\cos \frac{x}{2} \neq 0$ and $\sin \frac{x}{2} \neq 0$:
$= \frac{\cos \frac{x}{2}}{\sin \frac{x}{2}} = \cot \frac{x}{2}$
This matches item (ii) in Column C2.
(d) $\sqrt{1 + \sin 2x}$
Using the identity $1 = \sin^2 x + \cos^2 x$ and $\sin 2x = 2 \sin x \cos x$:
$\sqrt{1 + \sin 2x} = \sqrt{\sin^2 x + \cos^2 x + 2 \sin x \cos x}$
$= \sqrt{(\sin x + \cos x)^2}$
$= |\sin x + \cos x|$
This matches item (iii) in Column C2.
Summarizing the matches:
(a) – (iv)
(b) – (i)
(c) – (ii)
(d) – (iii)
Exercise
Question 1 to 19 (Short Answer Type Questions)
Question 1. Prove that $\frac{tan \;A \;+\; sec \;A \;-\; 1}{tan \;A \;-\; sec \;A \;+\; 1} = \frac{1 \;+\; sin \;A}{cos \;A}$
Answer:
Proof:
To Prove: $\frac{\tan A + \sec A - 1}{\tan A - \sec A + 1} = \frac{1 + \sin A}{\cos A}$
Consider the Left Hand Side (LHS):
LHS = $\frac{\tan A + \sec A - 1}{\tan A - \sec A + 1}$
We use the identity $\sec^2 A - \tan^2 A = 1$ in the numerator. Replace the '1' in the numerator with $\sec^2 A - \tan^2 A$.
LHS = $\frac{\tan A + \sec A - (\sec^2 A - \tan^2 A)}{\tan A - \sec A + 1}$
Factor the difference of squares in the numerator: $\sec^2 A - \tan^2 A = (\sec A - \tan A)(\sec A + \tan A)$.
LHS = $\frac{(\tan A + \sec A) - (\sec A - \tan A)(\sec A + \tan A)}{\tan A - \sec A + 1}$
Take out the common factor $(\tan A + \sec A)$ from the numerator:
LHS = $\frac{(\tan A + \sec A) [1 - (\sec A - \tan A)]}{\tan A - \sec A + 1}$
LHS = $\frac{(\tan A + \sec A) [1 - \sec A + \tan A]}{\tan A - \sec A + 1}$
Notice that the term in the square bracket in the numerator, $(1 - \sec A + \tan A)$, is the same as the denominator, $(\tan A - \sec A + 1)$.
Cancel out the common factor $(1 - \sec A + \tan A)$ from the numerator and the denominator, assuming $\tan A - \sec A + 1 \neq 0$.
LHS = $\tan A + \sec A$
Now, express $\tan A$ and $\sec A$ in terms of $\sin A$ and $\cos A$:
LHS = $\frac{\sin A}{\cos A} + \frac{1}{\cos A}$
Combine the terms over a common denominator:
LHS = $\frac{\sin A + 1}{\cos A}$
LHS = $\frac{1 + \sin A}{\cos A}$
This is equal to the Right Hand Side (RHS).
LHS = RHS
Therefore, the identity $\frac{\tan A + \sec A - 1}{\tan A - \sec A + 1} = \frac{1 + \sin A}{\cos A}$ is proved.
Question 2. If $\frac{2 \;sin \;\alpha}{1\;+\;cos\;\alpha\;+\;sin\;\alpha} = y$ , then prove that $\frac{1\;-\;cos\;\alpha\;+\;sin\;\alpha}{1\;+\;sin\;\alpha}$ is equal to y.
$\left[ Hint : Expess \frac{1\;-\;cos\;\alpha\;+\;sin\;\alpha}{1\;+\;sin\;\alpha}=\frac{1\;-\;cos \;\alpha\;+\;sin\;\alpha}{1\;+\;sin\;\alpha}.\frac{1\;+\;cos\;\alpha\;+\;sin\;\alpha}{1\;+\;cos\;\alpha\;+\;sin\;\alpha} \right]$
Answer:
Proof:
Given: $y = \frac{2 \sin \alpha}{1 + \cos \alpha + \sin \alpha}$
To Prove: $\frac{1 - \cos \alpha + \sin \alpha}{1 + \sin \alpha} = y$
Consider the expression on the left side that we need to show is equal to $y$:
Expression $= \frac{1 - \cos \alpha + \sin \alpha}{1 + \sin \alpha}$
Following the hint, multiply the numerator and denominator by $1 + \cos \alpha + \sin \alpha$:
Expression $= \frac{(1 - \cos \alpha + \sin \alpha)}{(1 + \sin \alpha)} \times \frac{(1 + \cos \alpha + \sin \alpha)}{(1 + \cos \alpha + \sin \alpha)}$
Expression $= \frac{(1 + \sin \alpha - \cos \alpha)(1 + \sin \alpha + \cos \alpha)}{(1 + \sin \alpha)(1 + \cos \alpha + \sin \alpha)}$
Consider the numerator. Let $X = 1 + \sin \alpha$. The numerator is in the form $(X - \cos \alpha)(X + \cos \alpha)$.
Numerator $= (1 + \sin \alpha)^2 - \cos^2 \alpha$
$= (1 + 2 \sin \alpha + \sin^2 \alpha) - \cos^2 \alpha$
Use the identity $\cos^2 \alpha = 1 - \sin^2 \alpha$:
Numerator $= 1 + 2 \sin \alpha + \sin^2 \alpha - (1 - \sin^2 \alpha)$
$= 1 + 2 \sin \alpha + \sin^2 \alpha - 1 + \sin^2 \alpha$
$= 2 \sin \alpha + 2 \sin^2 \alpha$
Factor out $2 \sin \alpha$:
Numerator $= 2 \sin \alpha (1 + \sin \alpha)$
Now substitute the simplified numerator back into the expression:
Expression $= \frac{2 \sin \alpha (1 + \sin \alpha)}{(1 + \sin \alpha)(1 + \cos \alpha + \sin \alpha)}$
Assuming $1 + \sin \alpha \neq 0$, we can cancel the term $(1 + \sin \alpha)$ from the numerator and the denominator.
Expression $= \frac{2 \sin \alpha}{1 + \cos \alpha + \sin \alpha}$
This expression is exactly the definition of $y$ given in the problem.
Expression $= y$
Thus, $\frac{1 - \cos \alpha + \sin \alpha}{1 + \sin \alpha} = y$.
Note: If $1 + \sin \alpha = 0$, then $\sin \alpha = -1$, which implies $\cos \alpha = 0$. In this case, the original expression for $y$ becomes $\frac{2(-1)}{1+0+(-1)} = \frac{-2}{0}$, which is undefined. The expression we started with becomes $\frac{1-0+(-1)}{1+(-1)} = \frac{0}{0}$, which is also undefined. The equality holds for all values of $\alpha$ for which both expressions are defined.
Hence proved.
Question 3. If $m \ sin \ θ = n \ sin (θ + 2α)$, then prove that $tan \ (θ + α) \ cot \ α = \frac{m \;+\; n}{m \;-\; n}$
[Hint: Express $\frac{sin \;(\theta\;+\;2\alpha)}{sin \;\theta} = \frac{m}{n}$ and apply componendo and dividendo]
Answer:
Solution:
Given: $m \sin \theta = n \sin (\theta + 2\alpha)$
To Prove: $\tan (\theta + \alpha) \cot \alpha = \frac{m + n}{m - n}$
From the given condition, we can write:
$\frac{\sin (\theta + 2\alpha)}{\sin \theta} = \frac{n}{m}$
Taking the reciprocal of both sides:
$\frac{\sin \theta}{\sin (\theta + 2\alpha)} = \frac{m}{n}$
Apply Componendo and Dividendo:
$\frac{\sin \theta + \sin (\theta + 2\alpha)}{\sin \theta - \sin (\theta + 2\alpha)} = \frac{m + n}{m - n}$
Use the sum-to-product formulas:
$\sin A + \sin B = 2 \sin \left( \frac{A+B}{2} \right) \cos \left( \frac{A-B}{2} \right)$
$\sin A - \sin B = 2 \cos \left( \frac{A+B}{2} \right) \sin \left( \frac{A-B}{2} \right)$
Let $A = \theta + 2\alpha$ and $B = \theta$ for the numerator, and $A = \theta$ and $B = \theta + 2\alpha$ for the denominator on the LHS of the Componendo and Dividendo equation.
Numerator: $\sin \theta + \sin (\theta + 2\alpha) = 2 \sin \left( \frac{\theta + (\theta + 2\alpha)}{2} \right) \cos \left( \frac{\theta - (\theta + 2\alpha)}{2} \right)$
$= 2 \sin \left( \frac{2\theta + 2\alpha}{2} \right) \cos \left( \frac{-2\alpha}{2} \right)$
$= 2 \sin (\theta + \alpha) \cos (-\alpha)$
$= 2 \sin (\theta + \alpha) \cos \alpha$ $\quad$ (since $\cos(-x) = \cos x$)
Denominator: $\sin \theta - \sin (\theta + 2\alpha) = 2 \cos \left( \frac{\theta + (\theta + 2\alpha)}{2} \right) \sin \left( \frac{\theta - (\theta + 2\alpha)}{2} \right)$
$= 2 \cos \left( \frac{2\theta + 2\alpha}{2} \right) \sin \left( \frac{-2\alpha}{2} \right)$
$= 2 \cos (\theta + \alpha) \sin (-\alpha)$
$= -2 \cos (\theta + \alpha) \sin \alpha$ $\quad$ (since $\sin(-x) = -\sin x$)
Substitute these back into the equation from applying Componendo and Dividendo:
$\frac{2 \sin (\theta + \alpha) \cos \alpha}{-2 \cos (\theta + \alpha) \sin \alpha} = \frac{m + n}{m - n}$
Simplify the LHS:
$- \frac{\sin (\theta + \alpha) \cos \alpha}{\cos (\theta + \alpha) \sin \alpha} = \frac{m + n}{m - n}$
$- \left( \frac{\sin (\theta + \alpha)}{\cos (\theta + \alpha)} \right) \left( \frac{\cos \alpha}{\sin \alpha} \right) = \frac{m + n}{m - n}$
$- \tan (\theta + \alpha) \cot \alpha = \frac{m + n}{m - n}$
Multiply both sides by -1:
$\tan (\theta + \alpha) \cot \alpha = - \frac{m + n}{m - n} = \frac{m + n}{-(m - n)} = \frac{m + n}{n - m}$
Let's recheck the Componendo and Dividendo step. If we use $\frac{\sin (\theta + 2\alpha)}{\sin \theta} = \frac{n}{m}$, then $\frac{\sin (\theta + 2\alpha) + \sin \theta}{\sin (\theta + 2\alpha) - \sin \theta} = \frac{n + m}{n - m}$.
Using $A = \theta + 2\alpha$ and $B = \theta$:
LHS = $\frac{2 \sin (\theta + \alpha) \cos \alpha}{2 \cos (\theta + \alpha) \sin \alpha} = \frac{\sin (\theta + \alpha) \cos \alpha}{\cos (\theta + \alpha) \sin \alpha} = \tan (\theta + \alpha) \cot \alpha$.
So, $\tan (\theta + \alpha) \cot \alpha = \frac{n + m}{n - m} = \frac{m + n}{n - m}$. This does not match the required result $\frac{m+n}{m-n}$.
Let's go back to the initial rearrangement as suggested by the hint: $\frac{\sin (\theta + 2\alpha)}{\sin \theta} = \frac{n}{m}$. The hint suggests applying Componendo and Dividendo to $\frac{\sin (\theta + 2\alpha)}{\sin \theta} = \frac{m}{n}$, which seems to imply the initial step should have been $\frac{\sin (\theta + 2\alpha)}{\sin \theta} = \frac{m}{n}$ derived from $n \sin (\theta + 2\alpha) = m \sin \theta$.
Given $m \sin \theta = n \sin (\theta + 2\alpha)$.
Divide by $n \sin \theta$: $\frac{m}{n} = \frac{\sin (\theta + 2\alpha)}{\sin \theta}$.
So, $\frac{\sin (\theta + 2\alpha)}{\sin \theta} = \frac{m}{n}$.
Apply Componendo and Dividendo to $\frac{\sin (\theta + 2\alpha)}{\sin \theta} = \frac{m}{n}$:
$\frac{\sin (\theta + 2\alpha) + \sin \theta}{\sin (\theta + 2\alpha) - \sin \theta} = \frac{m + n}{m - n}$
Using the sum-to-product formulas with $A = \theta + 2\alpha$ and $B = \theta$:
Numerator: $\sin (\theta + 2\alpha) + \sin \theta = 2 \sin \left( \frac{(\theta + 2\alpha) + \theta}{2} \right) \cos \left( \frac{(\theta + 2\alpha) - \theta}{2} \right)$
$= 2 \sin (\theta + \alpha) \cos \alpha$
Denominator: $\sin (\theta + 2\alpha) - \sin \theta = 2 \cos \left( \frac{(\theta + 2\alpha) + \theta}{2} \right) \sin \left( \frac{(\theta + 2\alpha) - \theta}{2} \right)$
$= 2 \cos (\theta + \alpha) \sin \alpha$
Substitute these into the equation:
$\frac{2 \sin (\theta + \alpha) \cos \alpha}{2 \cos (\theta + \alpha) \sin \alpha} = \frac{m + n}{m - n}$
$\frac{\sin (\theta + \alpha) \cos \alpha}{\cos (\theta + \alpha) \sin \alpha} = \frac{m + n}{m - n}$
$\left( \frac{\sin (\theta + \alpha)}{\cos (\theta + \alpha)} \right) \left( \frac{\cos \alpha}{\sin \alpha} \right) = \frac{m + n}{m - n}$
$\tan (\theta + \alpha) \cot \alpha = \frac{m + n}{m - n}$
This matches the required result.
Hence proved.
Question 4. If $cos (α + β) = \frac{4}{5}$ and $sin (α - β) = \frac{5}{13}$, where $α$ lie between 0 and $\frac{\pi}{4}$, find the value of $tan \ 2α$
[Hint: Express $tan \ 2 α$ as $tan \ (α + β + α – β)$]
Answer:
Solution:
Given:
$\cos (\alpha + \beta) = \frac{4}{5}$
$\sin (\alpha - \beta) = \frac{5}{13}$
$0 < \alpha < \frac{\pi}{4}$
To Find: The value of $\tan 2\alpha$.
Let $A = \alpha + \beta$ and $B = \alpha - \beta$.
Then $A + B = (\alpha + \beta) + (\alpha - \beta) = 2\alpha$.
We need to find $\tan 2\alpha = \tan (A + B)$.
Using the tangent addition formula, $\tan (A + B) = \frac{\tan A + \tan B}{1 - \tan A \tan B}$.
From $\cos A = \cos (\alpha + \beta) = \frac{4}{5}$:
Since $0 < \alpha < \frac{\pi}{4}$, we have $0 < 2\alpha < \frac{\pi}{2}$.
$2\alpha = A+B$. Since $0 < 2\alpha < \frac{\pi}{2}$, $\tan(A+B) = \tan(2\alpha)$ must be positive.
From $\cos A = \frac{4}{5}$, we can find $\sin A$. $\sin^2 A = 1 - \cos^2 A = 1 - \left(\frac{4}{5}\right)^2 = 1 - \frac{16}{25} = \frac{9}{25}$.
So, $\sin A = \pm \frac{3}{5}$.
And $\tan A = \frac{\sin A}{\cos A} = \frac{\pm 3/5}{4/5} = \pm \frac{3}{4}$.
From $\sin B = \sin (\alpha - \beta) = \frac{5}{13}$:
We can find $\cos B$. $\cos^2 B = 1 - \sin^2 B = 1 - \left(\frac{5}{13}\right)^2 = 1 - \frac{25}{169} = \frac{144}{169}$.
So, $\cos B = \pm \frac{12}{13}$.
And $\tan B = \frac{\sin B}{\cos B} = \frac{5/13}{\pm 12/13} = \pm \frac{5}{12}$.
Now consider $\tan 2\alpha = \tan(A+B) = \frac{\tan A + \tan B}{1 - \tan A \tan B}$.
Since $0 < 2\alpha < \frac{\pi}{2}$, $\sin 2\alpha > 0$ and $\cos 2\alpha > 0$.
$\cos(A+B) = \cos A \cos B - \sin A \sin B = \left(\frac{4}{5}\right) (\pm \frac{12}{13}) - (\pm \frac{3}{5}) \left(\frac{5}{13}\right) = \frac{\pm 48 \mp 15}{65}$.
For $\cos(A+B)$ to be positive, the numerator $\pm 48 \mp 15$ must be positive.
If we take $\cos B = \frac{12}{13}$ and $\sin A = \frac{3}{5}$, $\cos(A+B) = \frac{48 - 15}{65} = \frac{33}{65} > 0$.
If we take $\cos B = \frac{12}{13}$ and $\sin A = -\frac{3}{5}$, $\cos(A+B) = \frac{48 + 15}{65} = \frac{63}{65} > 0$.
If we take $\cos B = -\frac{12}{13}$ and $\sin A = \frac{3}{5}$, $\cos(A+B) = \frac{-48 - 15}{65} = \frac{-63}{65} < 0$.
If we take $\cos B = -\frac{12}{13}$ and $\sin A = -\frac{3}{5}$, $\cos(A+B) = \frac{-48 + 15}{65} = \frac{-33}{65} < 0$.
So, we must have $\cos B = \frac{12}{13}$.
Now consider $\sin(A+B) = \sin A \cos B + \cos A \sin B = (\pm \frac{3}{5}) \left(\frac{12}{13}\right) + \left(\frac{4}{5}\right) \left(\frac{5}{13}\right) = \frac{\pm 36 + 20}{65}$.
For $\sin(A+B)$ to be positive, the numerator $\pm 36 + 20$ must be positive.
If we take $\sin A = \frac{3}{5}$, $\sin(A+B) = \frac{36 + 20}{65} = \frac{56}{65} > 0$.
If we take $\sin A = -\frac{3}{5}$, $\sin(A+B) = \frac{-36 + 20}{65} = \frac{-16}{65} < 0$.
So, we must have $\sin A = \frac{3}{5}$.
Thus, the only combination of signs consistent with $0 < 2\alpha < \frac{\pi}{2}$ is $\sin A = \frac{3}{5}$ and $\cos B = \frac{12}{13}$.
This means $\tan A = \frac{\sin A}{\cos A} = \frac{3/5}{4/5} = \frac{3}{4}$.
And $\tan B = \frac{\sin B}{\cos B} = \frac{5/13}{12/13} = \frac{5}{12}$.
Now substitute these values into the formula for $\tan 2\alpha$:
$\tan 2\alpha = \frac{\tan A + \tan B}{1 - \tan A \tan B} = \frac{\frac{3}{4} + \frac{5}{12}}{1 - \left(\frac{3}{4}\right) \left(\frac{5}{12}\right)}$
$\tan 2\alpha = \frac{\frac{9}{12} + \frac{5}{12}}{1 - \frac{15}{48}}$
$\tan 2\alpha = \frac{\frac{14}{12}}{\frac{48 - 15}{48}} = \frac{\frac{14}{12}}{\frac{33}{48}}$
$\tan 2\alpha = \frac{14}{12} \times \frac{48}{33} = \frac{14}{1} \times \frac{4}{33}$
$\tan 2\alpha = \frac{56}{33}$
The value of $\tan 2\alpha$ is $\mathbf{\frac{56}{33}}$.
Question 5. If $tan \;x = \frac{b}{a}$, then find the value of $\sqrt{\frac{a \;+\; b}{a \;-\; b}}$ + $\sqrt{\frac{a \;-\; b}{a \;+\; b}}$
Answer:
Solution:
Let the given expression be $E$.
$E = \sqrt{\frac{a + b}{a - b}} + \sqrt{\frac{a - b}{a + b}}$
To combine the two terms, find a common denominator:
The common denominator is $\sqrt{(a - b)(a + b)}$.
$E = \frac{\left(\sqrt{\frac{a + b}{a - b}}\right) \sqrt{(a - b)(a + b)} + \left(\sqrt{\frac{a - b}{a + b}}\right) \sqrt{(a - b)(a + b)}}{\sqrt{(a - b)(a + b)}}$
$E = \frac{\sqrt{(a + b)(a + b)} + \sqrt{(a - b)(a - b)}}{\sqrt{(a - b)(a + b)}}$
For the square roots to be real, we need $(a+b)(a-b) \ge 0$, which means $a^2 - b^2 \ge 0$, or $|a| \ge |b|$. Also, the denominators $a-b$ and $a+b$ cannot be zero, so $a \ne b$ and $a \ne -b$. Thus $a^2 \ne b^2$. So we require $a^2 - b^2 > 0$, i.e., $|a| > |b|$.
Assuming $a^2 > b^2$:
$E = \frac{\sqrt{(a + b)^2} + \sqrt{(a - b)^2}}{\sqrt{a^2 - b^2}}$
$E = \frac{|a + b| + |a - b|}{\sqrt{a^2 - b^2}}$
This expression depends on the signs of $a+b$ and $a-b$. However, for the original square roots to be real and defined, $\frac{a+b}{a-b}$ must be positive. This happens when $a+b$ and $a-b$ have the same sign.
Case 1: $a+b > 0$ and $a-b > 0$. This implies $a > b$. Since $a-b > 0$, $a+b$ will also be positive if $b \ge 0$. If $b<0$, we need $a > -b$. The condition $a>b$ and $a>-b$ is equivalent to $a > |b|$. In this case, $|a+b| = a+b$ and $|a-b| = a-b$.
$E = \frac{(a + b) + (a - b)}{\sqrt{a^2 - b^2}} = \frac{2a}{\sqrt{a^2 - b^2}}$
Case 2: $a+b < 0$ and $a-b < 0$. This implies $a < b$. Since $a-b < 0$, $a+b$ will also be negative if $b \le 0$. If $b>0$, we need $a < -b$. The condition $a
$E = \frac{-(a + b) + -(a - b)}{\sqrt{a^2 - b^2}} = \frac{-2a}{\sqrt{a^2 - b^2}}$
So, $E = \frac{2a \cdot \text{sgn}(a)}{\sqrt{a^2 - b^2}}$, or alternatively, we can keep the simpler form $\frac{2a}{\sqrt{a^2-b^2}}$ and remember the context. Let's proceed with $\frac{2a}{\sqrt{a^2-b^2}}$ assuming the signs work out.
We are given that $\tan x = \frac{b}{a}$. This means $b = a \tan x$.
Substitute $b = a \tan x$ into the expression for $E$:
$E = \frac{2a}{\sqrt{a^2 - (a \tan x)^2}} = \frac{2a}{\sqrt{a^2 - a^2 \tan^2 x}}$
$E = \frac{2a}{\sqrt{a^2 (1 - \tan^2 x)}} = \frac{2a}{\sqrt{a^2} \sqrt{1 - \tan^2 x}}$
$E = \frac{2a}{|a| \sqrt{1 - \tan^2 x}}$
For the expression $\sqrt{1 - \tan^2 x}$ to be real, we must have $1 - \tan^2 x \ge 0$, which means $\tan^2 x \le 1$, or $|\tan x| \le 1$. Since the original expression requires $a^2 \ne b^2$, we must have $\tan^2 x \ne 1$, so $|\tan x| < 1$. This ensures $1 - \tan^2 x > 0$.
We have two cases depending on the sign of $a$:
If $a > 0$, then $|a| = a$.
$E = \frac{2a}{a \sqrt{1 - \tan^2 x}} = \frac{2}{\sqrt{1 - \tan^2 x}}$
If $a < 0$, then $|a| = -a$.
$E = \frac{2a}{-a \sqrt{1 - \tan^2 x}} = \frac{-2}{\sqrt{1 - \tan^2 x}}$
Using the identity $1 - \tan^2 x = 1 - \frac{\sin^2 x}{\cos^2 x} = \frac{\cos^2 x - \sin^2 x}{\cos^2 x} = \frac{\cos 2x}{\cos^2 x}$:
$\sqrt{1 - \tan^2 x} = \sqrt{\frac{\cos 2x}{\cos^2 x}} = \frac{\sqrt{\cos 2x}}{\sqrt{\cos^2 x}} = \frac{\sqrt{\cos 2x}}{|\cos x|}$
For this to be real, $\cos 2x \ge 0$. Combined with $|\tan x| < 1$, which implies $\cos 2x > 0$.
Substituting this back into the expressions for $E$:
If $a > 0$, $E = \frac{2}{\frac{\sqrt{\cos 2x}}{|\cos x|}} = \frac{2 |\cos x|}{\sqrt{\cos 2x}}$.
If $a < 0$, $E = \frac{-2}{\frac{\sqrt{\cos 2x}}{|\cos x|}} = \frac{-2 |\cos x|}{\sqrt{\cos 2x}}$.
The value depends on the sign of $a$ and the sign of $\cos x$. However, the condition $|\tan x| < 1$ means $x$ lies in intervals like $(-\pi/4, \pi/4), (3\pi/4, 5\pi/4)$, etc.
If $x \in (-\pi/4, \pi/4) + 2n\pi$, then $\cos x > 0$, $|\cos x| = \cos x$.
If $x \in (3\pi/4, 5\pi/4) + 2n\pi$, then $\cos x < 0$, $|\cos x| = -\cos x$.
Also, the sign of $a$ determines the quadrant of $x$ for a given sign of $b$. For example, if $b>0$, $a>0$ implies $x$ in Q1, $a<0$ implies $x$ in Q2.
Let's assume the simplest case where $a > 0$ and $x$ is in $(-\pi/4, \pi/4)$.
Then $E = \frac{2 \cos x}{\sqrt{\cos 2x}}$.
Based on standard problem patterns and expected simplifications, the most common form related to this structure is $\frac{2}{\sqrt{1-\tan^2 x}}$. While it doesn't simplify to a single sec/csc term generally, it is the direct result of substituting $\tan x = b/a$ into the simplified algebraic expression, assuming $a>0$.
The value of the expression is $\mathbf{\frac{2a}{\sqrt{a^2-b^2}}}$ or $\mathbf{\frac{2}{\sqrt{1-\tan^2 x}}}$ (assuming $a>0$ and $|\tan x|<1$).
Question 6. Prove that cos θ cos $\frac{\theta}{2}$ - cos 3θ cos $\frac{9\theta}{2}$ = sin 7θ sin 8θ .
[Hint: Express L.H.S. = $\frac{1}{2}$ (2 cos θ cos $\frac{\theta}{2}$ - 2 cos 3θ cos $\frac{9\theta}{2}$) ]
Answer:
Solution:
To Prove: $\cos \theta \cos \frac{\theta}{2} - \cos 3\theta \cos \frac{9\theta}{2} = \sin 7\theta \sin 8\theta$
Consider the Left Hand Side (LHS):
LHS = $\cos \theta \cos \frac{\theta}{2} - \cos 3\theta \cos \frac{9\theta}{2}$
Following the hint, multiply and divide by 2:
LHS = $\frac{1}{2} \left[ 2 \cos \theta \cos \frac{\theta}{2} - 2 \cos 3\theta \cos \frac{9\theta}{2} \right]$
Use the product-to-sum formula: $2 \cos A \cos B = \cos(A+B) + \cos(A-B)$.
For the first term, $A = \theta$, $B = \frac{\theta}{2}$:
$2 \cos \theta \cos \frac{\theta}{2} = \cos\left(\theta + \frac{\theta}{2}\right) + \cos\left(\theta - \frac{\theta}{2}\right) = \cos \frac{3\theta}{2} + \cos \frac{\theta}{2}$
For the second term, $A = 3\theta$, $B = \frac{9\theta}{2}$:
$2 \cos 3\theta \cos \frac{9\theta}{2} = \cos\left(3\theta + \frac{9\theta}{2}\right) + \cos\left(3\theta - \frac{9\theta}{2}\right) = \cos \frac{15\theta}{2} + \cos \left(-\frac{3\theta}{2}\right) = \cos \frac{15\theta}{2} + \cos \frac{3\theta}{2}$
Substitute these results back into the LHS expression:
LHS = $\frac{1}{2} \left[ \left( \cos \frac{3\theta}{2} + \cos \frac{\theta}{2} \right) - \left( \cos \frac{15\theta}{2} + \cos \frac{3\theta}{2} \right) \right]$
LHS = $\frac{1}{2} \left[ \cos \frac{3\theta}{2} + \cos \frac{\theta}{2} - \cos \frac{15\theta}{2} - \cos \frac{3\theta}{2} \right]$
Cancel out the $\cos \frac{3\theta}{2}$ terms:
LHS = $\frac{1}{2} \left[ \cos \frac{\theta}{2} - \cos \frac{15\theta}{2} \right]$
Now consider the Right Hand Side (RHS):
RHS = $\sin 7\theta \sin 8\theta$
Use the product-to-sum formula: $\sin A \sin B = \frac{1}{2} [\cos(A-B) - \cos(A+B)]$.
Let $A = 7\theta$, $B = 8\theta$:
RHS = $\frac{1}{2} [\cos(7\theta - 8\theta) - \cos(7\theta + 8\theta)]$
RHS = $\frac{1}{2} [\cos(-\theta) - \cos(15\theta)]$
Using the identity $\cos(-x) = \cos x$:
RHS = $\frac{1}{2} [\cos \theta - \cos 15\theta]$
Comparing the simplified LHS and RHS:
LHS = $\frac{1}{2} \left[ \cos \frac{\theta}{2} - \cos \frac{15\theta}{2} \right]$
RHS = $\frac{1}{2} [\cos \theta - \cos 15\theta]$
In general, $\cos \frac{\theta}{2} - \cos \frac{15\theta}{2} \neq \cos \theta - \cos 15\theta$.
For example, if $\theta = \pi$: LHS $= \frac{1}{2} [\cos \frac{\pi}{2} - \cos \frac{15\pi}{2}] = \frac{1}{2} [0 - 0] = 0$. RHS $= \frac{1}{2} [\cos \pi - \cos 15\pi] = \frac{1}{2} [-1 - (-1)] = \frac{1}{2} [-1 + 1] = 0$.
If $\theta = \frac{\pi}{3}$: LHS $= \frac{1}{2} [\cos \frac{\pi}{6} - \cos \frac{15\pi}{6}] = \frac{1}{2} [\cos \frac{\pi}{6} - \cos \frac{5\pi}{2}] = \frac{1}{2} [\frac{\sqrt{3}}{2} - 0] = \frac{\sqrt{3}}{4}$. RHS $= \frac{1}{2} [\cos \frac{\pi}{3} - \cos 15\frac{\pi}{3}] = \frac{1}{2} [\cos \frac{\pi}{3} - \cos 5\pi] = \frac{1}{2} [\frac{1}{2} - (-1)] = \frac{1}{2} [\frac{3}{2}] = \frac{3}{4}$. $\frac{\sqrt{3}}{4} \neq \frac{3}{4}$.
Since the simplified expressions for LHS and RHS are not equal for all values of $\theta$, the given identity appears to be incorrect as stated.
The steps carried out based on the hint lead to $\text{LHS} = \frac{1}{2} (\cos \frac{\theta}{2} - \cos \frac{15\theta}{2})$.
The steps to simplify the RHS lead to $\text{RHS} = \frac{1}{2} (\cos \theta - \cos 15\theta)$.
These expressions are not identical. There seems to be a typographical error in the problem statement.
Question 7. If a cos θ + b sin θ = m and a sin θ – b cos θ = n, then show that a2 + b2 = m2 + n2
Answer:
Proof:
Given:
a cos θ + b sin θ = m
... (i)
a sin θ – b cos θ = n
... (ii)
To Show: $a^2 + b^2 = m^2 + n^2$
Square both sides of equation (i):
$(a \cos \theta + b \sin \theta)^2 = m^2$
$a^2 \cos^2 \theta + b^2 \sin^2 \theta + 2ab \cos \theta \sin \theta = m^2$
$a^2 \cos^2 \theta + b^2 \sin^2 \theta + 2ab \sin \theta \cos \theta = m^2$
... (iii)
Square both sides of equation (ii):
$(a \sin \theta - b \cos \theta)^2 = n^2$
$a^2 \sin^2 \theta + b^2 \cos^2 \theta - 2ab \sin \theta \cos \theta = n^2$
$a^2 \sin^2 \theta + b^2 \cos^2 \theta - 2ab \sin \theta \cos \theta = n^2$
... (iv)
Add equation (iii) and equation (iv):
$(a^2 \cos^2 \theta + b^2 \sin^2 \theta + 2ab \sin \theta \cos \theta) + (a^2 \sin^2 \theta + b^2 \cos^2 \theta - 2ab \sin \theta \cos \theta) = m^2 + n^2$
Group the terms:
$a^2 \cos^2 \theta + a^2 \sin^2 \theta + b^2 \sin^2 \theta + b^2 \cos^2 \theta + 2ab \sin \theta \cos \theta - 2ab \sin \theta \cos \theta = m^2 + n^2$
Factor out $a^2$ and $b^2$, and notice that the terms with $2ab$ cancel out:
$a^2 (\cos^2 \theta + \sin^2 \theta) + b^2 (\sin^2 \theta + \cos^2 \theta) + 0 = m^2 + n^2$
Using the fundamental trigonometric identity $\sin^2 \theta + \cos^2 \theta = 1$:
$a^2 (1) + b^2 (1) = m^2 + n^2$
$a^2 + b^2 = m^2 + n^2$
Thus, we have shown that $a^2 + b^2 = m^2 + n^2$.
Hence proved.
Question 8. Find the value of tan 22°30’.
[Hint: Let θ = 45°, use $\tan\frac{\theta}{2} = \frac{\sin\frac{\theta}{2}}{\cos\frac{\theta}{2}} = \frac{2\sin\frac{\theta}{2}\cos\frac{\theta}{2}}{2\cos^2\frac{\theta}{2}} = \frac{\sin\theta}{1 + \cos\theta}$]
Answer:
Solution:
We need to find the value of $\tan 22^\circ 30'$.
Let $\theta = 45^\circ$. Then $\frac{\theta}{2} = \frac{45^\circ}{2} = 22^\circ 30'$.
We use the half-angle tangent formula given in the hint:
$\tan \frac{\theta}{2} = \frac{\sin \theta}{1 + \cos \theta}$
Substitute $\theta = 45^\circ$ into the formula:
$\tan 22^\circ 30' = \tan \frac{45^\circ}{2} = \frac{\sin 45^\circ}{1 + \cos 45^\circ}$
We know that $\sin 45^\circ = \frac{1}{\sqrt{2}}$ and $\cos 45^\circ = \frac{1}{\sqrt{2}}$.
Substitute these values:
$\tan 22^\circ 30' = \frac{\frac{1}{\sqrt{2}}}{1 + \frac{1}{\sqrt{2}}}$
Simplify the expression:
$\tan 22^\circ 30' = \frac{\frac{1}{\sqrt{2}}}{\frac{\sqrt{2} + 1}{\sqrt{2}}}$
$\tan 22^\circ 30' = \frac{1}{\sqrt{2}} \times \frac{\sqrt{2}}{\sqrt{2} + 1}$
$\tan 22^\circ 30' = \frac{1}{\sqrt{2} + 1}$
Rationalize the denominator by multiplying the numerator and denominator by the conjugate of the denominator, which is $\sqrt{2} - 1$:
$\tan 22^\circ 30' = \frac{1}{(\sqrt{2} + 1)} \times \frac{(\sqrt{2} - 1)}{(\sqrt{2} - 1)}$
$\tan 22^\circ 30' = \frac{\sqrt{2} - 1}{(\sqrt{2})^2 - 1^2}$
$\tan 22^\circ 30' = \frac{\sqrt{2} - 1}{2 - 1}$
$\tan 22^\circ 30' = \sqrt{2} - 1$
The value of $\tan 22^\circ 30'$ is $\mathbf{\sqrt{2} - 1}$.
Question 9. Prove that sin 4A = 4sin A cos3 A – 4 cos A sin3 A.
Answer:
Proof:
To Prove: $\sin 4A = 4 \sin A \cos^3 A - 4 \cos A \sin^3 A$
Consider the Right Hand Side (RHS):
RHS = $4 \sin A \cos^3 A - 4 \cos A \sin^3 A$
Factor out the common term $4 \sin A \cos A$:
RHS = $4 \sin A \cos A (\cos^2 A - \sin^2 A)$
We recognize the double angle identities:
$\sin 2A = 2 \sin A \cos A$
$\cos 2A = \cos^2 A - \sin^2 A$
Substitute these identities into the RHS expression:
RHS = $2 (2 \sin A \cos A) (\cos^2 A - \sin^2 A)$
RHS = $2 (\sin 2A) (\cos 2A)$
This expression is in the form $2 \sin x \cos x$, where $x = 2A$.
Using the double angle identity $\sin 2x = 2 \sin x \cos x$:
RHS = $\sin (2 \times 2A)$
RHS = $\sin 4A$
This is equal to the Left Hand Side (LHS).
RHS = LHS
Therefore, the identity $\sin 4A = 4 \sin A \cos^3 A - 4 \cos A \sin^3 A$ is proved.
Question 10. If tan θ + sin θ = m and tan θ – sin θ = n, then prove that m2 – n2 = 4sin θ tan θ
[Hint: m + n = 2tan θ, m – n = 2 sin θ, then use m2 – n2 = (m + n) (m – n)]
Answer:
Proof:
Given:
$m = \tan \theta + \sin \theta$
$n = \tan \theta - \sin \theta$
To Prove: $m^2 - n^2 = 4 \sin \theta \tan \theta$
Consider the sum of $m$ and $n$:
$m + n = (\tan \theta + \sin \theta) + (\tan \theta - \sin \theta)$
$m + n = \tan \theta + \sin \theta + \tan \theta - \sin \theta$
$m + n = 2 \tan \theta$
... (i)
Consider the difference of $m$ and $n$:
$m - n = (\tan \theta + \sin \theta) - (\tan \theta - \sin \theta)$
$m - n = \tan \theta + \sin \theta - \tan \theta + \sin \theta$
$m - n = 2 \sin \theta$
... (ii)
Consider the expression $m^2 - n^2$. This is a difference of squares, which can be factored as $(m+n)(m-n)$.
$m^2 - n^2 = (m+n)(m-n)$
Substitute the results from equations (i) and (ii) into the factored expression:
$m^2 - n^2 = (2 \tan \theta) (2 \sin \theta)$
$m^2 - n^2 = 4 \tan \theta \sin \theta$
This is equal to the expression we needed to prove.
$m^2 - n^2 = 4 \sin \theta \tan \theta$
Hence proved.
Question 11. If tan (A + B) = p, tan (A – B) = q, then show that tan 2 A = $\frac{p \;+\; q}{1 \;-\; pq}$
[Hint: Use 2A = (A + B) + (A – B)]
Answer:
Solution:
Given:
$\tan (A + B) = p$
$\tan (A - B) = q$
To Show: $\tan 2A = \frac{p + q}{1 - pq}$
Consider the angle $2A$. We can express it as the sum of $(A+B)$ and $(A-B)$:
$2A = (A + B) + (A - B)$
Take the tangent of both sides:
$\tan (2A) = \tan [(A + B) + (A - B)]$
Use the tangent addition formula: $\tan (x + y) = \frac{\tan x + \tan y}{1 - \tan x \tan y}$.
Let $x = A + B$ and $y = A - B$.
$\tan (2A) = \frac{\tan (A + B) + \tan (A - B)}{1 - \tan (A + B) \tan (A - B)}$
Substitute the given values $\tan (A + B) = p$ and $\tan (A - B) = q$:
$\tan (2A) = \frac{p + q}{1 - pq}$
This is the required result.
Hence proved.
Question 12. If $cos \ α + cos \ β = 0 = sin \ α + sin \ β$, then prove that $cos \ 2α + cos \ 2β $$= – 2cos \ (α + β)$.
[Hint: (cos $α$ + cos $β$)2 – (sin $α$ + sin $β$)2 = 0]
Answer:
Proof:
Given:
$\cos \alpha + \cos \beta = 0$
... (i)
$\sin \alpha + \sin \beta = 0$
... (ii)
To Prove: $\cos 2\alpha + \cos 2\beta = -2 \cos (\alpha + \beta)$
From the given conditions, we have:
$(\cos \alpha + \cos \beta)^2 = 0^2 = 0$
$(\sin \alpha + \sin \beta)^2 = 0^2 = 0$
Subtracting the second squared equation from the first:
$(\cos \alpha + \cos \beta)^2 - (\sin \alpha + \sin \beta)^2 = 0 - 0 = 0$
Expand the squared terms:
$(\cos^2 \alpha + \cos^2 \beta + 2 \cos \alpha \cos \beta) - (\sin^2 \alpha + \sin^2 \beta + 2 \sin \alpha \sin \beta) = 0$
Remove the parentheses and rearrange the terms:
$\cos^2 \alpha + \cos^2 \beta + 2 \cos \alpha \cos \beta - \sin^2 \alpha - \sin^2 \beta - 2 \sin \alpha \sin \beta = 0$
$(\cos^2 \alpha - \sin^2 \alpha) + (\cos^2 \beta - \sin^2 \beta) + 2 (\cos \alpha \cos \beta - \sin \alpha \sin \beta) = 0$
Use the double angle identity $\cos 2x = \cos^2 x - \sin^2 x$:
$\cos 2\alpha + \cos 2\beta + 2 (\cos \alpha \cos \beta - \sin \alpha \sin \beta) = 0$
Use the angle addition identity $\cos (A + B) = \cos A \cos B - \sin A \sin B$:
$\cos 2\alpha + \cos 2\beta + 2 \cos (\alpha + \beta) = 0$
Rearrange the equation to isolate the sum of double angles:
$\cos 2\alpha + \cos 2\beta = -2 \cos (\alpha + \beta)$
This is the required identity.
Hence proved.
Question 13. If $\frac{sin (x \;+\; y)}{sin (x \;-\; y)}$ = $\frac{a \;+\; b}{a \;-\; b}$, then show that $\frac{tan\; x}{tan \;y}$ = $\frac{a}{b}$
[Hint: Use Componendo and Dividendo].
Answer:
Proof:
Given: $\frac{\sin (x + y)}{\sin (x - y)} = \frac{a + b}{a - b}$
To Show: $\frac{\tan x}{\tan y} = \frac{a}{b}$
Apply Componendo and Dividendo to the given equation:
$\frac{\sin (x + y) + \sin (x - y)}{\sin (x + y) - \sin (x - y)} = \frac{(a + b) + (a - b)}{(a + b) - (a - b)}$
Use the sum-to-product formulas for the left-hand side:
$\sin A + \sin B = 2 \sin \left( \frac{A+B}{2} \right) \cos \left( \frac{A-B}{2} \right)$
$\sin A - \sin B = 2 \cos \left( \frac{A+B}{2} \right) \sin \left( \frac{A-B}{2} \right)$
Let $A = x + y$ and $B = x - y$.
$A + B = (x + y) + (x - y) = 2x$
$A - B = (x + y) - (x - y) = 2y$
Substitute these into the sum-to-product formulas:
Numerator of LHS: $\sin (x + y) + \sin (x - y) = 2 \sin \left( \frac{2x}{2} \right) \cos \left( \frac{2y}{2} \right) = 2 \sin x \cos y$
Denominator of LHS: $\sin (x + y) - \sin (x - y) = 2 \cos \left( \frac{2x}{2} \right) \sin \left( \frac{2y}{2} \right) = 2 \cos x \sin y$
Substitute these back into the equation from Componendo and Dividendo:
$\frac{2 \sin x \cos y}{2 \cos x \sin y} = \frac{(a + b) + (a - b)}{(a + b) - (a - b)}$
Simplify both sides:
$\frac{\sin x \cos y}{\cos x \sin y} = \frac{a + b + a - b}{a + b - a + b}$
$\frac{\sin x \cos y}{\cos x \sin y} = \frac{2a}{2b}$
$\frac{\sin x \cos y}{\cos x \sin y} = \frac{a}{b}$
Rewrite the left-hand side as a product of tangent and cotangent:
$\left( \frac{\sin x}{\cos x} \right) \left( \frac{\cos y}{\sin y} \right) = \frac{a}{b}$
$\tan x \cot y = \frac{a}{b}$
Since $\cot y = \frac{1}{\tan y}$ (assuming $\tan y \neq 0$):
$\tan x \cdot \frac{1}{\tan y} = \frac{a}{b}$
$\frac{\tan x}{\tan y} = \frac{a}{b}$
This is the required result. Note that this derivation assumes $\sin(x-y) \neq 0$, $\cos x \neq 0$, $\sin y \neq 0$, and $b \neq 0$ for the steps to be valid. The condition $\frac{\tan x}{\tan y} = \frac{a}{b}$ also implies that if $\tan y = 0$, then $b=0$, and if $\tan x = 0$, then $a=0$.
Hence proved.
Question 14. If $tan \;θ = \frac{sin \;\alpha\;-\;cos \;\alpha}{sin \;\alpha\;+\;cos \;\alpha}$, then show that sin $α$ + cos $α$ = $\sqrt{2}$ cos θ.
[Hint: Express $tan \;θ = tan \left( \alpha-\frac{\pi}{4} \right)\;\;\;,\; θ = α - \frac{\pi}{4}$ ]
Answer:
Proof:
Given: $\tan \theta = \frac{\sin \alpha - \cos \alpha}{\sin \alpha + \cos \alpha}$
To Show: $\sin \alpha + \cos \alpha = \sqrt{2} \cos \theta$
Consider the given expression for $\tan \theta$:
$\tan \theta = \frac{\sin \alpha - \cos \alpha}{\sin \alpha + \cos \alpha}$
Divide the numerator and the denominator of the Right Hand Side by $\cos \alpha$ (assuming $\cos \alpha \neq 0$):
$\tan \theta = \frac{\frac{\sin \alpha}{\cos \alpha} - \frac{\cos \alpha}{\cos \alpha}}{\frac{\sin \alpha}{\cos \alpha} + \frac{\cos \alpha}{\cos \alpha}}$
$\tan \theta = \frac{\tan \alpha - 1}{\tan \alpha + 1}$
We know that $\tan \frac{\pi}{4} = 1$. Substitute this value:
$\tan \theta = \frac{\tan \alpha - \tan \frac{\pi}{4}}{1 + \tan \alpha \cdot 1} = \frac{\tan \alpha - \tan \frac{\pi}{4}}{1 + \tan \alpha \tan \frac{\pi}{4}}$
Using the tangent subtraction formula, $\tan (A - B) = \frac{\tan A - \tan B}{1 + \tan A \tan B}$:
$\tan \theta = \tan \left( \alpha - \frac{\pi}{4} \right)$
From this equation, the general solution is $\theta = \alpha - \frac{\pi}{4} + n\pi$ for some integer $n$. Following the hint which implies a specific relationship (likely for the principal value or a chosen branch):
$\theta = \alpha - \frac{\pi}{4}$
(From hint) ... (i)
This can be rewritten as:
$\alpha - \frac{\pi}{4} = \theta$
Now consider the expression $\sin \alpha + \cos \alpha$.
We can rewrite this expression by factoring out $\sqrt{1^2 + 1^2} = \sqrt{2}$:
$\sin \alpha + \cos \alpha = \sqrt{2} \left( \frac{1}{\sqrt{2}} \sin \alpha + \frac{1}{\sqrt{2}} \cos \alpha \right)$
Recognize that $\frac{1}{\sqrt{2}} = \cos \frac{\pi}{4} = \sin \frac{\pi}{4}$. Substitute these values:
$\sin \alpha + \cos \alpha = \sqrt{2} \left( \cos \frac{\pi}{4} \sin \alpha + \sin \frac{\pi}{4} \cos \alpha \right)$
Using the identity $\cos A \cos B + \sin A \sin B = \cos (A - B)$ with $A = \alpha$ and $B = \frac{\pi}{4}$ (or $A = \frac{\pi}{4}$ and $B = \alpha$):
$\sin \alpha + \cos \alpha = \sqrt{2} \left( \cos \alpha \cos \frac{\pi}{4} + \sin \alpha \sin \frac{\pi}{4} \right)$
$\sin \alpha + \cos \alpha = \sqrt{2} \cos \left( \alpha - \frac{\pi}{4} \right)$
From equation (i), we have $\alpha - \frac{\pi}{4} = \theta$.
Substitute this into the expression:
$\sin \alpha + \cos \alpha = \sqrt{2} \cos (\theta)$
This is the required result.
Hence proved.
Question 15. If sin θ + cos θ = 1, then find the general value of θ.
Answer:
Solution:
The given equation is $\sin \theta + \cos \theta = 1$.
This is an equation of the form $a \sin \theta + b \cos \theta = c$, where $a = 1$, $b = 1$, and $c = 1$.
We can solve this by converting the left-hand side into a single trigonometric function.
Multiply and divide the LHS by $\sqrt{a^2 + b^2} = \sqrt{1^2 + 1^2} = \sqrt{2}$.
$\sqrt{2} \left( \frac{1}{\sqrt{2}} \sin \theta + \frac{1}{\sqrt{2}} \cos \theta \right) = 1$
Recognize that $\frac{1}{\sqrt{2}} = \cos \frac{\pi}{4} = \sin \frac{\pi}{4}$.
$\sqrt{2} \left( \cos \frac{\pi}{4} \sin \theta + \sin \frac{\pi}{4} \cos \theta \right) = 1$
Use the identity $\sin A \cos B + \cos A \sin B = \sin (A + B)$:
$\sqrt{2} \sin \left( \theta + \frac{\pi}{4} \right) = 1$
Divide by $\sqrt{2}$:
$\sin \left( \theta + \frac{\pi}{4} \right) = \frac{1}{\sqrt{2}}$
We know that $\sin \frac{\pi}{4} = \frac{1}{\sqrt{2}}$.
The general solution for $\sin x = \sin y$ is $x = n\pi + (-1)^n y$, where $n$ is an integer.
Here, $x = \theta + \frac{\pi}{4}$ and $y = \frac{\pi}{4}$.
So, $\theta + \frac{\pi}{4} = n\pi + (-1)^n \frac{\pi}{4}$, where $n \in \mathbb{Z}$.
Isolate $\theta$:
$\theta = n\pi + (-1)^n \frac{\pi}{4} - \frac{\pi}{4}$
We can consider two cases based on the value of $n$ (even or odd).
Case 1: $n$ is an even integer. Let $n = 2k$ for some integer $k$.
$(-1)^{2k} = 1$.
$\theta = 2k\pi + (1) \frac{\pi}{4} - \frac{\pi}{4}$
$\theta = 2k\pi + \frac{\pi}{4} - \frac{\pi}{4}$
$\theta = 2k\pi$
Case 2: $n$ is an odd integer. Let $n = 2k + 1$ for some integer $k$.
$(-1)^{2k+1} = -1$.
$\theta = (2k + 1)\pi + (-1) \frac{\pi}{4} - \frac{\pi}{4}$
$\theta = 2k\pi + \pi - \frac{\pi}{4} - \frac{\pi}{4}$
$\theta = 2k\pi + \pi - \frac{2\pi}{4}$
$\theta = 2k\pi + \pi - \frac{\pi}{2}$
$\theta = 2k\pi + \frac{\pi}{2}$
The general solution is $\theta = 2k\pi$ or $\theta = 2k\pi + \frac{\pi}{2}$, where $k$ is any integer.
This can be written as:
$\theta = 2k\pi, \quad k \in \mathbb{Z}$
$\theta = 2k\pi + \frac{\pi}{2}, \quad k \in \mathbb{Z}$
The general value of $\theta$ is $\mathbf{2k\pi}$ or $\mathbf{2k\pi + \frac{\pi}{2}}$, where $k \in \mathbb{Z}$.
Question 16. Find the most general value of θ satisfying the equation tan θ = –1 and cos θ = $\frac{1}{\sqrt{2}}$ .
Answer:
Solution:
We are asked to find the general value of $\theta$ that satisfies both equations:
1) $\tan \theta = -1$
2) $\cos \theta = \frac{1}{\sqrt{2}}$
Consider the first equation, $\tan \theta = -1$.
The principal value for which $\tan \theta = -1$ is $\theta = -\frac{\pi}{4}$.
The general solution for $\tan \theta = \tan \alpha$ is $\theta = n\pi + \alpha$, where $n \in \mathbb{Z}$.
So, the general solution for $\tan \theta = -1 = \tan(-\frac{\pi}{4})$ is:
$\theta = n\pi - \frac{\pi}{4}$, where $n \in \mathbb{Z}$
... (i)
The angles satisfying this equation lie in the second and fourth quadrants.
Consider the second equation, $\cos \theta = \frac{1}{\sqrt{2}}$.
The principal value for which $\cos \theta = \frac{1}{\sqrt{2}}$ is $\theta = \frac{\pi}{4}$.
The general solution for $\cos \theta = \cos \alpha$ is $\theta = 2m\pi \pm \alpha$, where $m \in \mathbb{Z}$.
So, the general solution for $\cos \theta = \frac{1}{\sqrt{2}} = \cos(\frac{\pi}{4})$ is:
$\theta = 2m\pi \pm \frac{\pi}{4}$, where $m \in \mathbb{Z}$
... (ii)
The angles satisfying this equation lie in the first and fourth quadrants.
We need to find the values of $\theta$ that satisfy both equations simultaneously. These values must lie in the common quadrant of the solutions, which is the fourth quadrant.
From equation (ii), the angles in the fourth quadrant are of the form $\theta = 2m\pi - \frac{\pi}{4}$.
We need to check if this form is included in the general solution from equation (i).
Equation (i) gives $\theta = n\pi - \frac{\pi}{4}$. For this to match the fourth-quadrant solution from (ii), we must have $n\pi = 2m\pi$, which implies $n = 2m$.
This means that $n$ must be an even integer.
So, the general solution satisfying both equations is obtained from equation (i) by taking $n$ as an even integer, say $n = 2k$ for some integer $k \in \mathbb{Z}$.
$\theta = (2k)\pi - \frac{\pi}{4}$
$\theta = 2k\pi - \frac{\pi}{4}$, where $k \in \mathbb{Z}$
We can verify that if $\theta = 2k\pi - \frac{\pi}{4}$:
$\tan \theta = \tan (2k\pi - \frac{\pi}{4}) = \tan (-\frac{\pi}{4}) = -1$. (Correct)
$\cos \theta = \cos (2k\pi - \frac{\pi}{4}) = \cos (-\frac{\pi}{4}) = \cos (\frac{\pi}{4}) = \frac{1}{\sqrt{2}}$. (Correct)
The most general value of $\theta$ satisfying both equations is $\mathbf{2k\pi - \frac{\pi}{4}}$, where $k$ is an integer.
Question 17. If cot θ + tan θ = 2 cosec θ, then find the general value of θ.
Answer:
Solution:
The given equation is $\cot \theta + \tan \theta = 2 \text{ cosec } \theta$.
First, express the terms in terms of $\sin \theta$ and $\cos \theta$:
$\cot \theta = \frac{\cos \theta}{\sin \theta}$, $\tan \theta = \frac{\sin \theta}{\cos \theta}$, and $\text{cosec } \theta = \frac{1}{\sin \theta}$.
Substitute these into the equation:
$\frac{\cos \theta}{\sin \theta} + \frac{\sin \theta}{\cos \theta} = \frac{2}{\sin \theta}$
Find a common denominator for the left-hand side:
$\frac{\cos^2 \theta + \sin^2 \theta}{\sin \theta \cos \theta} = \frac{2}{\sin \theta}$
Using the fundamental identity $\cos^2 \theta + \sin^2 \theta = 1$:
$\frac{1}{\sin \theta \cos \theta} = \frac{2}{\sin \theta}$
The original equation is defined only if $\sin \theta \neq 0$ and $\cos \theta \neq 0$. This means $\theta \neq n\pi$ and $\theta \neq n\pi + \frac{\pi}{2}$ for any integer $n$.
Since $\sin \theta \neq 0$, we can multiply both sides by $\sin \theta$:
$\frac{1}{\cos \theta} = 2$
This simplifies to:
$\cos \theta = \frac{1}{2}$
The principal value for which $\cos \theta = \frac{1}{2}$ is $\theta = \frac{\pi}{3}$.
The general solution for $\cos \theta = \cos \alpha$ is $\theta = 2n\pi \pm \alpha$, where $n \in \mathbb{Z}$.
So, the general solution for $\cos \theta = \cos \frac{\pi}{3}$ is:
$\theta = 2n\pi \pm \frac{\pi}{3}$, where $n \in \mathbb{Z}$.
We must check if these solutions violate the domain restrictions $\sin \theta \neq 0$ and $\cos \theta \neq 0$.
If $\cos \theta = \frac{1}{2}$, then $\sin^2 \theta = 1 - (\frac{1}{2})^2 = \frac{3}{4}$, so $\sin \theta = \pm \frac{\sqrt{3}}{2}$. Neither of these is zero, so $\sin \theta \neq 0$.
Also, $\cos \theta = \frac{1}{2} \neq 0$.
Therefore, the general solutions $\theta = 2n\pi \pm \frac{\pi}{3}$ are valid for the original equation.
The general value of $\theta$ is $\mathbf{2n\pi \pm \frac{\pi}{3}}$, where $n \in \mathbb{Z}$.
Question 18. If 2sin2 θ = 3cos θ, where 0 ≤ θ ≤ 2π, then find the value of θ.
Answer:
Solution:
The given equation is $2\sin^2 \theta = 3\cos \theta$.
The given range for $\theta$ is $0 \leq \theta \leq 2\pi$.
We need to express the equation in terms of a single trigonometric function.
Use the identity $\sin^2 \theta = 1 - \cos^2 \theta$.
Substitute this into the equation:
$2(1 - \cos^2 \theta) = 3\cos \theta$
$2 - 2\cos^2 \theta = 3\cos \theta$
Rearrange the terms to form a quadratic equation in $\cos \theta$:
$2\cos^2 \theta + 3\cos \theta - 2 = 0$
Let $x = \cos \theta$. The quadratic equation becomes:
$2x^2 + 3x - 2 = 0$
Solve this quadratic equation for $x$. We can factor it:
$(2x - 1)(x + 2) = 0$
This gives two possible values for $x$:
$2x - 1 = 0 \implies x = \frac{1}{2}$
$x + 2 = 0 \implies x = -2$
Substitute back $\cos \theta$ for $x$:
$\cos \theta = \frac{1}{2}$ or $\cos \theta = -2$
Since the range of the cosine function is $[-1, 1]$, the value $\cos \theta = -2$ is not possible for any real $\theta$.
So, we only consider the equation $\cos \theta = \frac{1}{2}$.
We need to find the values of $\theta$ in the range $0 \leq \theta \leq 2\pi$ for which $\cos \theta = \frac{1}{2}$.
The principal value for which $\cos \theta = \frac{1}{2}$ is $\theta = \frac{\pi}{3}$ (in the first quadrant).
Since $\cos \theta$ is positive, $\theta$ must be in the first or fourth quadrants.
The solution in the first quadrant is $\theta_1 = \frac{\pi}{3}$.
The solution in the fourth quadrant is $\theta_2 = 2\pi - \frac{\pi}{3} = \frac{6\pi - \pi}{3} = \frac{5\pi}{3}$.
Both of these values, $\frac{\pi}{3}$ and $\frac{5\pi}{3}$, lie within the given range $0 \leq \theta \leq 2\pi$.
The values of $\theta$ are $\mathbf{\frac{\pi}{3}}$ and $\mathbf{\frac{5\pi}{3}}$.
Question 19. If sec x cos 5x + 1 = 0, where 0 < x ≤ $\frac{\pi}{2}$ , then find the value of x.
Answer:
Solution:
The given equation is $\sec x \cos 5x + 1 = 0$.
The range for $x$ is $0 < x \leq \frac{\pi}{2}$.
Rewrite the equation in terms of cosine. Since $\sec x = \frac{1}{\cos x}$, the equation becomes:
$\frac{1}{\cos x} \cos 5x + 1 = 0$
Note that for $\sec x$ to be defined, $\cos x \neq 0$. In the given range $0 < x \leq \frac{\pi}{2}$, $\cos x > 0$ for $0 < x < \frac{\pi}{2}$ and $\cos x = 0$ at $x = \frac{\pi}{2}$. So $x = \frac{\pi}{2}$ must be excluded initially if we multiply by $\cos x$.
Multiply the equation by $\cos x$ (assuming $\cos x \neq 0$):
$\cos 5x + \cos x = 0$
Use the sum-to-product formula: $\cos A + \cos B = 2 \cos \left( \frac{A+B}{2} \right) \cos \left( \frac{A-B}{2} \right)$.
Let $A = 5x$ and $B = x$.
$\frac{A+B}{2} = \frac{5x+x}{2} = 3x$
$\frac{A-B}{2} = \frac{5x-x}{2} = 2x$
Substitute these into the equation:
$2 \cos (3x) \cos (2x) = 0$
This implies either $\cos (3x) = 0$ or $\cos (2x) = 0$.
Case 1: $\cos (3x) = 0$
The general solution for $\cos \theta = 0$ is $\theta = k\pi + \frac{\pi}{2}$, where $k \in \mathbb{Z}$.
$3x = k\pi + \frac{\pi}{2}$
$x = \frac{k\pi}{3} + \frac{\pi}{6}$, where $k \in \mathbb{Z}$.
We need to find the values of $x$ in the range $0 < x \leq \frac{\pi}{2}$.
For $k=0$, $x = \frac{0 \cdot \pi}{3} + \frac{\pi}{6} = \frac{\pi}{6}$.
$0 < \frac{\pi}{6} \leq \frac{\pi}{2}$. This is a valid solution. Check $\cos(\pi/6) = \frac{\sqrt{3}}{2} \neq 0$.
For $k=1$, $x = \frac{1 \cdot \pi}{3} + \frac{\pi}{6} = \frac{2\pi + \pi}{6} = \frac{3\pi}{6} = \frac{\pi}{2}$.
$0 < \frac{\pi}{2} \leq \frac{\pi}{2}$. This is at the boundary of the range. However, for the original equation to be defined, we must have $\cos x \neq 0$. Since $\cos (\frac{\pi}{2}) = 0$, $x = \frac{\pi}{2}$ is not a valid solution.
For other integer values of $k$, $x$ falls outside the range $0 < x \leq \frac{\pi}{2}$.
Case 2: $\cos (2x) = 0$
The general solution for $\cos \theta = 0$ is $\theta = m\pi + \frac{\pi}{2}$, where $m \in \mathbb{Z}$.
$2x = m\pi + \frac{\pi}{2}$
$x = \frac{m\pi}{2} + \frac{\pi}{4}$, where $m \in \mathbb{Z}$.
We need to find the values of $x$ in the range $0 < x \leq \frac{\pi}{2}$.
For $m=0$, $x = \frac{0 \cdot \pi}{2} + \frac{\pi}{4} = \frac{\pi}{4}$.
$0 < \frac{\pi}{4} \leq \frac{\pi}{2}$. This is a valid solution. Check $\cos(\pi/4) = \frac{1}{\sqrt{2}} \neq 0$.
For other integer values of $m$, $x$ falls outside the range $0 < x \leq \frac{\pi}{2}$.
Combining the valid solutions from both cases within the given range $0 < x \leq \frac{\pi}{2}$ and satisfying the condition $\cos x \neq 0$, the values of $x$ are $\frac{\pi}{6}$ and $\frac{\pi}{4}$.
The values of $x$ are $\mathbf{\frac{\pi}{6}}$ and $\mathbf{\frac{\pi}{4}}$.
Question 20 to 29 (Long Answer Type Questions)
Question 20. If sin (θ + $α$) = a and sin (θ + $β$) = b, then prove that cos 2($α$ – $β$) – 4ab cos ($α$ – $β$) = 1 – 2a2 – 2b2
[Hint: Express cos ($α$ – $β$) = cos ((θ + $α$) – (θ + $β$))]
Answer:
Given:
$\sin(\theta + \alpha) = a$
$\sin(\theta + \beta) = b$
To Prove:
$\cos 2(\alpha - \beta) - 4ab \cos (\alpha - \beta) = 1 - 2a^2 - 2b^2$
Proof:
Consider the term $\cos(\alpha - \beta)$. Using the hint provided and the angle subtraction formula for cosine, we have:
$\cos(\alpha - \beta) = \cos((\theta + \alpha) - (\theta + \beta))$
$\cos(\alpha - \beta) = \cos(\theta + \alpha)\cos(\theta + \beta) + \sin(\theta + \alpha)\sin(\theta + \beta)$
... (i)
Substitute the given values $\sin(\theta + \alpha) = a$ and $\sin(\theta + \beta) = b$ into equation (i):
$\cos(\alpha - \beta) = \cos(\theta + \alpha)\cos(\theta + \beta) + ab$
... (ii)
From equation (ii), we can express the product of cosines:
$\cos(\theta + \alpha)\cos(\theta + \beta) = \cos(\alpha - \beta) - ab$
... (iii)
Now, consider the term $\cos 2(\alpha - \beta)$. We can write this as $\cos(2(\theta + \alpha) - 2(\theta + \beta))$. Let $A = \theta + \alpha$ and $B = \theta + \beta$. Then $\cos 2(\alpha - \beta) = \cos(2A - 2B)$.
Using the angle subtraction formula for cosine:
$\cos(2A - 2B) = \cos(2A)\cos(2B) + \sin(2A)\sin(2B)$
... (iv)
We know the double angle identities $\cos 2x = 1 - 2\sin^2 x$ and $\sin 2x = 2\sin x \cos x$. Apply these to $A = \theta + \alpha$ and $B = \theta + \beta$:
$\cos(2A) = \cos(2(\theta + \alpha)) = 1 - 2\sin^2(\theta + \alpha) = 1 - 2a^2$
$\cos(2B) = \cos(2(\theta + \beta)) = 1 - 2\sin^2(\theta + \beta) = 1 - 2b^2$
$\sin(2A) = \sin(2(\theta + \alpha)) = 2\sin(\theta + \alpha)\cos(\theta + \alpha) = 2a \cos(\theta + \alpha)$
$\sin(2B) = \sin(2(\theta + \beta)) = 2\sin(\theta + \beta)\cos(\theta + \beta) = 2b \cos(\theta + \beta)$
Substitute these expressions into equation (iv):
$\cos 2(\alpha - \beta) = (1 - 2a^2)(1 - 2b^2) + (2a \cos(\theta + \alpha))(2b \cos(\theta + \beta))$
... (v)
Expand the first term and rearrange the second term in equation (v):
$\cos 2(\alpha - \beta) = (1 - 2b^2 - 2a^2 + 4a^2b^2) + 4ab \cos(\theta + \alpha)\cos(\theta + \beta)$
Now, substitute the expression for $\cos(\theta + \alpha)\cos(\theta + \beta)$ from equation (iii) into this equation:
$\cos 2(\alpha - \beta) = 1 - 2a^2 - 2b^2 + 4a^2b^2 + 4ab (\cos(\alpha - \beta) - ab)$
Expand the last term:
$\cos 2(\alpha - \beta) = 1 - 2a^2 - 2b^2 + 4a^2b^2 + 4ab \cos(\alpha - \beta) - 4a^2b^2$
Notice that the terms $4a^2b^2$ and $-4a^2b^2$ cancel out:
$\cos 2(\alpha - \beta) = 1 - 2a^2 - 2b^2 + 4ab \cos(\alpha - \beta)$
Rearrange the terms to match the desired identity:
$\cos 2(\alpha - \beta) - 4ab \cos (\alpha - \beta) = 1 - 2a^2 - 2b^2$
Thus, the identity is proven.
Question 21. If cos (θ + φ) = m cos (θ – φ), then prove that tan θ = $\frac{1 \;-\; m}{1 \;+\; m}$ cot φ .
[Hint: Express $\frac{\cos (\theta + \phi)}{\cos (\theta - \phi)} = \frac{m}{1}$ and apply Componendo and Dividendo]
Answer:
Given:
$\cos (\theta + \phi) = m \cos (\theta - \phi)$
To Prove:
$\tan \theta = \frac{1 - m}{1 + m} \cot \phi$
Proof:
From the given equation, we can rearrange it to form a ratio:
$\frac{\cos (\theta + \phi)}{\cos (\theta - \phi)} = \frac{m}{1}$
... (i)
Taking the reciprocal of equation (i):
$\frac{\cos (\theta - \phi)}{\cos (\theta + \phi)} = \frac{1}{m}$
... (ii)
Applying Componendo and Dividendo to equation (ii):
$\frac{\cos (\theta - \phi) + \cos (\theta + \phi)}{\cos (\theta - \phi) - \cos (\theta + \phi)} = \frac{1 + m}{1 - m}$
[Applying Componendo and Dividendo] ... (iii)
Now, we use the sum and difference formulas for cosine:
$\cos(A - B) = \cos A \cos B + \sin A \sin B$
$\cos(A + B) = \cos A \cos B - \sin A \sin B$
Consider the numerator of the Left Hand Side (LHS) of equation (iii):
$\cos (\theta - \phi) + \cos (\theta + \phi) = (\cos \theta \cos \phi + \sin \theta \sin \phi) + (\cos \theta \cos \phi - \sin \theta \sin \phi)$
$\cos (\theta - \phi) + \cos (\theta + \phi) = 2 \cos \theta \cos \phi$
Consider the denominator of the LHS of equation (iii):
$\cos (\theta - \phi) - \cos (\theta + \phi) = (\cos \theta \cos \phi + \sin \theta \sin \phi) - (\cos \theta \cos \phi - \sin \theta \sin \phi)$
$\cos (\theta - \phi) - \cos (\theta + \phi) = \cos \theta \cos \phi + \sin \theta \sin \phi - \cos \theta \cos \phi + \sin \theta \sin \phi$
$\cos (\theta - \phi) - \cos (\theta + \phi) = 2 \sin \theta \sin \phi$
Substitute these back into equation (iii):
$\frac{2 \cos \theta \cos \phi}{2 \sin \theta \sin \phi} = \frac{1 + m}{1 - m}$
Simplify the LHS:
$\frac{\cancel{2} \cos \theta \cos \phi}{\cancel{2} \sin \theta \sin \phi} = \frac{1 + m}{1 - m}$
$\left(\frac{\cos \theta}{\sin \theta}\right) \left(\frac{\cos \phi}{\sin \phi}\right) = \frac{1 + m}{1 - m}$
Using the identities $\cot x = \frac{\cos x}{\sin x}$, we get:
$\cot \theta \cot \phi = \frac{1 + m}{1 - m}$
We want to express $\tan \theta$. Since $\cot \theta = \frac{1}{\tan \theta}$ and $\cot \phi = \frac{1}{\tan \phi}$, we can write:
$\frac{1}{\tan \theta} \frac{1}{\tan \phi} = \frac{1 + m}{1 - m}$
$\frac{1}{\tan \theta \tan \phi} = \frac{1 + m}{1 - m}$
Take the reciprocal of both sides:
$\tan \theta \tan \phi = \frac{1 - m}{1 + m}$
Divide both sides by $\tan \phi$ and replace $\frac{1}{\tan \phi}$ with $\cot \phi$:
$\tan \theta = \frac{1 - m}{1 + m} \frac{1}{\tan \phi}$
$\tan \theta = \frac{1 - m}{1 + m} \cot \phi$
This is the required identity.
Question 22. Find the value of the expression
$3[ sin^{4} (\frac{3\pi}{2}-\alpha) +sin^{4}(3\pi+\alpha)] -2[ sin^{6} ( \frac{\pi}{2}+\alpha )+sin^{6}(5\pi-\alpha)]$
Answer:
Let the given expression be denoted by $E$.
$E = 3[ \sin^{4} (\frac{3\pi}{2}-\alpha) +\sin^{4}(3\pi+\alpha)] -2[ \sin^{6} ( \frac{\pi}{2}+\alpha )+\sin^{6}(5\pi-\alpha)]$
We simplify the terms inside the square brackets using trigonometric identities for allied angles.
For the first term: $\sin(\frac{3\pi}{2}-\alpha)$
Using the identity $\sin(\frac{3\pi}{2}-x) = -\cos x$, we have:
$\sin(\frac{3\pi}{2}-\alpha) = -\cos \alpha$
Therefore, $\sin^4(\frac{3\pi}{2}-\alpha) = (-\cos \alpha)^4 = \cos^4 \alpha$
For the second term: $\sin(3\pi+\alpha)$
Using the identity $\sin(n\pi+x) = (-1)^n \sin x$, where $n=3$ (odd), we have:
$\sin(3\pi+\alpha) = (-1)^3 \sin \alpha = -\sin \alpha$
Therefore, $\sin^4(3\pi+\alpha) = (-\sin \alpha)^4 = \sin^4 \alpha$
The first part of the expression becomes:
$3[ \sin^{4} (\frac{3\pi}{2}-\alpha) +\sin^{4}(3\pi+\alpha)] = 3[\cos^4 \alpha + \sin^4 \alpha]$
For the third term: $\sin(\frac{\pi}{2}+\alpha)$
Using the identity $\sin(\frac{\pi}{2}+x) = \cos x$, we have:
$\sin(\frac{\pi}{2}+\alpha) = \cos \alpha$
Therefore, $\sin^6(\frac{\pi}{2}+\alpha) = (\cos \alpha)^6 = \cos^6 \alpha$
For the fourth term: $\sin(5\pi-\alpha)$
Using the identity $\sin(n\pi-x) = (-1)^{n+1} \sin x$, where $n=5$ (odd), we have:
$\sin(5\pi-\alpha) = (-1)^{5+1} \sin \alpha = (-1)^6 \sin \alpha = \sin \alpha$
Alternatively, $\sin(5\pi-\alpha) = \sin(\pi + 4\pi - \alpha) = \sin(\pi - \alpha) = \sin \alpha$
Therefore, $\sin^6(5\pi-\alpha) = (\sin \alpha)^6 = \sin^6 \alpha$
The second part of the expression becomes:
$2[ \sin^{6} ( \frac{\pi}{2}+\alpha )+\sin^{6}(5\pi-\alpha)] = 2[\cos^6 \alpha + \sin^6 \alpha]$
Now substitute these simplified parts back into the original expression $E$:
$E = 3[\cos^4 \alpha + \sin^4 \alpha] - 2[\cos^6 \alpha + \sin^6 \alpha]$
We use the identities:
$\sin^4 x + \cos^4 x = (\sin^2 x + \cos^2 x)^2 - 2\sin^2 x \cos^2 x = 1^2 - 2\sin^2 x \cos^2 x = 1 - 2\sin^2 x \cos^2 x$
$\sin^6 x + \cos^6 x = (\sin^2 x + \cos^2 x)(\sin^4 x - \sin^2 x \cos^2 x + \cos^4 x) = 1 \cdot ((\sin^4 x + \cos^4 x) - \sin^2 x \cos^2 x)$
$\sin^6 x + \cos^6 x = (1 - 2\sin^2 x \cos^2 x) - \sin^2 x \cos^2 x = 1 - 3\sin^2 x \cos^2 x$
Substitute these identities into the expression for $E$:
$E = 3(1 - 2\sin^2 \alpha \cos^2 \alpha) - 2(1 - 3\sin^2 \alpha \cos^2 \alpha)$
$E = 3 - 6\sin^2 \alpha \cos^2 \alpha - 2 + 6\sin^2 \alpha \cos^2 \alpha$
$E = (3 - 2) + (-6\sin^2 \alpha \cos^2 \alpha + 6\sin^2 \alpha \cos^2 \alpha)$
$E = 1 + 0$
$E = 1$
The value of the expression is 1.
Question 23. If a cos 2θ + b sin 2θ = c has $α$ and $β$ as its roots, then prove that tan $α$ + tan $β$ = $\frac{2}{a \;+\; c}$
[Hint: Use the identities $cos \ 2θ = \frac{1\;-\;tan^{2}\theta}{1\;+\;tan^{2}\theta}$ and $sin \;2θ = \frac{2 tan \;\theta}{1 \;+\; tan^{2}\;\theta}$ ]
Answer:
Given:
The equation $a \cos 2\theta + b \sin 2\theta = c$ has roots $\alpha$ and $\beta$. This means that when $\theta = \alpha$ or $\theta = \beta$, the equation is satisfied.
To Prove:
$\tan \alpha + \tan \beta = \frac{2}{a + c}$
(Note: Based on the derivation below, the result appears to be $\frac{2b}{a+c}$. There might be a typo in the question as stated. We will prove $\tan \alpha + \tan \beta = \frac{2b}{a+c}$)
Proof:
We are given the equation:
$a \cos 2\theta + b \sin 2\theta = c$
Using the hints, we substitute the identities $\cos 2\theta = \frac{1-\tan^2\theta}{1+\tan^2\theta}$ and $\sin 2\theta = \frac{2\tan\theta}{1+\tan^2\theta}$.
Let $t = \tan \theta$. Substituting these into the equation, we get:
$a \left(\frac{1-t^2}{1+t^2}\right) + b \left(\frac{2t}{1+t^2}\right) = c$
... (i)
Multiply both sides by $(1+t^2)$, assuming $1+\tan^2\theta \neq 0$, which is true for real $\theta$:
$a(1-t^2) + b(2t) = c(1+t^2)$
Expand and rearrange the terms to form a quadratic equation in $t$:
$a - at^2 + 2bt = c + ct^2$
Move all terms to one side:
$0 = ct^2 + at^2 - 2bt + c - a$
Combine the $t^2$ terms:
$(c+a)t^2 - 2bt + (c-a) = 0$
... (ii)
This is a quadratic equation in $t = \tan \theta$. Since $\alpha$ and $\beta$ are the roots of the original equation in $\theta$, the values $t_1 = \tan \alpha$ and $t_2 = \tan \beta$ are the roots of this quadratic equation (ii), provided $\cos \alpha \neq 0$ and $\cos \beta \neq 0$ (so that $\tan \alpha$ and $\tan \beta$ are defined).
For a quadratic equation $Ax^2 + Bx + C = 0$, the sum of the roots is given by $-\frac{B}{A}$.
In our quadratic equation $(c+a)t^2 - 2bt + (c-a) = 0$, we have $A = c+a$, $B = -2b$, and $C = c-a$.
The sum of the roots, $\tan \alpha + \tan \beta$, is:
$\tan \alpha + \tan \beta = -\frac{-2b}{c+a}$
$\tan \alpha + \tan \beta = \frac{2b}{c+a}$
or
$\tan \alpha + \tan \beta = \frac{2b}{a+c}$
Comparing this with the expression we were asked to prove ($\frac{2}{a+c}$), it appears there is a factor of 'b' missing in the question statement. Assuming the equation and the hint are correct, the sum of the tangents of the roots is $\frac{2b}{a+c}$.
If the question intended to ask for $\frac{2b}{a+c}$, then the proof is complete.
If the question is stated exactly as intended, there might be another approach or the initial equation/desired result is incorrect.
Assuming the derived result $\tan \alpha + \tan \beta = \frac{2b}{a+c}$ is the expected answer based on the provided equation and hint, the proof is as shown above.
Question 24. If x = sec φ – tan φ and y = cosec φ + cot φ then show that xy + x – y + 1 = 0
[Hint: Find xy + 1 and then show that x – y = – (xy + 1)]
Answer:
Given:
$x = \sec \phi - \tan \phi$
$y = \text{cosec} \phi + \cot \phi$
To Prove:
$xy + x - y + 1 = 0$
Proof:
First, express $x$ and $y$ in terms of $\sin \phi$ and $\cos \phi$:
$x = \sec \phi - \tan \phi = \frac{1}{\cos \phi} - \frac{\sin \phi}{\cos \phi} = \frac{1 - \sin \phi}{\cos \phi}$
$y = \text{cosec} \phi + \cot \phi = \frac{1}{\sin \phi} + \frac{\cos \phi}{\sin \phi} = \frac{1 + \cos \phi}{\sin \phi}$
Now, let's calculate the product $xy$:
$xy = \left(\frac{1 - \sin \phi}{\cos \phi}\right) \left(\frac{1 + \cos \phi}{\sin \phi}\right)$
$xy = \frac{(1 - \sin \phi)(1 + \cos \phi)}{\cos \phi \sin \phi}$
$xy = \frac{1 \cdot 1 + 1 \cdot \cos \phi - \sin \phi \cdot 1 - \sin \phi \cos \phi}{\cos \phi \sin \phi}$
$xy = \frac{1 + \cos \phi - \sin \phi - \sin \phi \cos \phi}{\cos \phi \sin \phi}$
Next, let's calculate $xy + 1$ as suggested by the hint:
$xy + 1 = \frac{1 + \cos \phi - \sin \phi - \sin \phi \cos \phi}{\cos \phi \sin \phi} + 1$
$xy + 1 = \frac{1 + \cos \phi - \sin \phi - \sin \phi \cos \phi + \cos \phi \sin \phi}{\cos \phi \sin \phi}$
$\sin \phi \cos \phi - \sin \phi \cos \phi = 0$
[Terms cancel out]
$xy + 1 = \frac{1 + \cos \phi - \sin \phi}{\cos \phi \sin \phi}$
... (i)
Now, let's calculate $x - y$:
$x - y = \frac{1 - \sin \phi}{\cos \phi} - \frac{1 + \cos \phi}{\sin \phi}$
Find a common denominator, which is $\cos \phi \sin \phi$:
$x - y = \frac{(1 - \sin \phi)\sin \phi}{\cos \phi \sin \phi} - \frac{(1 + \cos \phi)\cos \phi}{\cos \phi \sin \phi}$
$x - y = \frac{(1 - \sin \phi)\sin \phi - (1 + \cos \phi)\cos \phi}{\cos \phi \sin \phi}$
$x - y = \frac{\sin \phi - \sin^2 \phi - (\cos \phi + \cos^2 \phi)}{\cos \phi \sin \phi}$
$x - y = \frac{\sin \phi - \sin^2 \phi - \cos \phi - \cos^2 \phi}{\cos \phi \sin \phi}$
Group the $\sin^2 \phi$ and $\cos^2 \phi$ terms:
$x - y = \frac{\sin \phi - \cos \phi - (\sin^2 \phi + \cos^2 \phi)}{\cos \phi \sin \phi}$
Using the identity $\sin^2 \phi + \cos^2 \phi = 1$:
$x - y = \frac{\sin \phi - \cos \phi - 1}{\cos \phi \sin \phi}$
... (ii)
Now, compare equation (i) and equation (ii).
From (i), $xy + 1 = \frac{1 + \cos \phi - \sin \phi}{\cos \phi \sin \phi}$.
From (ii), $x - y = \frac{\sin \phi - \cos \phi - 1}{\cos \phi \sin \phi}$.
Notice that the numerator of (ii) is the negative of the numerator of (i):
$- (1 + \cos \phi - \sin \phi) = -1 - \cos \phi + \sin \phi = \sin \phi - \cos \phi - 1$
Thus, we can write:
$x - y = \frac{-(1 + \cos \phi - \sin \phi)}{\cos \phi \sin \phi}$
$x - y = - \left(\frac{1 + \cos \phi - \sin \phi}{\cos \phi \sin \phi}\right)$
Substitute the expression for $xy + 1$ from equation (i):
$x - y = -(xy + 1)$
... (iii)
Rearrange equation (iii) to get the desired result:
$x - y = -xy - 1$
Add $xy$ and $1$ to both sides:
$xy + x - y + 1 = 0$
Hence, the identity is proven.
Question 25. If θ lies in the first quadrant and cos θ = $\frac{8}{17}$ then find the value of
cos (30° + θ) + cos (45° – θ) + cos (120° – θ).
Answer:
Given:
$\cos \theta = \frac{8}{17}$, where $\theta$ is in the first quadrant.
To Find:
The value of the expression $\cos (30° + \theta) + \cos (45° – \theta) + \cos (120° – θ)$.
Solution:
First, we find the value of $\sin \theta$. Since $\theta$ is in the first quadrant, $\sin \theta$ is positive.
Using the identity $\sin^2 \theta + \cos^2 \theta = 1$:
$\sin^2 \theta = 1 - \cos^2 \theta$
$\sin^2 \theta = 1 - \left(\frac{8}{17}\right)^2$
$\sin^2 \theta = 1 - \frac{64}{289}$
$\sin^2 \theta = \frac{289 - 64}{289}$
$\sin^2 \theta = \frac{225}{289}$
$\sin \theta = \sqrt{\frac{225}{289}} = \frac{15}{17}$
(Since $\theta$ is in the first quadrant)
Now we use the cosine addition and subtraction formulas:
$\cos(A + B) = \cos A \cos B - \sin A \sin B$
$\cos(A - B) = \cos A \cos B + \sin A \sin B$
We also need the values of trigonometric functions for 30°, 45°, and 120°:
$\cos 30° = \frac{\sqrt{3}}{2}$, $\sin 30° = \frac{1}{2}$
$\cos 45° = \frac{1}{\sqrt{2}}$, $\sin 45° = \frac{1}{\sqrt{2}}$
$\cos 120° = \cos(180°-60°) = -\cos 60° = -\frac{1}{2}$, $\sin 120° = \sin(180°-60°) = \sin 60° = \frac{\sqrt{3}}{2}$
Let's evaluate each term of the expression using the calculated values of $\sin \theta$ and $\cos \theta$:
Term 1: $\cos (30° + \theta)$
$\cos (30° + \theta) = \cos 30° \cos \theta - \sin 30° \sin \theta$
Substitute the values:
$\cos (30° + \theta) = \frac{\sqrt{3}}{2} \cdot \frac{8}{17} - \frac{1}{2} \cdot \frac{15}{17}$
$\cos (30° + \theta) = \frac{8\sqrt{3}}{34} - \frac{15}{34} = \frac{8\sqrt{3} - 15}{34}$
... (i)
Term 2: $\cos (45° – \theta)$
$\cos (45° – \theta) = \cos 45° \cos \theta + \sin 45° \sin \theta$
Substitute the values:
$\cos (45° – \theta) = \frac{1}{\sqrt{2}} \cdot \frac{8}{17} + \frac{1}{\sqrt{2}} \cdot \frac{15}{17}$
$\cos (45° – \theta) = \frac{8}{17\sqrt{2}} + \frac{15}{17\sqrt{2}} = \frac{23}{17\sqrt{2}}$
Rationalize the denominator:
$\cos (45° – \theta) = \frac{23}{17\sqrt{2}} \cdot \frac{\sqrt{2}}{\sqrt{2}} = \frac{23\sqrt{2}}{34}$
... (ii)
Term 3: $\cos (120° – θ)$
$\cos (120° – θ) = \cos 120° \cos \theta + \sin 120° \sin \theta$
Substitute the values:
$\cos (120° – θ) = -\frac{1}{2} \cdot \frac{8}{17} + \frac{\sqrt{3}}{2} \cdot \frac{15}{17}$
$\cos (120° – θ) = -\frac{8}{34} + \frac{15\sqrt{3}}{34} = \frac{15\sqrt{3} - 8}{34}$
... (iii)
Now, add the values from (i), (ii), and (iii):
Expression Value = $\cos (30° + \theta) + \cos (45° – \theta) + \cos (120° – θ)$
Expression Value = $\frac{8\sqrt{3} - 15}{34} + \frac{23\sqrt{2}}{34} + \frac{15\sqrt{3} - 8}{34}$
Expression Value = $\frac{(8\sqrt{3} - 15) + 23\sqrt{2} + (15\sqrt{3} - 8)}{34}$
Expression Value = $\frac{8\sqrt{3} - 15 + 23\sqrt{2} + 15\sqrt{3} - 8}{34}$
Group like terms:
Expression Value = $\frac{(8\sqrt{3} + 15\sqrt{3}) + 23\sqrt{2} + (-15 - 8)}{34}$
Expression Value = $\frac{23\sqrt{3} + 23\sqrt{2} - 23}{34}$
Factor out 23 from the numerator:
Expression Value = $\frac{23(\sqrt{3} + \sqrt{2} - 1)}{34}$
The value of the expression is $\frac{23(\sqrt{3} + \sqrt{2} - 1)}{34}$.
Question 26. Find the value of the expression cos4 $\frac{\pi}{8}$ + cos4 $\frac{3\pi}{8}$ + cos4 $\frac{5\pi}{8}$ + cos4 $\frac{7\pi}{8}$
Hint: Simplify the expression to
2 (cos4 $\frac{\pi}{8}$ + cos4 $\frac{3\pi}{8}$) = $2\left[ \left( \cos^2\frac{\pi}{8}+\cos^2\frac{3\pi}{8} \right)^2 - 2\cos^2\frac{\pi}{8}\cos^2\frac{3\pi}{8}\right]$
Answer:
To Find:
The value of the expression $E = \cos^4 \frac{\pi}{8} + \cos^4 \frac{3\pi}{8} + \cos^4 \frac{5\pi}{8} + \cos^4 \frac{7\pi}{8}$.
Solution:
We use the angle relationships:
$\frac{7\pi}{8} = \pi - \frac{\pi}{8}$
$\frac{5\pi}{8} = \pi - \frac{3\pi}{8}$
Using the identity $\cos(\pi - x) = -\cos x$, we have:
$\cos \frac{7\pi}{8} = \cos(\pi - \frac{\pi}{8}) = -\cos \frac{\pi}{8}$
$\cos \frac{5\pi}{8} = \cos(\pi - \frac{3\pi}{8}) = -\cos \frac{3\pi}{8}$
Since the terms in the expression are raised to the power of 4 (an even power), the negative sign disappears:
$\cos^4 \frac{7\pi}{8} = (-\cos \frac{\pi}{8})^4 = \cos^4 \frac{\pi}{8}$
$\cos^4 \frac{5\pi}{8} = (-\cos \frac{3\pi}{8})^4 = \cos^4 \frac{3\pi}{8}$
Substitute these back into the original expression:
$E = \cos^4 \frac{\pi}{8} + \cos^4 \frac{3\pi}{8} + \cos^4 \frac{3\pi}{8} + \cos^4 \frac{\pi}{8}$
$E = 2 \cos^4 \frac{\pi}{8} + 2 \cos^4 \frac{3\pi}{8}$
$E = 2 (\cos^4 \frac{\pi}{8} + \cos^4 \frac{3\pi}{8})$
... (i)
Now consider the second term inside the parenthesis. We use the angle relationship:
$\frac{3\pi}{8} = \frac{4\pi - \pi}{8} = \frac{\pi}{2} - \frac{\pi}{8}$
Using the identity $\cos(\frac{\pi}{2} - x) = \sin x$, we have:
$\cos \frac{3\pi}{8} = \cos(\frac{\pi}{2} - \frac{\pi}{8}) = \sin \frac{\pi}{8}$
Substitute this into equation (i):
$E = 2 (\cos^4 \frac{\pi}{8} + (\sin \frac{\pi}{8})^4)$
$E = 2 (\cos^4 \frac{\pi}{8} + \sin^4 \frac{\pi}{8})$
... (ii)
We use the identity for the sum of fourth powers of sine and cosine: $\sin^4 x + \cos^4 x = 1 - 2 \sin^2 x \cos^2 x$.
$\cos^4 \frac{\pi}{8} + \sin^4 \frac{\pi}{8} = 1 - 2 \sin^2 \frac{\pi}{8} \cos^2 \frac{\pi}{8}$
... (iii)
Now consider the term $\sin^2 \frac{\pi}{8} \cos^2 \frac{\pi}{8}$. We can rewrite this using the double angle identity for sine, $2 \sin x \cos x = \sin 2x$, which implies $\sin x \cos x = \frac{1}{2} \sin 2x$.
$\sin^2 \frac{\pi}{8} \cos^2 \frac{\pi}{8} = \left(\sin \frac{\pi}{8} \cos \frac{\pi}{8}\right)^2 = \left(\frac{1}{2} \sin (2 \cdot \frac{\pi}{8})\right)^2$
$\sin^2 \frac{\pi}{8} \cos^2 \frac{\pi}{8} = \left(\frac{1}{2} \sin \frac{\pi}{4}\right)^2$
We know that $\sin \frac{\pi}{4} = \frac{1}{\sqrt{2}}$.
$\left(\frac{1}{2} \sin \frac{\pi}{4}\right)^2 = \left(\frac{1}{2} \cdot \frac{1}{\sqrt{2}}\right)^2 = \left(\frac{1}{2\sqrt{2}}\right)^2 = \frac{1}{4 \cdot 2} = \frac{1}{8}$
So, substitute this value back into equation (iii):
$\cos^4 \frac{\pi}{8} + \sin^4 \frac{\pi}{8} = 1 - 2 \cdot \frac{1}{8}$
$\cos^4 \frac{\pi}{8} + \sin^4 \frac{\pi}{8} = 1 - \frac{\cancel{2}}{\cancel{8}_{4}} = 1 - \frac{1}{4}$
$\cos^4 \frac{\pi}{8} + \sin^4 \frac{\pi}{8} = \frac{4 - 1}{4} = \frac{3}{4}$
... (iv)
Finally, substitute the result from equation (iv) back into equation (ii) for the expression $E$:
$E = 2 \left(\cos^4 \frac{\pi}{8} + \sin^4 \frac{\pi}{8}\right)$
$E = 2 \left(\frac{3}{4}\right)$
$E = \cancel{2} \cdot \frac{3}{\cancel{4}_{2}}$
$E = \frac{3}{2}$
The value of the expression is $\frac{3}{2}$.
Note: Following the hint's specific expansion of the grouped terms:
$2\left[ \left( \cos^2\frac{\pi}{8}+\cos^2\frac{3\pi}{8} \right)^2 - 2\cos^2\frac{\pi}{8}\cos^2\frac{3\pi}{8}\right]$
Substitute $\cos \frac{3\pi}{8} = \sin \frac{\pi}{8}$:
$2\left[ \left( \cos^2\frac{\pi}{8}+\sin^2\frac{\pi}{8} \right)^2 - 2\cos^2\frac{\pi}{8}\sin^2\frac{\pi}{8}\right]$
Using $\cos^2 x + \sin^2 x = 1$:
$2\left[ (1)^2 - 2\cos^2\frac{\pi}{8}\sin^2\frac{\pi}{8}\right]$
$2\left[ 1 - 2(\sin\frac{\pi}{8}\cos\frac{\pi}{8})^2\right]$
Using $\sin x \cos x = \frac{1}{2} \sin 2x$ and $\sin \frac{\pi}{4} = \frac{1}{\sqrt{2}}$ as before:
$2\left[ 1 - 2\left(\frac{1}{2}\sin(2\cdot\frac{\pi}{8})\right)^2\right]$
$2\left[ 1 - 2\left(\frac{1}{2}\sin\frac{\pi}{4}\right)^2\right]$
$2\left[ 1 - 2\left(\frac{1}{2}\cdot\frac{1}{\sqrt{2}}\right)^2\right]$
$2\left[ 1 - 2\left(\frac{1}{2\sqrt{2}}\right)^2\right]$
$2\left[ 1 - 2\left(\frac{1}{8}\right)\right]$
$2\left[ 1 - \frac{1}{4}\right]$
$2\left[ \frac{3}{4}\right] = \frac{3}{2}$
Both methods yield the same result.
Question 27. Find the general solution of the equation 5cos2 θ + 7sin2 θ – 6 = 0
Answer:
Given:
The trigonometric equation $5\cos^2 \theta + 7\sin^2 \theta – 6 = 0$.
To Find:
The general solution of the given equation.
Solution:
We are given the equation:
$5\cos^2 \theta + 7\sin^2 \theta - 6 = 0$
We use the identity $\cos^2 \theta + \sin^2 \theta = 1$. We can rewrite $\cos^2 \theta$ as $1 - \sin^2 \theta$ or $\sin^2 \theta$ as $1 - \cos^2 \theta$. Let's express the equation in terms of $\sin^2 \theta$:
$5(1 - \sin^2 \theta) + 7\sin^2 \theta - 6 = 0$
[Using $\cos^2 \theta = 1 - \sin^2 \theta$]
Expand the equation:
$5 - 5\sin^2 \theta + 7\sin^2 \theta - 6 = 0$
Combine like terms:
$(7\sin^2 \theta - 5\sin^2 \theta) + (5 - 6) = 0$
$2\sin^2 \theta - 1 = 0$
Rearrange the equation to solve for $\sin^2 \theta$:
$2\sin^2 \theta = 1$
$\sin^2 \theta = \frac{1}{2}$
To find the general solution, we need to find an angle $\alpha$ such that $\sin^2 \alpha = \frac{1}{2}$.
We know that $\sin \frac{\pi}{4} = \frac{1}{\sqrt{2}}$.
Squaring this, we get $\sin^2 \frac{\pi}{4} = \left(\frac{1}{\sqrt{2}}\right)^2 = \frac{1}{2}$.
So, the equation becomes:
$\sin^2 \theta = \sin^2 \frac{\pi}{4}$
The general solution for $\sin^2 x = \sin^2 \alpha$ is $x = n\pi \pm \alpha$, where $n$ is an integer ($n \in \mathbb{Z}$).
Applying this formula to our equation with $x = \theta$ and $\alpha = \frac{\pi}{4}$, the general solution is:
$\theta = n\pi \pm \frac{\pi}{4}$, where $n \in \mathbb{Z}$.
Alternatively, we could have expressed the equation in terms of $\cos^2 \theta$:
$5\cos^2 \theta + 7(1 - \cos^2 \theta) - 6 = 0$
$5\cos^2 \theta + 7 - 7\cos^2 \theta - 6 = 0$
$-2\cos^2 \theta + 1 = 0$
$1 = 2\cos^2 \theta$
$\cos^2 \theta = \frac{1}{2}$
We know that $\cos \frac{\pi}{4} = \frac{1}{\sqrt{2}}$.
Squaring this, we get $\cos^2 \frac{\pi}{4} = \left(\frac{1}{\sqrt{2}}\right)^2 = \frac{1}{2}$.
So, the equation becomes:
$\cos^2 \theta = \cos^2 \frac{\pi}{4}$
The general solution for $\cos^2 x = \cos^2 \alpha$ is $x = n\pi \pm \alpha$, where $n \in \mathbb{Z}$.
Applying this formula, the general solution is:
$\theta = n\pi \pm \frac{\pi}{4}$, where $n \in \mathbb{Z}$.
Both approaches yield the same general solution.
The general solution is $\theta = n\pi \pm \frac{\pi}{4}$, $n \in \mathbb{Z}$.
Question 28. Find the general solution of the equation
sin x – 3sin 2x + sin 3x = cos x – 3cos 2x + cos 3x
Answer:
Given:
The equation $\sin x – 3\sin 2x + \sin 3x = \cos x – 3\cos 2x + \cos 3x$.
To Find:
The general solution of the given equation.
Solution:
Rearrange the equation by grouping terms:
$(\sin x + \sin 3x) - 3\sin 2x = (\cos x + \cos 3x) - 3\cos 2x$
Use the sum-to-product formulas:
$\sin A + \sin B = 2 \sin \frac{A+B}{2} \cos \frac{A-B}{2}$
$\cos A + \cos B = 2 \cos \frac{A+B}{2} \cos \frac{A-B}{2}$
Applying these to the grouped terms (with $A=3x, B=x$):
$\sin x + \sin 3x = 2 \sin \frac{3x+x}{2} \cos \frac{3x-x}{2} = 2 \sin 2x \cos x$
$\cos x + \cos 3x = 2 \cos \frac{3x+x}{2} \cos \frac{3x-x}{2} = 2 \cos 2x \cos x$
Substitute these results back into the equation:
$2 \sin 2x \cos x - 3\sin 2x = 2 \cos 2x \cos x - 3\cos 2x$
Move all terms to the left side:
$2 \sin 2x \cos x - 3\sin 2x - 2 \cos 2x \cos x + 3\cos 2x = 0$
Group terms with $\sin 2x$ and $\cos 2x$:
$\sin 2x (2 \cos x - 3) - \cos 2x (2 \cos x - 3) = 0$
Factor out the common term $(2 \cos x - 3)$:
$(2 \cos x - 3)(\sin 2x - \cos 2x) = 0$
... (i)
For equation (i) to hold, at least one of the factors must be equal to zero.
Case 1: $2 \cos x - 3 = 0$
$2 \cos x = 3$
$\cos x = \frac{3}{2}$
Since the range of the cosine function is $[-1, 1]$, $\cos x = \frac{3}{2}$ has no real solution.
Case 2: $\sin 2x - \cos 2x = 0$
$\sin 2x = \cos 2x$
Divide both sides by $\cos 2x$ (assuming $\cos 2x \neq 0$; if $\cos 2x = 0$, then $\sin 2x = 0$, which contradicts $\sin^2 y + \cos^2 y = 1$):
$\frac{\sin 2x}{\cos 2x} = 1$
$\tan 2x = 1$
We know that $\tan \frac{\pi}{4} = 1$. So, we can write:
$\tan 2x = \tan \frac{\pi}{4}$
The general solution for $\tan y = \tan \alpha$ is given by $y = n\pi + \alpha$, where $n$ is an integer ($n \in \mathbb{Z}$).
Here, $y = 2x$ and $\alpha = \frac{\pi}{4}$.
$2x = n\pi + \frac{\pi}{4}$
Divide by 2 to solve for $x$:
$x = \frac{n\pi}{2} + \frac{\pi}{8}$, where $n \in \mathbb{Z}$.
This is the general solution for the given equation.
Question 29. Find the general solution of the equation ($\sqrt{3}$ - 1) cos θ + ($\sqrt{3}$ + 1) sin θ = 2
[Hint: Put $\sqrt{3}$ - 1 = r sin $α$ , $\sqrt{3}$ + 1 = r cos $α$ which gives tan $α$ = tan $\left( \frac{\pi}{4} -\frac{\pi}{6} \right)$ $α$ = $\frac{\pi}{12}$ ]
Answer:
Given:
The equation $(\sqrt{3} - 1) \cos \theta + (\sqrt{3} + 1) \sin \theta = 2$.
To Find:
The general solution of the given equation.
Solution:
We are given the equation in the form $a \cos \theta + b \sin \theta = c$, where $a = \sqrt{3} - 1$, $b = \sqrt{3} + 1$, and $c = 2$.
To solve this type of equation, we can transform the left side into a single trigonometric function. Following the hint, let:
$\sqrt{3} - 1 = r \sin \alpha$
... (i)
$\sqrt{3} + 1 = r \cos \alpha$
... (ii)
Square equations (i) and (ii) and add them to find $r^2$:
$(\sqrt{3} - 1)^2 + (\sqrt{3} + 1)^2 = (r \sin \alpha)^2 + (r \cos \alpha)^2$
$(3 - 2\sqrt{3} + 1) + (3 + 2\sqrt{3} + 1) = r^2 \sin^2 \alpha + r^2 \cos^2 \alpha$
$(4 - 2\sqrt{3}) + (4 + 2\sqrt{3}) = r^2 (\sin^2 \alpha + \cos^2 \alpha)$
$8 = r^2 (1)$
$r^2 = 8$
$r = \sqrt{8} = 2\sqrt{2}$ (We take the positive value for $r$).
Now, divide equation (i) by equation (ii) to find $\tan \alpha$:
$\frac{r \sin \alpha}{r \cos \alpha} = \frac{\sqrt{3} - 1}{\sqrt{3} + 1}$
$\tan \alpha = \frac{\sqrt{3} - 1}{\sqrt{3} + 1}$
Multiply the numerator and denominator by the conjugate of the denominator, $\sqrt{3} - 1$:
$\tan \alpha = \frac{(\sqrt{3} - 1)(\sqrt{3} - 1)}{(\sqrt{3} + 1)(\sqrt{3} - 1)} = \frac{(\sqrt{3})^2 - 2(\sqrt{3})(1) + 1^2}{(\sqrt{3})^2 - 1^2}$
$\tan \alpha = \frac{3 - 2\sqrt{3} + 1}{3 - 1} = \frac{4 - 2\sqrt{3}}{2} = 2 - \sqrt{3}$
We recognize $2 - \sqrt{3}$ as the value of $\tan 15^\circ$ or $\tan \frac{\pi}{12}$. As per the hint, $\tan \alpha = \tan \left( \frac{\pi}{4} - \frac{\pi}{6} \right) = \tan \left( \frac{3\pi - 2\pi}{12} \right) = \tan \frac{\pi}{12}$.
So, we can take $\alpha = \frac{\pi}{12}$ (since $\sqrt{3}-1 > 0$ and $\sqrt{3}+1 > 0$, $\sin \alpha > 0$ and $\cos \alpha > 0$, so $\alpha$ is in the first quadrant, and $\frac{\pi}{12}$ is in the first quadrant).
Substitute the expressions for $\sqrt{3}-1$ and $\sqrt{3}+1$ back into the original equation:
$(r \sin \alpha) \cos \theta + (r \cos \alpha) \sin \theta = 2$
$r (\sin \alpha \cos \theta + \cos \alpha \sin \theta) = 2$
Using the sum formula for sine, $\sin(A+B) = \sin A \cos B + \cos A \sin B$:
$r \sin(\theta + \alpha) = 2$
... (iii)
Substitute the values of $r = 2\sqrt{2}$ and $\alpha = \frac{\pi}{12}$ into equation (iii):
$2\sqrt{2} \sin\left(\theta + \frac{\pi}{12}\right) = 2$
Divide both sides by $2\sqrt{2}$:
$\sin\left(\theta + \frac{\pi}{12}\right) = \frac{2}{2\sqrt{2}} = \frac{1}{\sqrt{2}}$
We know that $\sin \frac{\pi}{4} = \frac{1}{\sqrt{2}}$.
So, the equation is:
$\sin\left(\theta + \frac{\pi}{12}\right) = \sin \frac{\pi}{4}$
The general solution for $\sin y = \sin k$ is given by $y = n\pi + (-1)^n k$, where $n \in \mathbb{Z}$.
Here, $y = \theta + \frac{\pi}{12}$ and $k = \frac{\pi}{4}$.
Therefore, the general solution is:
$\theta + \frac{\pi}{12} = n\pi + (-1)^n \frac{\pi}{4}$, where $n \in \mathbb{Z}$.
To find $\theta$, subtract $\frac{\pi}{12}$ from both sides:
$\theta = n\pi + (-1)^n \frac{\pi}{4} - \frac{\pi}{12}$, where $n \in \mathbb{Z}$.
This can also be written as:
$\theta = n\pi + (-1)^n \frac{3\pi}{12} - \frac{\pi}{12}$, where $n \in \mathbb{Z}$.
The general solution is $\theta = n\pi + (-1)^n \frac{\pi}{4} - \frac{\pi}{12}$, where $n \in \mathbb{Z}$.
Question 30 to 59 (Multiple Choice Questions)
Choose the correct answer from the given four options in the Exercises 30 to 59 (M.C.Q.).
Question 30. If sin θ + cosec θ = 2, then sin2 θ + cosec2 θ is equal to
(A) 1
(B) 4
(C) 2
(D) None of these
Answer:
Solution:
Given that $\sin \theta + \text{cosec} \theta = 2$.
We want to find the value of $\sin^2 \theta + \text{cosec}^2 \theta$.
Square both sides of the given equation:
$(\sin \theta + \text{cosec} \theta)^2 = (2)^2$
Expand the left side using the algebraic identity $(a+b)^2 = a^2 + 2ab + b^2$:
$\sin^2 \theta + 2 (\sin \theta)(\text{cosec} \theta) + \text{cosec}^2 \theta = 4$
We know that $\text{cosec} \theta = \frac{1}{\sin \theta}$. Therefore, $\sin \theta \cdot \text{cosec} \theta = \sin \theta \cdot \frac{1}{\sin \theta} = 1$ (assuming $\sin \theta \neq 0$).
Substitute this into the equation:
$\sin^2 \theta + 2(1) + \text{cosec}^2 \theta = 4$
$\sin^2 \theta + 2 + \text{cosec}^2 \theta = 4$
Subtract 2 from both sides of the equation:
$\sin^2 \theta + \text{cosec}^2 \theta = 4 - 2$
$\sin^2 \theta + \text{cosec}^2 \theta = 2$
Alternate Solution:
Given $\sin \theta + \text{cosec} \theta = 2$.
Rewrite $\text{cosec} \theta$ as $\frac{1}{\sin \theta}$:
$\sin \theta + \frac{1}{\sin \theta} = 2$
Multiply the entire equation by $\sin \theta$ (assuming $\sin \theta \neq 0$):
$\sin^2 \theta + 1 = 2 \sin \theta$
Rearrange the terms to form a quadratic equation in $\sin \theta$:
$\sin^2 \theta - 2 \sin \theta + 1 = 0$
This equation is in the form of a perfect square trinomial $(a-b)^2 = a^2 - 2ab + b^2$, where $a = \sin \theta$ and $b = 1$.
$(\sin \theta - 1)^2 = 0$
Taking the square root of both sides:
$\sin \theta - 1 = 0$
Solving for $\sin \theta$:
$\sin \theta = 1$
If $\sin \theta = 1$, then $\text{cosec} \theta = \frac{1}{\sin \theta} = \frac{1}{1} = 1$.
Now, substitute these values into the expression we need to find:
$\sin^2 \theta + \text{cosec}^2 \theta = (1)^2 + (1)^2 = 1 + 1 = 2$
Both methods yield the same result.
The value of $\sin^2 \theta + \text{cosec}^2 \theta$ is 2.
Comparing this with the given options:
(A) 1
(B) 4
(C) 2
(D) None of these
The correct option is (C).
The final answer is (C).
Question 31. If f(x) = cos2 x + sec2 x, then
(A) f (x) < 1
(B) f (x) = 1
(C) 2 < f (x) < 1
(D) f(x) ≥ 2
[Hint: A.M ≥ G.M.]
Answer:
Solution:
We are given the function $f(x) = \cos^2 x + \sec^2 x$.
We need to determine the range or possible values of $f(x)$.
We know that for any real number $y$, $y^2 \ge 0$. Also, the domain of $\sec x$ is such that $\cos x \neq 0$. In this domain, $\cos^2 x > 0$ and $\sec^2 x > 0$.
We can apply the Arithmetic Mean - Geometric Mean (A.M. - G.M.) inequality for the two positive numbers $\cos^2 x$ and $\sec^2 x$. The inequality states that for non-negative numbers $a$ and $b$, $\frac{a+b}{2} \ge \sqrt{ab}$, with equality holding if and only if $a=b$.
Let $a = \cos^2 x$ and $b = \sec^2 x$.
Applying the A.M. - G.M. inequality:
$\frac{\cos^2 x + \sec^2 x}{2} \ge \sqrt{\cos^2 x \cdot \sec^2 x}$
We know that $\sec x = \frac{1}{\cos x}$. So, $\sec^2 x = \frac{1}{\cos^2 x}$.
Substitute this into the inequality:
$\frac{\cos^2 x + \sec^2 x}{2} \ge \sqrt{\cos^2 x \cdot \frac{1}{\cos^2 x}}$
Assuming $\cos x \neq 0$, $\cos^2 x \cdot \frac{1}{\cos^2 x} = 1$.
$\frac{\cos^2 x + \sec^2 x}{2} \ge \sqrt{1}$
$\frac{\cos^2 x + \sec^2 x}{2} \ge 1$
Multiply both sides by 2:
$\cos^2 x + \sec^2 x \ge 2$
Since $f(x) = \cos^2 x + \sec^2 x$, we have:
$f(x) \ge 2$
The equality $f(x) = 2$ holds when $\cos^2 x = \sec^2 x$, which means $\cos^4 x = 1$. This is true when $\cos^2 x = 1$, i.e., $\cos x = \pm 1$. This occurs when $x = n\pi$ for any integer $n$. In these cases, $\sec x = \pm 1$, and $\cos^2 x + \sec^2 x = (\pm 1)^2 + (\pm 1)^2 = 1 + 1 = 2$.
Thus, the minimum value of $f(x)$ is 2, and $f(x)$ can take any value greater than or equal to 2.
Therefore, $f(x) \ge 2$.
Comparing this with the given options:
(A) f (x) < 1
(B) f (x) = 1
(C) 2 < f (x) < 1
(D) f(x) ≥ 2
The correct option is (D).
The final answer is (D).
Question 32. If tan θ = $\frac{1}{2}$ and tan φ = $\frac{1}{3}$ , then the value of θ + φ is
(A) $\frac{π}{6}$
(B) π
(C) 0
(D) $\frac{π}{4}$
Answer:
Solution:
We are given that $\tan \theta = \frac{1}{2}$ and $\tan \phi = \frac{1}{3}$.
We need to find the value of $\theta + \phi$.
We can use the tangent addition formula:
$\tan(\theta + \phi) = \frac{\tan \theta + \tan \phi}{1 - \tan \theta \tan \phi}$
Substitute the given values of $\tan \theta$ and $\tan \phi$ into the formula:
$\tan(\theta + \phi) = \frac{\frac{1}{2} + \frac{1}{3}}{1 - (\frac{1}{2})(\frac{1}{3})}$
Calculate the numerator:
$\frac{1}{2} + \frac{1}{3} = \frac{3}{6} + \frac{2}{6} = \frac{3+2}{6} = \frac{5}{6}$
Calculate the denominator:
$1 - (\frac{1}{2})(\frac{1}{3}) = 1 - \frac{1 \times 1}{2 \times 3} = 1 - \frac{1}{6}$
To subtract, find a common denominator:
$1 - \frac{1}{6} = \frac{6}{6} - \frac{1}{6} = \frac{6-1}{6} = \frac{5}{6}$
Now substitute the numerator and denominator back into the formula for $\tan(\theta + \phi)$:
$\tan(\theta + \phi) = \frac{\frac{5}{6}}{\frac{5}{6}}$
$\tan(\theta + \phi) = 1$
We know that $\tan \frac{\pi}{4} = 1$.
Since $\tan(\theta + \phi) = 1$, and assuming $\theta$ and $\phi$ are angles for which the given $\tan$ values are defined (typically in the range $(-\frac{\pi}{2}, \frac{\pi}{2})$), the principal value of $\theta + \phi$ is $\frac{\pi}{4}$.
Thus, $\theta + \phi = \frac{\pi}{4}$.
Comparing this with the given options:
(A) $\frac{π}{6}$
(B) π
(C) 0
(D) $\frac{π}{4}$
The correct option is (D).
The final answer is (D).
Question 33. Which of the following is not correct?
(A) sin θ = $-\frac{1}{5}$
(B) cos θ = 1
(C) sec θ = $\frac{1}{2}$
(D) tan θ = 20
Answer:
Solution:
We need to identify which of the given trigonometric values is not correct, meaning it is outside the possible range of values for that trigonometric function.
Let's examine the range of each trigonometric function involved in the options:
For $\sin \theta$, the range of values is $[-1, 1]$. This means that for any angle $\theta$, $-1 \leq \sin \theta \leq 1$.
For $\cos \theta$, the range of values is $[-1, 1]$. This means that for any angle $\theta$, $-1 \leq \cos \theta \leq 1$.
For $\sec \theta$, which is the reciprocal of $\cos \theta$, the range of values is $(-\infty, -1] \cup [1, \infty)$. This means that for any angle $\theta$ where $\cos \theta \neq 0$, $\sec \theta \leq -1$ or $\sec \theta \ge 1$. Equivalently, $|\sec \theta| \ge 1$.
For $\tan \theta$, the range of values is $(-\infty, \infty)$. This means that for any angle $\theta$ where $\cos \theta \neq 0$, $\tan \theta$ can take any real value.
Now let's check each option against the valid range of the respective trigonometric function:
(A) $\sin \theta = -\frac{1}{5}$
The value $-\frac{1}{5}$ is between -1 and 1 (since $-1 \leq -\frac{1}{5} \leq 1$). This value is within the range of $\sin \theta$. So, this is a correct statement for some angle $\theta$.
(B) $\cos \theta = 1$
The value $1$ is between -1 and 1 (since $-1 \leq 1 \leq 1$). This value is within the range of $\cos \theta$. So, this is a correct statement for some angle $\theta$ (e.g., $\theta = 0$).
(C) $\sec \theta = \frac{1}{2}$
The value $\frac{1}{2}$ is $0.5$. According to the range of $\sec \theta$, the value must be less than or equal to -1 or greater than or equal to 1 (i.e., $|\sec \theta| \ge 1$). Since $0.5$ is not less than or equal to -1 and not greater than or equal to 1, this value is outside the range of $\sec \theta$. So, there is no angle $\theta$ for which $\sec \theta = \frac{1}{2}$. This statement is not correct.
(D) $\tan \theta = 20$
The value $20$ is a real number. According to the range of $\tan \theta$, which is $(-\infty, \infty)$, $\tan \theta$ can take any real value. So, this is a correct statement for some angle $\theta$.
Based on the analysis of the ranges, the statement that is not correct is (C) $\sec \theta = \frac{1}{2}$.
The correct option is (C).
The final answer is (C).
Question 34. The value of tan 1° tan 2° tan 3° ... tan 89° is
(A) 0
(B) 1
(C) $\frac{1}{2}$
(D) Not defined
Answer:
Solution:
We need to find the value of the product:
$P = \tan 1^\circ \tan 2^\circ \tan 3^\circ \dots \tan 89^\circ$
The product contains 89 terms, starting from $\tan 1^\circ$ and ending at $\tan 89^\circ$.
We can use the complementary angle identity for tangent: $\tan (90^\circ - \theta) = \cot \theta$.
We also know that $\cot \theta = \frac{1}{\tan \theta}$.
Therefore, $\tan \theta \cdot \tan (90^\circ - \theta) = \tan \theta \cdot \cot \theta = \tan \theta \cdot \frac{1}{\tan \theta} = 1$, provided that $\tan \theta \neq 0$.
Let's examine the terms in the product:
The angles range from $1^\circ$ to $89^\circ$. We can pair terms such that the sum of the angles is $90^\circ$.
- $\tan 1^\circ$ is paired with $\tan 89^\circ$
- $\tan 2^\circ$ is paired with $\tan 88^\circ$
- ...
- $\tan 44^\circ$ is paired with $\tan 46^\circ$
The middle term is $\tan 45^\circ$, which is unpaired in this grouping.
Let's write out the product by grouping these pairs:
$P = (\tan 1^\circ \cdot \tan 89^\circ) \cdot (\tan 2^\circ \cdot \tan 88^\circ) \cdot \dots \cdot (\tan 44^\circ \cdot \tan 46^\circ) \cdot \tan 45^\circ$
Using the identity $\tan \theta \cdot \tan (90^\circ - \theta) = 1$, we have:
- $\tan 1^\circ \cdot \tan 89^\circ = \tan 1^\circ \cdot \tan (90^\circ - 1^\circ) = \tan 1^\circ \cdot \cot 1^\circ = 1$
- $\tan 2^\circ \cdot \tan 88^\circ = \tan 2^\circ \cdot \tan (90^\circ - 2^\circ) = \tan 2^\circ \cdot \cot 2^\circ = 1$
- ...
- $\tan 44^\circ \cdot \tan 46^\circ = \tan 44^\circ \cdot \tan (90^\circ - 44^\circ) = \tan 44^\circ \cdot \cot 44^\circ = 1$
There are 44 such pairs, each having a product of 1.
The middle term is $\tan 45^\circ$. We know that $\tan 45^\circ = 1$.
Substitute these values back into the product $P$:
$P = \underbrace{1 \cdot 1 \cdot \dots \cdot 1}_{\text{44 pairs}} \cdot 1$
$P = 1^{44} \cdot 1$
$P = 1 \cdot 1$
$P = 1$
The value of the product $\tan 1^\circ \tan 2^\circ \dots \tan 89^\circ$ is 1.
Comparing this with the given options:
(A) 0
(B) 1
(C) $\frac{1}{2}$
(D) Not defined
The correct option is (B).
The final answer is (B).
Question 35. The value of $\frac{1 \;−\; tan^2 \;15°}{1 \;+\; tan^2 \;15°}$ is
(A) 1
(B) $\sqrt{3}$
(C) $\frac{\sqrt{3}}{2}$
(D) 2
Answer:
Solution:
We need to find the value of the expression $\frac{1 - \tan^2 15^\circ}{1 + \tan^2 15^\circ}$.
This expression has the form $\frac{1 - \tan^2 \theta}{1 + \tan^2 \theta}$, where $\theta = 15^\circ$.
We can use the double angle identity for cosine, which can be expressed in terms of tangent:
$\cos 2\theta = \frac{1 - \tan^2 \theta}{1 + \tan^2 \theta}$
(Trigonometric Identity)
Comparing the given expression with the right side of the identity, we can see that $\theta = 15^\circ$.
Therefore, the given expression is equal to $\cos(2 \times 15^\circ)$.
Calculate the argument of the cosine function:
$2 \times 15^\circ = 30^\circ$
So, the expression is equal to $\cos 30^\circ$.
We know the standard value of $\cos 30^\circ$:
$\cos 30^\circ = \frac{\sqrt{3}}{2}$
Thus, the value of the expression is $\frac{\sqrt{3}}{2}$.
Comparing this value with the given options:
(A) 1
(B) $\sqrt{3}$
(C) $\frac{\sqrt{3}}{2}$
(D) 2
The calculated value matches option (C).
The final answer is (C).
Question 36. The value of cos 1° cos 2° cos 3° ... cos 179° is
(A) $\frac{1}{\sqrt{2}}$
(B) 0
(C) 1
(D) –1
Answer:
Solution:
We need to find the value of the product:
$P = \cos 1^\circ \cos 2^\circ \cos 3^\circ \dots \cos 179^\circ$
The product consists of the cosine of every integer angle from $1^\circ$ to $179^\circ$.
We can list some terms in the product:
$P = \cos 1^\circ \cdot \cos 2^\circ \cdot \dots \cdot \cos 89^\circ \cdot \cos 90^\circ \cdot \cos 91^\circ \cdot \dots \cdot \cos 179^\circ$
Notice that the angle $90^\circ$ is included in the sequence of angles from $1^\circ$ to $179^\circ$.
We know the exact value of $\cos 90^\circ$:
$\cos 90^\circ = 0$
(Standard value)
Since $\cos 90^\circ = 0$ is one of the terms in the product $P$, and any product involving a factor of zero is equal to zero, the entire product must be zero.
$P = \cos 1^\circ \cdot \cos 2^\circ \cdot \dots \cdot \cos 89^\circ \cdot (0) \cdot \cos 91^\circ \cdot \dots \cdot \cos 179^\circ = 0$
Thus, the value of $\cos 1^\circ \cos 2^\circ \cos 3^\circ \dots \cos 179^\circ$ is 0.
Comparing this with the given options:
(A) $\frac{1}{\sqrt{2}}$
(B) 0
(C) 1
(D) –1
The calculated value matches option (B).
The final answer is (B).
Question 37. If tan θ = 3 and θ lies in third quadrant, then the value of sin θ is
(A) $\frac{1}{\sqrt{10}}$
(B) $-\frac{1}{\sqrt{10}}$
(C) $\frac{-3}{\sqrt{10}}$
(D) $\frac{3}{\sqrt{10}}$
Answer:
Solution:
We are given that $\tan \theta = 3$ and that $\theta$ lies in the third quadrant.
We need to find the value of $\sin \theta$.
We can use the trigonometric identity relating $\tan \theta$ and $\sec \theta$:
$\sec^2 \theta = 1 + \tan^2 \theta$
Substitute the given value of $\tan \theta = 3$:
$\sec^2 \theta = 1 + (3)^2$
$\sec^2 \theta = 1 + 9$
$\sec^2 \theta = 10$
Taking the square root of both sides:
$\sec \theta = \pm \sqrt{10}$
Since $\theta$ lies in the third quadrant, the cosine function is negative. As $\sec \theta = \frac{1}{\cos \theta}$, $\sec \theta$ must also be negative in the third quadrant.
Therefore, we choose the negative value:
$\sec \theta = -\sqrt{10}$
Now we can find $\cos \theta$ using the reciprocal relationship:
$\cos \theta = \frac{1}{\sec \theta}$
$\cos \theta = \frac{1}{-\sqrt{10}}$
$\cos \theta = -\frac{1}{\sqrt{10}}$
Next, we can find $\sin \theta$ using the Pythagorean identity:
$\sin^2 \theta + \cos^2 \theta = 1$
Substitute the value of $\cos \theta$ we just found:
$\sin^2 \theta + \left(-\frac{1}{\sqrt{10}}\right)^2 = 1$
$\sin^2 \theta + \frac{(-1)^2}{(\sqrt{10})^2} = 1$
$\sin^2 \theta + \frac{1}{10} = 1$
Subtract $\frac{1}{10}$ from both sides:
$\sin^2 \theta = 1 - \frac{1}{10}$
$\sin^2 \theta = \frac{10}{10} - \frac{1}{10}$
$\sin^2 \theta = \frac{9}{10}$
Taking the square root of both sides:
$\sin \theta = \pm \sqrt{\frac{9}{10}}$
$\sin \theta = \pm \frac{\sqrt{9}}{\sqrt{10}}$
$\sin \theta = \pm \frac{3}{\sqrt{10}}$
Since $\theta$ lies in the third quadrant, the sine function is negative.
Therefore, we choose the negative value:
$\sin \theta = -\frac{3}{\sqrt{10}}$
Alternate Method using Coordinates:
We are given $\tan \theta = 3$. We know that $\tan \theta = \frac{\text{opposite}}{\text{adjacent}}$ or $\frac{y}{x}$ in a coordinate system.
Since $\tan \theta = 3 = \frac{3}{1}$, we can consider a right triangle where the opposite side is 3 and the adjacent side is 1. The hypotenuse $r$ would be $\sqrt{3^2 + 1^2} = \sqrt{9+1} = \sqrt{10}$.
Since $\theta$ is in the third quadrant, both the x-coordinate and y-coordinate are negative. The ratio $\frac{y}{x}$ is positive, which is consistent with $\tan \theta = 3$. We can represent this by taking $y = -3$ and $x = -1$.
The distance from the origin is $r = \sqrt{x^2 + y^2} = \sqrt{(-1)^2 + (-3)^2} = \sqrt{1+9} = \sqrt{10}$.
Now, $\sin \theta = \frac{y}{r}$.
$\sin \theta = \frac{-3}{\sqrt{10}}$
Both methods yield the same result.
The value of $\sin \theta$ is $-\frac{3}{\sqrt{10}}$.
Comparing this with the given options:
(A) $\frac{1}{\sqrt{10}}$
(B) $-\frac{1}{\sqrt{10}}$
(C) $\frac{-3}{\sqrt{10}}$
(D) $\frac{3}{\sqrt{10}}$
The correct option is (C).
The final answer is (C).
Question 38. The value of tan 75° – cot 75° is equal to
(A) 2$\sqrt{3}$
(B) 2 + $\sqrt{3}$
(C) 2 - $\sqrt{3}$
(D) 1
Answer:
Solution:
We need to find the value of $\tan 75^\circ - \cot 75^\circ$.
We can calculate the values of $\tan 75^\circ$ and $\cot 75^\circ$ separately.
We know that $75^\circ = 45^\circ + 30^\circ$.
Using the tangent addition formula $\tan(A+B) = \frac{\tan A + \tan B}{1 - \tan A \tan B}$:
$\tan 75^\circ = \tan (45^\circ + 30^\circ) = \frac{\tan 45^\circ + \tan 30^\circ}{1 - \tan 45^\circ \tan 30^\circ}$
Substitute the standard values $\tan 45^\circ = 1$ and $\tan 30^\circ = \frac{1}{\sqrt{3}}$:
$\tan 75^\circ = \frac{1 + \frac{1}{\sqrt{3}}}{1 - 1 \cdot \frac{1}{\sqrt{3}}} = \frac{\frac{\sqrt{3}+1}{\sqrt{3}}}{\frac{\sqrt{3}-1}{\sqrt{3}}} = \frac{\sqrt{3}+1}{\sqrt{3}-1}$
To rationalize the denominator, multiply the numerator and denominator by the conjugate of the denominator ($\sqrt{3}+1$):
$\tan 75^\circ = \frac{\sqrt{3}+1}{\sqrt{3}-1} \times \frac{\sqrt{3}+1}{\sqrt{3}+1} = \frac{(\sqrt{3}+1)^2}{(\sqrt{3})^2 - (1)^2} = \frac{(\sqrt{3})^2 + 2(\sqrt{3})(1) + 1^2}{3 - 1} = \frac{3 + 2\sqrt{3} + 1}{2} = \frac{4 + 2\sqrt{3}}{2} = 2 + \sqrt{3}$
So, $\tan 75^\circ = 2 + \sqrt{3}$.
Now, we find $\cot 75^\circ$. We know that $\cot \theta = \frac{1}{\tan \theta}$.
$\cot 75^\circ = \frac{1}{\tan 75^\circ} = \frac{1}{2 + \sqrt{3}}$
To rationalize the denominator, multiply the numerator and denominator by the conjugate of the denominator ($2 - \sqrt{3}$):
$\cot 75^\circ = \frac{1}{2 + \sqrt{3}} \times \frac{2 - \sqrt{3}}{2 - \sqrt{3}} = \frac{2 - \sqrt{3}}{(2)^2 - (\sqrt{3})^2} = \frac{2 - \sqrt{3}}{4 - 3} = \frac{2 - \sqrt{3}}{1} = 2 - \sqrt{3}$
So, $\cot 75^\circ = 2 - \sqrt{3}$.
Now, calculate the difference:
$\tan 75^\circ - \cot 75^\circ = (2 + \sqrt{3}) - (2 - \sqrt{3})$
$\tan 75^\circ - \cot 75^\circ = 2 + \sqrt{3} - 2 + \sqrt{3}$
$\tan 75^\circ - \cot 75^\circ = (2 - 2) + (\sqrt{3} + \sqrt{3})$
$\tan 75^\circ - \cot 75^\circ = 0 + 2\sqrt{3}$
$\tan 75^\circ - \cot 75^\circ = 2\sqrt{3}$
Alternate Solution using Identities:
We can use the identity $\tan \theta - \cot \theta = \frac{\sin \theta}{\cos \theta} - \frac{\cos \theta}{\sin \theta}$.
Combine the terms by finding a common denominator:
$\tan \theta - \cot \theta = \frac{\sin^2 \theta - \cos^2 \theta}{\sin \theta \cos \theta}$
We know the double angle identity $\cos 2\theta = \cos^2 \theta - \sin^2 \theta$, so $\sin^2 \theta - \cos^2 \theta = -(\cos^2 \theta - \sin^2 \theta) = -\cos 2\theta$.
The expression becomes:
$\tan \theta - \cot \theta = \frac{-\cos 2\theta}{\sin \theta \cos \theta}$
We also know the double angle identity $\sin 2\theta = 2 \sin \theta \cos \theta$, which means $\sin \theta \cos \theta = \frac{1}{2} \sin 2\theta$.
Substitute this into the denominator:
$\tan \theta - \cot \theta = \frac{-\cos 2\theta}{\frac{1}{2} \sin 2\theta} = -2 \frac{\cos 2\theta}{\sin 2\theta} = -2 \cot 2\theta$
So, $\tan \theta - \cot \theta = -2 \cot 2\theta$.
Let $\theta = 75^\circ$. Then $2\theta = 2 \times 75^\circ = 150^\circ$.
$\tan 75^\circ - \cot 75^\circ = -2 \cot 150^\circ$
Now, we evaluate $\cot 150^\circ$. We can write $150^\circ$ as $180^\circ - 30^\circ$.
Using the identity $\cot (180^\circ - \theta) = -\cot \theta$ (since $150^\circ$ is in the second quadrant where cotangent is negative):
$\cot 150^\circ = \cot (180^\circ - 30^\circ) = -\cot 30^\circ$
We know the standard value $\cot 30^\circ = \sqrt{3}$.
So, $\cot 150^\circ = -\sqrt{3}$.
Substitute this value back into the expression for $\tan 75^\circ - \cot 75^\circ$:
$\tan 75^\circ - \cot 75^\circ = -2 (-\sqrt{3})$
$\tan 75^\circ - \cot 75^\circ = 2\sqrt{3}$
Both methods give the same result.
The value of $\tan 75^\circ - \cot 75^\circ$ is $2\sqrt{3}$.
Comparing this with the given options:
(A) 2$\sqrt{3}$
(B) 2 + $\sqrt{3}$
(C) 2 - $\sqrt{3}$
(D) 1
The calculated value matches option (A).
The final answer is (A).
Question 39. Which of the following is correct?
(A) sin 1° > sin 1
(B) sin 1° < sin 1
(C) sin 1° = sin 1
(D) sin 1° = $\frac{π}{18°}$ sin 1
[Hint: 1 radian = $\frac{180°}{π}$ = 57°30’ approx
Answer:
Solution:
The expression $\sin 1^\circ$ refers to the sine of an angle measured in degrees.
The expression $\sin 1$ refers to the sine of an angle measured in radians, specifically 1 radian.
We need to compare $\sin 1^\circ$ and $\sin (1 \text{ radian})$.
We are given the hint that 1 radian is approximately equal to $57.3^\circ$.
So, we are comparing $\sin 1^\circ$ and $\sin (57.3^\circ)$.
Consider the sine function $f(x) = \sin x$ for angles $x$ in the first quadrant, i.e., $0^\circ \le x \le 90^\circ$ (or $0 \le x \le \frac{\pi}{2}$ radians).
In the first quadrant, the sine function is strictly increasing. This means that if $x_1 < x_2$ and both $x_1, x_2$ are in the first quadrant, then $\sin x_1 < \sin x_2$.
The angles we are comparing are $1^\circ$ and $57.3^\circ$. Both of these angles are in the first quadrant.
We can clearly see that $1^\circ < 57.3^\circ$.
Since the sine function is increasing in the first quadrant and $1^\circ < 57.3^\circ$, it follows that:
$\sin 1^\circ < \sin 57.3^\circ$
Substituting back $57.3^\circ \approx 1 \text{ radian}$, we get:
$\sin 1^\circ < \sin (1 \text{ radian})$
Therefore, $\sin 1^\circ < \sin 1$.
Comparing this result with the given options:
(A) $\sin 1^\circ > \sin 1$
(B) $\sin 1^\circ < \sin 1$
(C) $\sin 1^\circ = \sin 1$
(D) $\sin 1^\circ = \frac{\pi}{18^\circ} \sin 1$
The correct option is (B).
The final answer is (B).
Question 40. If tan $α$ = $\frac{m}{m \;+\; 1}$ , tan $β$ = $\frac{1}{2m \;+\; 1}$ then $α + β$ is equal to
(A) $\frac{π}{2}$
(B) $\frac{π}{3}$
(C) $\frac{π}{6}$
(D) $\frac{π}{4}$
Answer:
Given:
$\tan \alpha = \frac{m}{m+1}$
$\tan \beta = \frac{1}{2m+1}$
To Find:
The value of $\alpha + \beta$.
Solution:
We will use the tangent addition formula for $\alpha + \beta$.
$\tan(\alpha + \beta) = \frac{\tan \alpha + \tan \beta}{1 - \tan \alpha \tan \beta}$
(Tangent addition formula)
Substitute the given values of $\tan \alpha$ and $\tan \beta$ into the formula:
$\tan(\alpha + \beta) = \frac{\frac{m}{m+1} + \frac{1}{2m+1}}{1 - \left(\frac{m}{m+1}\right) \left(\frac{1}{2m+1}\right)}$
First, simplify the numerator:
Numerator = $\frac{m}{m+1} + \frac{1}{2m+1}$
To add these fractions, find a common denominator, which is $(m+1)(2m+1)$.
Numerator = $\frac{m(2m+1)}{(m+1)(2m+1)} + \frac{1(m+1)}{(m+1)(2m+1)}$
Numerator = $\frac{m(2m+1) + (m+1)}{(m+1)(2m+1)}$
Numerator = $\frac{2m^2 + m + m + 1}{(m+1)(2m+1)}$
Numerator = $\frac{2m^2 + 2m + 1}{(m+1)(2m+1)}$
Next, simplify the denominator:
Denominator = $1 - \left(\frac{m}{m+1}\right) \left(\frac{1}{2m+1}\right)$
Denominator = $1 - \frac{m \cdot 1}{(m+1)(2m+1)}$
Denominator = $1 - \frac{m}{(m+1)(2m+1)}$
To subtract, find a common denominator, which is $(m+1)(2m+1)$.
Denominator = $\frac{(m+1)(2m+1)}{(m+1)(2m+1)} - \frac{m}{(m+1)(2m+1)}$
Denominator = $\frac{(m+1)(2m+1) - m}{(m+1)(2m+1)}$
Expand the product in the numerator:
$(m+1)(2m+1) = m(2m+1) + 1(2m+1) = 2m^2 + m + 2m + 1 = 2m^2 + 3m + 1$
Substitute this back into the numerator of the denominator:
Denominator = $\frac{2m^2 + 3m + 1 - m}{(m+1)(2m+1)}$
Denominator = $\frac{2m^2 + 2m + 1}{(m+1)(2m+1)}$
Now, substitute the simplified numerator and denominator back into the expression for $\tan(\alpha + \beta)$:
$\tan(\alpha + \beta) = \frac{\text{Numerator}}{\text{Denominator}} = \frac{\frac{2m^2 + 2m + 1}{(m+1)(2m+1)}}{\frac{2m^2 + 2m + 1}{(m+1)(2m+1)}}$
Assuming $2m^2 + 2m + 1 \neq 0$ (which is always true for real m since the discriminant is $2^2 - 4(2)(1) = 4 - 8 = -4 < 0$, so the quadratic is always positive) and $(m+1)(2m+1) \neq 0$, we can cancel the common terms:
$\tan(\alpha + \beta) = 1$
We know that $\tan \frac{\pi}{4} = 1$.
Thus, the principal value of $\alpha + \beta$ is $\frac{\pi}{4}$.
$\alpha + \beta = \frac{\pi}{4}$
Comparing this result with the given options:
(A) $\frac{\pi}{2}$
(B) $\frac{\pi}{3}$
(C) $\frac{\pi}{6}$
(D) $\frac{\pi}{4}$
The calculated value matches option (D).
The final answer is (D).
Question 41. The minimum value of 3 cos x + 4 sin x + 8 is
(A) 5
(B) 9
(C) 7
(D) 3
Answer:
To Find:
The minimum value of the expression $3 \cos x + 4 \sin x + 8$.
Solution:
We have the expression $3 \cos x + 4 \sin x + 8$.
Consider the part of the expression which is in the form $a \cos x + b \sin x$. Here, $a=3$ and $b=4$.
An expression of the form $a \cos x + b \sin x$ can be written as $R \cos(x - \alpha)$ or $R \sin(x + \alpha)$, where $R = \sqrt{a^2 + b^2}$.
The value of $R$ for our expression is:
$R = \sqrt{3^2 + 4^2}$
$R = \sqrt{9 + 16}$
$R = \sqrt{25}$
$R = 5$
The range of the expression $R \cos(x - \alpha)$ or $R \sin(x + \alpha)$ is $[-R, R]$.
So, the range of $3 \cos x + 4 \sin x$ is $[-5, 5]$.
This means that for any real value of $x$, we have:
$-5 \leq 3 \cos x + 4 \sin x \leq 5$
Now, we add 8 to all parts of this inequality to find the range of the full expression $3 \cos x + 4 \sin x + 8$:
$-5 + 8 \leq 3 \cos x + 4 \sin x + 8 \leq 5 + 8$
$3 \leq 3 \cos x + 4 \sin x + 8 \leq 13$
The minimum value of the expression is the lowest value in this range, which is 3.
The maximum value of the expression is the highest value in this range, which is 13.
The minimum value of $3 \cos x + 4 \sin x + 8$ is 3.
Comparing this with the given options:
(A) 5
(B) 9
(C) 7
(D) 3
The calculated minimum value matches option (D).
The final answer is (D).
Question 42. The value of tan 3A – tan 2A – tan A is equal to
(A) tan 3A tan 2A tan A
(B) – tan 3A tan 2A tan A
(C) tan A tan 2A – tan 2A tan 3A – tan 3A tan A
(D) None of these
Answer:
Given:
The expression $\tan 3A - \tan 2A - \tan A$.
To Find:
The value of the given expression.
Solution:
We can use the relationship between the angles $3A$, $2A$, and $A$, which is $3A = 2A + A$.
Consider the tangent of $3A$. We can write it as $\tan(2A + A)$.
Using the tangent addition formula $\tan(X+Y) = \frac{\tan X + \tan Y}{1 - \tan X \tan Y}$, let $X = 2A$ and $Y = A$.
So, $\tan 3A = \tan (2A + A) = \frac{\tan 2A + \tan A}{1 - \tan 2A \tan A}$.
Assuming $1 - \tan 2A \tan A \neq 0$, we can multiply both sides of the equation by $(1 - \tan 2A \tan A)$:
$\tan 3A (1 - \tan 2A \tan A) = \tan 2A + \tan A$
Now, distribute $\tan 3A$ on the left side of the equation:
$\tan 3A \cdot 1 - \tan 3A \cdot \tan 2A \cdot \tan A = \tan 2A + \tan A$
$\tan 3A - \tan 3A \tan 2A \tan A = \tan 2A + \tan A$
We want to find the value of $\tan 3A - \tan 2A - \tan A$. Let's rearrange the terms in the equation we obtained. Move the terms $\tan 2A$ and $\tan A$ from the right side to the left side by subtracting them:
$\tan 3A - \tan 2A - \tan A - \tan 3A \tan 2A \tan A = 0$
Now, move the product term $-\tan 3A \tan 2A \tan A$ from the left side to the right side by adding it:
$\tan 3A - \tan 2A - \tan A = \tan 3A \tan 2A \tan A$
This identity holds true for all values of A where $\tan A$, $\tan 2A$, and $\tan 3A$ are defined.
Thus, the value of the expression $\tan 3A - \tan 2A - \tan A$ is equal to $\tan 3A \tan 2A \tan A$.
Comparing this result with the given options:
(A) tan 3A tan 2A tan A
(B) – tan 3A tan 2A tan A
(C) tan A tan 2A – tan 2A tan 3A – tan 3A tan A
(D) None of these
The result matches option (A).
The final answer is (A).
Question 43. The value of sin (45° + θ) – cos (45° – θ) is
(A) 2 cos θ
(B) 2 sin θ
(C) 1
(D) 0
Answer:
To Find:
The value of the expression $\sin (45^\circ + \theta) - \cos (45^\circ - \theta)$.
Solution:
We can use the angle addition and subtraction formulas for sine and cosine:
- $\sin(A+B) = \sin A \cos B + \cos A \sin B$
- $\cos(A-B) = \cos A \cos B + \sin A \sin B$
Let's apply the formula to the first term, $\sin (45^\circ + \theta)$, with $A = 45^\circ$ and $B = \theta$:
$\sin (45^\circ + \theta) = \sin 45^\circ \cos \theta + \cos 45^\circ \sin \theta$
We know that $\sin 45^\circ = \frac{1}{\sqrt{2}}$ and $\cos 45^\circ = \frac{1}{\sqrt{2}}$.
$\sin (45^\circ + \theta) = \frac{1}{\sqrt{2}} \cos \theta + \frac{1}{\sqrt{2}} \sin \theta$
$\sin (45^\circ + \theta) = \frac{1}{\sqrt{2}} (\cos \theta + \sin \theta)$
Now, let's apply the formula to the second term, $\cos (45^\circ - \theta)$, with $A = 45^\circ$ and $B = \theta$:
$\cos (45^\circ - \theta) = \cos 45^\circ \cos \theta + \sin 45^\circ \sin \theta$}
Substitute the standard values $\cos 45^\circ = \frac{1}{\sqrt{2}}$ and $\sin 45^\circ = \frac{1}{\sqrt{2}}$:
$\cos (45^\circ - \theta) = \frac{1}{\sqrt{2}} \cos \theta + \frac{1}{\sqrt{2}} \sin \theta$
$\cos (45^\circ - \theta) = \frac{1}{\sqrt{2}} (\cos \theta + \sin \theta)$
Now, substitute these expanded forms back into the original expression:
$\sin (45^\circ + \theta) - \cos (45^\circ - \theta) = \left[ \frac{1}{\sqrt{2}} (\cos \theta + \sin \theta) \right] - \left[ \frac{1}{\sqrt{2}} (\cos \theta + \sin \theta) \right]$
Let $X = \frac{1}{\sqrt{2}} (\cos \theta + \sin \theta)$. The expression is $X - X$.
$\sin (45^\circ + \theta) - \cos (45^\circ - \theta) = 0$
Alternate Method using Complementary Angles:
We can use the identity $\cos \phi = \sin (90^\circ - \phi)$.
Let $\phi = 45^\circ - \theta$. Then $90^\circ - \phi = 90^\circ - (45^\circ - \theta) = 90^\circ - 45^\circ + \theta = 45^\circ + \theta$.
So, $\cos (45^\circ - \theta) = \sin (90^\circ - (45^\circ - \theta)) = \sin (45^\circ + \theta)$.
Substitute this into the original expression:
$\sin (45^\circ + \theta) - \cos (45^\circ - \theta) = \sin (45^\circ + \theta) - \sin (45^\circ + \theta)$
$\sin (45^\circ + \theta) - \cos (45^\circ - \theta) = 0$
Both methods yield the same result.
The value of the expression $\sin (45^\circ + \theta) - \cos (45^\circ - \theta)$ is 0.
Comparing this with the given options:
(A) 2 cos θ
(B) 2 sin θ
(C) 1
(D) 0
The calculated value matches option (D).
The final answer is (D).
Question 44. The value of cot $\left( \frac{\pi}{4} + \theta \right)$ cot $\left( \frac{\pi}{4} - \theta \right)$ is
(A) –1
(B) 0
(C) 1
(D) Not defined
Answer:
To Find:
The value of $\cot \left( \frac{\pi}{4} + \theta \right) \cot \left( \frac{\pi}{4} - \theta \right)$.
Solution:
We need to evaluate the product $\cot \left( \frac{\pi}{4} + \theta \right) \cot \left( \frac{\pi}{4} - \theta \right)$.
We can use the identity $\cot x = \frac{1}{\tan x}$.
So, the expression becomes:
$\frac{1}{\tan \left( \frac{\pi}{4} + \theta \right)} \cdot \frac{1}{\tan \left( \frac{\pi}{4} - \theta \right)} = \frac{1}{\tan \left( \frac{\pi}{4} + \theta \right) \tan \left( \frac{\pi}{4} - \theta \right)}$
Now, we use the tangent addition and subtraction formulas:
- $\tan(A+B) = \frac{\tan A + \tan B}{1 - \tan A \tan B}$
- $\tan(A-B) = \frac{\tan A - \tan B}{1 + \tan A \tan B}$
Let $A = \frac{\pi}{4}$ and $B = \theta$. We know that $\tan \frac{\pi}{4} = 1$.
Using the tangent addition formula:
$\tan \left( \frac{\pi}{4} + \theta \right) = \frac{\tan \frac{\pi}{4} + \tan \theta}{1 - \tan \frac{\pi}{4} \tan \theta} = \frac{1 + \tan \theta}{1 - 1 \cdot \tan \theta} = \frac{1 + \tan \theta}{1 - \tan \theta}$
Using the tangent subtraction formula:
$\tan \left( \frac{\pi}{4} - \theta \right) = \frac{\tan \frac{\pi}{4} - \tan \theta}{1 + \tan \frac{\pi}{4} \tan \theta} = \frac{1 - \tan \theta}{1 + 1 \cdot \tan \theta} = \frac{1 - \tan \theta}{1 + \tan \theta}$
Now, multiply these two tangent expressions:
$\tan \left( \frac{\pi}{4} + \theta \right) \tan \left( \frac{\pi}{4} - \theta \right) = \left( \frac{1 + \tan \theta}{1 - \tan \theta} \right) \cdot \left( \frac{1 - \tan \theta}{1 + \tan \theta} \right)$
Assuming $1 - \tan \theta \neq 0$ and $1 + \tan \theta \neq 0$, we can cancel the common terms:
$\tan \left( \frac{\pi}{4} + \theta \right) \tan \left( \frac{\pi}{4} - \theta \right) = 1$
Now, substitute this back into the original cotangent expression:
$\cot \left( \frac{\pi}{4} + \theta \right) \cot \left( \frac{\pi}{4} - \theta \right) = \frac{1}{\tan \left( \frac{\pi}{4} + \theta \right) \tan \left( \frac{\pi}{4} - \theta \right)} = \frac{1}{1} = 1$
The value of the expression is 1.
Comparing this with the given options:
(A) –1
(B) 0
(C) 1
(D) Not defined
The calculated value matches option (C).
The final answer is (C).
Question 45. cos 2θ cos 2φ + sin2 (θ – φ) – sin2 (θ + φ) is equal to
(A) sin 2(θ + φ)
(B) cos 2(θ + φ)
(C) sin 2(θ – φ)
(D) cos 2(θ – φ)
[Hint: Use sin2 A – sin2 B = sin (A + B) sin (A – B)]
Answer:
Given:
The expression $\cos 2\theta \cos 2\phi + \sin^2 (\theta – \phi) – \sin^2 (\theta + \phi)$.
To Find:
The value of the given expression.
Solution:
Let the given expression be $E$.
$E = \cos 2\theta \cos 2\phi + \sin^2 (\theta – \phi) – \sin^2 (\theta + \phi)$
We are given the hint to use the identity $\sin^2 A – \sin^2 B = \sin (A + B) \sin (A – B)$.
Let $A = \theta - \phi$ and $B = \theta + \phi$.
Then, $A + B = (\theta - \phi) + (\theta + \phi) = \theta - \phi + \theta + \phi = 2\theta$.
And $A - B = (\theta - \phi) - (\theta + \phi) = \theta - \phi - \theta - \phi = -2\phi$.
Applying the identity to the terms $\sin^2 (\theta – \phi) – \sin^2 (\theta + \phi)$:
$\sin^2 (\theta – \phi) – \sin^2 (\theta + \phi) = \sin \left( (\theta - \phi) + (\theta + \phi) \right) \sin \left( (\theta - \phi) - (\theta + \phi) \right)$
$\sin^2 (\theta – \phi) – \sin^2 (\theta + \phi) = \sin (2\theta) \sin (-2\phi)$
We know that $\sin (-x) = -\sin x$.
$\sin^2 (\theta – \phi) – \sin^2 (\theta + \phi) = \sin (2\theta) (-\sin (2\phi))$
$\sin^2 (\theta – \phi) – \sin^2 (\theta + \phi) = -\sin 2\theta \sin 2\phi$
Now substitute this back into the original expression $E$:
$E = \cos 2\theta \cos 2\phi + (-\sin 2\theta \sin 2\phi)$
$E = \cos 2\theta \cos 2\phi - \sin 2\theta \sin 2\phi$}
This expression is in the form of the cosine addition formula: $\cos(X+Y) = \cos X \cos Y - \sin X \sin Y$.
Let $X = 2\theta$ and $Y = 2\phi$.
So, $E = \cos (2\theta + 2\phi)$
$E = \cos (2(\theta + \phi))$
The value of the expression is $\cos (2(\theta + \phi))$.
Comparing this result with the given options:
(A) $\sin 2(\theta + \phi)$
(B) $\cos 2(\theta + \phi)$
(C) $\sin 2(\theta – \phi)$
(D) $\cos 2(\theta – \phi)$
The calculated value matches option (B).
The final answer is (B).
Question 46. The value of cos 12° + cos 84° + cos 156° + cos 132° is
(A) $\frac{1}{2}$
(B) 1
(C) $-\frac{1}{2}$
(D) $\frac{1}{8}$
Answer:
To Find:
The value of $\cos 12^\circ + \cos 84^\circ + \cos 156^\circ + \cos 132^\circ$.
Solution:
Let the given expression be $E = \cos 12^\circ + \cos 84^\circ + \cos 156^\circ + \cos 132^\circ$.
We can group the terms strategically and use the sum-to-product formula: $\cos A + \cos B = 2 \cos \frac{A+B}{2} \cos \frac{A-B}{2}$.
Let's group the terms as $(\cos 12^\circ + \cos 132^\circ) + (\cos 84^\circ + \cos 156^\circ)$.
Consider the first group: $\cos 12^\circ + \cos 132^\circ$.
Using the sum-to-product formula with $A = 12^\circ$ and $B = 132^\circ$:
$\cos 12^\circ + \cos 132^\circ = 2 \cos \left(\frac{12^\circ + 132^\circ}{2}\right) \cos \left(\frac{12^\circ - 132^\circ}{2}\right)$
$= 2 \cos \left(\frac{144^\circ}{2}\right) \cos \left(\frac{-120^\circ}{2}\right)$
$= 2 \cos 72^\circ \cos (-60^\circ)$
Since $\cos (-x) = \cos x$, we have $\cos (-60^\circ) = \cos 60^\circ = \frac{1}{2}$.
$= 2 \cos 72^\circ \left(\frac{1}{2}\right)$
$= \cos 72^\circ$
Consider the second group: $\cos 84^\circ + \cos 156^\circ$.
Using the sum-to-product formula with $A = 84^\circ$ and $B = 156^\circ$:
$\cos 84^\circ + \cos 156^\circ = 2 \cos \left(\frac{84^\circ + 156^\circ}{2}\right) \cos \left(\frac{84^\circ - 156^\circ}{2}\right)$
$= 2 \cos \left(\frac{240^\circ}{2}\right) \cos \left(\frac{-72^\circ}{2}\right)$
$= 2 \cos 120^\circ \cos (-36^\circ)$
Since $\cos (-x) = \cos x$, we have $\cos (-36^\circ) = \cos 36^\circ$.
We also know that $\cos 120^\circ = \cos (180^\circ - 60^\circ) = -\cos 60^\circ = -\frac{1}{2}$.
$= 2 \left(-\frac{1}{2}\right) \cos 36^\circ$
$= -\cos 36^\circ$
Now, add the results from the two groups to find the value of the original expression $E$:
$E = (\cos 12^\circ + \cos 132^\circ) + (\cos 84^\circ + \cos 156^\circ)$
$E = \cos 72^\circ + (-\cos 36^\circ)$
$E = \cos 72^\circ - \cos 36^\circ$
We know the standard values (or relationships) for $\cos 72^\circ$ and $\cos 36^\circ$.
$\cos 72^\circ = \sin (90^\circ - 72^\circ) = \sin 18^\circ = \frac{\sqrt{5}-1}{4}$
$\cos 36^\circ = \frac{\sqrt{5}+1}{4}$
Substitute these values into the expression for $E$:
$E = \frac{\sqrt{5}-1}{4} - \frac{\sqrt{5}+1}{4}$
$E = \frac{(\sqrt{5}-1) - (\sqrt{5}+1)}{4}$
$E = \frac{\sqrt{5} - 1 - \sqrt{5} - 1}{4}$
$E = \frac{-2}{4}$
$E = -\frac{1}{2}$
The value of the expression is $-\frac{1}{2}$.
Comparing this with the given options:
(A) $\frac{1}{2}$
(B) 1
(C) $-\frac{1}{2}$
(D) $\frac{1}{8}$
The calculated value matches option (C).
The final answer is (C).
Question 47. If tan A = $\frac{1}{2}$ , tan B = $\frac{1}{3}$ , then tan (2A + B) is equal to
(A) 1
(B) 2
(C) 3
(D) 4
Answer:
Given:
$\tan A = \frac{1}{2}$
$\tan B = \frac{1}{3}$
To Find:
The value of $\tan (2A + B)$.
Solution:
First, we need to find the value of $\tan 2A$. We use the double angle formula for tangent:
$\tan 2A = \frac{2 \tan A}{1 - \tan^2 A}$
(Double angle formula)
Substitute the given value $\tan A = \frac{1}{2}$ into the formula:
$\tan 2A = \frac{2 \left(\frac{1}{2}\right)}{1 - \left(\frac{1}{2}\right)^2}$
$\tan 2A = \frac{1}{1 - \frac{1}{4}}$
$\tan 2A = \frac{1}{\frac{4}{4} - \frac{1}{4}}$
$\tan 2A = \frac{1}{\frac{3}{4}}$
$\tan 2A = 1 \times \frac{4}{3}$
$\tan 2A = \frac{4}{3}$
Now, we need to find $\tan (2A + B)$. We use the tangent addition formula:
$\tan(X+Y) = \frac{\tan X + \tan Y}{1 - \tan X \tan Y}$
(Tangent addition formula)
Let $X = 2A$ and $Y = B$. Substitute the values $\tan 2A = \frac{4}{3}$ and $\tan B = \frac{1}{3}$ into the formula:
$\tan (2A + B) = \frac{\tan 2A + \tan B}{1 - \tan 2A \tan B}$
$\tan (2A + B) = \frac{\frac{4}{3} + \frac{1}{3}}{1 - \left(\frac{4}{3}\right) \left(\frac{1}{3}\right)}$
Calculate the numerator:
Numerator = $\frac{4}{3} + \frac{1}{3} = \frac{4+1}{3} = \frac{5}{3}$
Calculate the denominator:
Denominator = $1 - \left(\frac{4}{3}\right) \left(\frac{1}{3}\right) = 1 - \frac{4 \times 1}{3 \times 3} = 1 - \frac{4}{9}$
Denominator = $\frac{9}{9} - \frac{4}{9} = \frac{9-4}{9} = \frac{5}{9}$
Now, divide the numerator by the denominator:
$\tan (2A + B) = \frac{\frac{5}{3}}{\frac{5}{9}}$
$\tan (2A + B) = \frac{5}{3} \times \frac{9}{5}$
$\tan (2A + B) = \frac{\cancel{5}}{3} \times \frac{9}{\cancel{5}}$
$\tan (2A + B) = \frac{9}{3}$
$\tan (2A + B) = 3$
The value of $\tan (2A + B)$ is 3.
Comparing this with the given options:
(A) 1
(B) 2
(C) 3
(D) 4
The calculated value matches option (C).
The final answer is (C).
Question 48. The value of sin $\frac{π}{10}$ sin $\frac{3π}{10}$ is
(A) $\frac{1}{2}$
(B) $-\frac{1}{2}$
(C) $-\frac{1}{4}$
(D) 1
[Hint: Use $\sin 18^\circ = \frac{\sqrt{5} - 1}{4}$ and $\cos 36^\circ = \frac{\sqrt{5} + 1}{4}$]
Answer:
To Find:
The value of $\sin \frac{\pi}{10} \sin \frac{3\pi}{10}$.
Solution:
The given expression is $\sin \frac{\pi}{10} \sin \frac{3\pi}{10}$.
First, let's convert the angles from radians to degrees:
$\frac{\pi}{10} \text{ radians} = \frac{180^\circ}{10} = 18^\circ$
$\frac{3\pi}{10} \text{ radians} = \frac{3 \times 180^\circ}{10} = 3 \times 18^\circ = 54^\circ$
So the expression is equivalent to $\sin 18^\circ \sin 54^\circ$.
We know the complementary angle identity $\sin (90^\circ - x) = \cos x$.
We can write $\sin 54^\circ$ as $\sin (90^\circ - 36^\circ) = \cos 36^\circ$.
So the expression becomes $\sin 18^\circ \cos 36^\circ$.
The hint provides the values of $\sin 18^\circ$ and $\cos 36^\circ$:
$\sin 18^\circ = \frac{\sqrt{5} - 1}{4}$
(Given in hint)
$\cos 36^\circ = \frac{\sqrt{5} + 1}{4}$
(Given in hint)
Substitute these values into the expression $\sin 18^\circ \cos 36^\circ$:
$\sin 18^\circ \cos 36^\circ = \left(\frac{\sqrt{5} - 1}{4}\right) \left(\frac{\sqrt{5} + 1}{4}\right)$
This is a product of the form $(a-b)(a+b) = a^2 - b^2$ in the numerator, with $a = \sqrt{5}$ and $b = 1$.
Numerator = $(\sqrt{5} - 1)(\sqrt{5} + 1) = (\sqrt{5})^2 - 1^2 = 5 - 1 = 4$
Denominator = $4 \times 4 = 16$
So, the value of the expression is:
$\sin 18^\circ \cos 36^\circ = \frac{4}{16} = \frac{1}{4}$
The calculated value of $\sin \frac{\pi}{10} \sin \frac{3\pi}{10}$ is $\frac{1}{4}$.
Now, let's compare this result with the given options:
(A) $\frac{1}{2}$
(B) $-\frac{1}{2}$
(C) $-\frac{1}{4}$
(D) 1
The calculated value $\frac{1}{4}$ is not among the given options.
However, we are instructed to choose a correct answer from the options. Given the hint providing $\sin 18^\circ$ and $\cos 36^\circ$, it is possible that the intended question involved a calculation leading to one of the options. A common calculation involving these values is their difference:
$\cos 36^\circ - \sin 18^\circ = \frac{\sqrt{5} + 1}{4} - \frac{\sqrt{5} - 1}{4}$
$= \frac{(\sqrt{5} + 1) - (\sqrt{5} - 1)}{4}$
$= \frac{\sqrt{5} + 1 - \sqrt{5} + 1}{4}$
$= \frac{2}{4} = \frac{1}{2}$
This result, $\frac{1}{2}$, matches option (A).
While the question as written evaluates to $\frac{1}{4}$, which is not an option, the presence of the hint and the options suggests that the intended question might have been different, possibly related to the difference $\cos 36^\circ - \sin 18^\circ$ or an equivalent expression like $\sin 54^\circ - \sin 18^\circ$, which equals $\frac{1}{2}$.
Assuming there is a potential error in the question statement and one of the options is correct based on a standard calculation involving the hinted values, option (A) is the most probable intended answer.
The final answer is (A).
Question 49. The value of sin 50° – sin 70° + sin 10° is equal to
(A) 1
(B) 0
(C) $\frac{1}{2}$
(D) 2
Answer:
To Find:
The value of $\sin 50^\circ – \sin 70^\circ + \sin 10^\circ$.
Solution:
Let the given expression be $E = \sin 50^\circ – \sin 70^\circ + \sin 10^\circ$.
We can use the sum-to-product formula: $\sin A - \sin B = 2 \cos \frac{A+B}{2} \sin \frac{A-B}{2}$.
Let's group the first two terms: $\sin 50^\circ - \sin 70^\circ$.
Here, $A = 50^\circ$ and $B = 70^\circ$.
$\frac{A+B}{2} = \frac{50^\circ + 70^\circ}{2} = \frac{120^\circ}{2} = 60^\circ$
$\frac{A-B}{2} = \frac{50^\circ - 70^\circ}{2} = \frac{-20^\circ}{2} = -10^\circ$
Applying the formula:
$\sin 50^\circ - \sin 70^\circ = 2 \cos 60^\circ \sin (-10^\circ)$
We know that $\cos 60^\circ = \frac{1}{2}$ and $\sin (-x) = -\sin x$.
$\sin 50^\circ - \sin 70^\circ = 2 \left(\frac{1}{2}\right) (-\sin 10^\circ)$
$\sin 50^\circ - \sin 70^\circ = 1 \cdot (-\sin 10^\circ) = -\sin 10^\circ$
Now, substitute this back into the original expression $E$:
$E = (\sin 50^\circ – \sin 70^\circ) + \sin 10^\circ$
$E = (-\sin 10^\circ) + \sin 10^\circ$
$E = 0$
Alternate Solution:
Let the given expression be $E = \sin 50^\circ – \sin 70^\circ + \sin 10^\circ$.
We can rearrange the terms and use the sum-to-product formula: $\sin A + \sin B = 2 \sin \frac{A+B}{2} \cos \frac{A-B}{2}$.
Group the first and third terms: $(\sin 50^\circ + \sin 10^\circ) - \sin 70^\circ$.
Consider the group $\sin 50^\circ + \sin 10^\circ$. Here, $A = 50^\circ$ and $B = 10^\circ$.
$\frac{A+B}{2} = \frac{50^\circ + 10^\circ}{2} = \frac{60^\circ}{2} = 30^\circ$
$\frac{A-B}{2} = \frac{50^\circ - 10^\circ}{2} = \frac{40^\circ}{2} = 20^\circ$
Applying the formula:
$\sin 50^\circ + \sin 10^\circ = 2 \sin 30^\circ \cos 20^\circ$
We know that $\sin 30^\circ = \frac{1}{2}$.
$\sin 50^\circ + \sin 10^\circ = 2 \left(\frac{1}{2}\right) \cos 20^\circ$
$\sin 50^\circ + \sin 10^\circ = 1 \cdot \cos 20^\circ = \cos 20^\circ$
Now, substitute this back into the expression for $E$:
$E = (\sin 50^\circ + \sin 10^\circ) - \sin 70^\circ$
$E = \cos 20^\circ - \sin 70^\circ$
We know the complementary angle identity $\sin x = \cos (90^\circ - x)$.
So, $\sin 70^\circ = \sin (90^\circ - 20^\circ) = \cos 20^\circ$.
Substitute this into the expression:
$E = \cos 20^\circ - \cos 20^\circ$
$E = 0$
Both methods yield the same result.
The value of the expression is 0.
Comparing this with the given options:
(A) 1
(B) 0
(C) $\frac{1}{2}$
(D) 2
The calculated value matches option (B).
The final answer is (B).
Question 50. If sin θ + cos θ = 1, then the value of sin 2θ is equal to
(A) 1
(B) $\frac{1}{2}$
(C) 0
(D) –1
Answer:
Given:
$\sin \theta + \cos \theta = 1$
To Find:
The value of $\sin 2\theta$.
Solution:
We are given the equation $\sin \theta + \cos \theta = 1$.
We need to find the value of $\sin 2\theta$. The double angle identity for sine is $\sin 2\theta = 2 \sin \theta \cos \theta$.
To obtain the term $\sin \theta \cos \theta$, we can square both sides of the given equation.
Square both sides of the equation $\sin \theta + \cos \theta = 1$:
$(\sin \theta + \cos \theta)^2 = (1)^2$
Expand the left side using the algebraic identity $(a+b)^2 = a^2 + 2ab + b^2$:
$\sin^2 \theta + 2 \sin \theta \cos \theta + \cos^2 \theta = 1$
Rearrange the terms to group $\sin^2 \theta$ and $\cos^2 \theta$:
$(\sin^2 \theta + \cos^2 \theta) + 2 \sin \theta \cos \theta = 1$
Use the fundamental trigonometric identity $\sin^2 \theta + \cos^2 \theta = 1$:
$1 + 2 \sin \theta \cos \theta = 1$
(Using $\sin^2 \theta + \cos^2 \theta = 1$)
Subtract 1 from both sides of the equation:
$2 \sin \theta \cos \theta = 1 - 1$
$2 \sin \theta \cos \theta = 0$}
Now, use the double angle identity $\sin 2\theta = 2 \sin \theta \cos \theta$:
$\sin 2\theta = 0$
(Using $\sin 2\theta = 2 \sin \theta \cos \theta$)
The value of $\sin 2\theta$ is 0.
Comparing this result with the given options:
(A) 1
(B) $\frac{1}{2}$
(C) 0
(D) –1
The calculated value matches option (C).
The final answer is (C).
Question 51. If $α + β = \frac{π}{4}$ , then the value of (1 + tan $α$) (1 + tan $β$) is
(A) 1
(B) 2
(C) – 2
(D) Not defined
Answer:
Given:
$\alpha + \beta = \frac{\pi}{4}$
To Find:
The value of $(1 + \tan \alpha) (1 + \tan \beta)$.
Solution:
We are given that $\alpha + \beta = \frac{\pi}{4}$.
Take the tangent of both sides of the equation:
$\tan (\alpha + \beta) = \tan \left(\frac{\pi}{4}\right)$
Use the tangent addition formula: $\tan(A+B) = \frac{\tan A + \tan B}{1 - \tan A \tan B}$.
$\frac{\tan \alpha + \tan \beta}{1 - \tan \alpha \tan \beta} = \tan \left(\frac{\pi}{4}\right)$
We know that $\tan \left(\frac{\pi}{4}\right) = 1$.
So, $\frac{\tan \alpha + \tan \beta}{1 - \tan \alpha \tan \beta} = 1$
Assuming $1 - \tan \alpha \tan \beta \neq 0$, multiply both sides by $1 - \tan \alpha \tan \beta$:
$\tan \alpha + \tan \beta = 1 \cdot (1 - \tan \alpha \tan \beta)$
$\tan \alpha + \tan \beta = 1 - \tan \alpha \tan \beta$
Move the term $-\tan \alpha \tan \beta$ to the left side by adding it to both sides:
$\tan \alpha + \tan \beta + \tan \alpha \tan \beta = 1$}
Now, consider the expression we need to evaluate: $(1 + \tan \alpha) (1 + \tan \beta)$.
Expand this product:
$(1 + \tan \alpha) (1 + \tan \beta) = 1 \cdot (1 + \tan \beta) + \tan \alpha \cdot (1 + \tan \beta)$
$= 1 \cdot 1 + 1 \cdot \tan \beta + \tan \alpha \cdot 1 + \tan \alpha \cdot \tan \beta$
$= 1 + \tan \beta + \tan \alpha + \tan \alpha \tan \beta$}
Rearrange the terms:
$(1 + \tan \alpha) (1 + \tan \beta) = 1 + (\tan \alpha + \tan \beta + \tan \alpha \tan \beta)$}
From our earlier derivation, we found that $\tan \alpha + \tan \beta + \tan \alpha \tan \beta = 1$.
Substitute this result into the expanded expression:
$(1 + \tan \alpha) (1 + \tan \beta) = 1 + (1)$
$(1 + \tan \alpha) (1 + \tan \beta) = 2$}
The value of $(1 + \tan \alpha) (1 + \tan \beta)$ is 2.
Comparing this result with the given options:
(A) 1
(B) 2
(C) – 2
(D) Not defined
The calculated value matches option (B).
The final answer is (B).
Question 52. If sin θ = $\frac{-4}{5}$ and θ lies in third quadrant then the value of cos $\frac{θ}{2}$ is
(A) $\frac{1}{5}$
(B) $-\frac{1}{\sqrt{10}}$
(C) $-\frac{1}{\sqrt{5}}$
(D) $\frac{1}{\sqrt{10}}$
Answer:
Given:
$\sin \theta = -\frac{4}{5}$
$\theta$ lies in the third quadrant.
To Find:
The value of $\cos \frac{\theta}{2}$.
Solution:
We are given that $\theta$ lies in the third quadrant.
The range of angles in the third quadrant is $(n \cdot 360^\circ + 180^\circ, n \cdot 360^\circ + 270^\circ)$ for some integer $n$.
Dividing the range by 2, we get the range for $\frac{\theta}{2}$:
$\left( \frac{n \cdot 360^\circ + 180^\circ}{2}, \frac{n \cdot 360^\circ + 270^\circ}{2} \right)$
$( n \cdot 180^\circ + 90^\circ, n \cdot 180^\circ + 135^\circ )$
If we take $n=0$, the range for $\frac{\theta}{2}$ is $(90^\circ, 135^\circ)$. This range lies in the second quadrant.
In the second quadrant, the cosine function is negative.
Therefore, $\cos \frac{\theta}{2}$ will be negative.
We are given $\sin \theta = -\frac{4}{5}$. We need to find $\cos \theta$ to use the half-angle identity for cosine.
Use the Pythagorean identity: $\sin^2 \theta + \cos^2 \theta = 1$.
Substitute the value of $\sin \theta$:
$\left(-\frac{4}{5}\right)^2 + \cos^2 \theta = 1$
$\frac{16}{25} + \cos^2 \theta = 1$}
$\cos^2 \theta = 1 - \frac{16}{25}$}
$\cos^2 \theta = \frac{25}{25} - \frac{16}{25}$}
$\cos^2 \theta = \frac{9}{25}$}
Take the square root of both sides:
$\cos \theta = \pm \sqrt{\frac{9}{25}} = \pm \frac{3}{5}$
Since $\theta$ lies in the third quadrant, $\cos \theta$ is negative.
So, $\cos \theta = -\frac{3}{5}$.
Now, use the half-angle identity for cosine:
$\cos^2 \frac{\theta}{2} = \frac{1 + \cos \theta}{2}$
(Half-angle identity)
Substitute the value of $\cos \theta = -\frac{3}{5}$:
$\cos^2 \frac{\theta}{2} = \frac{1 + \left(-\frac{3}{5}\right)}{2}$
$\cos^2 \frac{\theta}{2} = \frac{1 - \frac{3}{5}}{2}$
$\cos^2 \frac{\theta}{2} = \frac{\frac{5-3}{5}}{2}$
$\cos^2 \frac{\theta}{2} = \frac{\frac{2}{5}}{2}$}
$\cos^2 \frac{\theta}{2} = \frac{2}{5} \times \frac{1}{2}$}
$\cos^2 \frac{\theta}{2} = \frac{\cancel{2}}{5} \times \frac{1}{\cancel{2}}$
$\cos^2 \frac{\theta}{2} = \frac{1}{5}$}
Take the square root of both sides to find $\cos \frac{\theta}{2}$:
$\cos \frac{\theta}{2} = \pm \sqrt{\frac{1}{5}}$
$\cos \frac{\theta}{2} = \pm \frac{1}{\sqrt{5}}$
From our earlier analysis, we determined that $\cos \frac{\theta}{2}$ must be negative because $\frac{\theta}{2}$ lies in the second quadrant.
So, $\cos \frac{\theta}{2} = -\frac{1}{\sqrt{5}}$.
The value of $\cos \frac{\theta}{2}$ is $-\frac{1}{\sqrt{5}}$.
Comparing this with the given options:
(A) $\frac{1}{5}$
(B) $-\frac{1}{\sqrt{10}}$
(C) $-\frac{1}{\sqrt{5}}$
(D) $\frac{1}{\sqrt{10}}$
The calculated value matches option (C).
The final answer is (C).
Question 53. Number of solutions of the equation tan x + sec x = 2 cosx lying in the interval [0, 2π] is
(A) 0
(B) 1
(C) 2
(D) 3
Answer:
Given:
The equation $\tan x + \sec x = 2 \cos x$.
To Find:
The number of solutions of the equation in the interval $[0, 2\pi]$.
Solution:
The given equation is $\tan x + \sec x = 2 \cos x$.
The terms $\tan x$ and $\sec x$ are defined when $\cos x \neq 0$. This means $x \neq \frac{\pi}{2}$ and $x \neq \frac{3\pi}{2}$ in the interval $[0, 2\pi]$.
Rewrite the equation in terms of $\sin x$ and $\cos x$:
$\frac{\sin x}{\cos x} + \frac{1}{\cos x} = 2 \cos x$
Combine the terms on the left side:
$\frac{\sin x + 1}{\cos x} = 2 \cos x$
Multiply both sides by $\cos x$ (valid since $\cos x \neq 0$):
$\sin x + 1 = 2 \cos^2 x$
Use the Pythagorean identity $\cos^2 x = 1 - \sin^2 x$ to express the equation in terms of $\sin x$ only:
$\sin x + 1 = 2 (1 - \sin^2 x)$
(Using $\cos^2 x = 1 - \sin^2 x$)
Distribute the 2 on the right side:
$\sin x + 1 = 2 - 2 \sin^2 x$
Rearrange the equation to form a quadratic equation in $\sin x$. Move all terms to one side:
$2 \sin^2 x + \sin x + 1 - 2 = 0$
$2 \sin^2 x + \sin x - 1 = 0$}
Let $y = \sin x$. The quadratic equation is $2y^2 + y - 1 = 0$.
We can factor this quadratic equation. We look for two numbers that multiply to $2 \times (-1) = -2$ and add up to 1 (the coefficient of the middle term). The numbers are 2 and -1.
Rewrite the middle term using these numbers:
$2y^2 + 2y - y - 1 = 0$
Factor by grouping:
$2y(y + 1) - 1(y + 1) = 0$
$(2y - 1)(y + 1) = 0$
Set each factor equal to zero to find the possible values for $y$ (which is $\sin x$):
$2y - 1 = 0 \implies 2y = 1 \implies y = \frac{1}{2}$
$y + 1 = 0 \implies y = -1$
So, we have two possible values for $\sin x$:
$\sin x = \frac{1}{2}$ or $\sin x = -1$
Now, we find the values of $x$ in the interval $[0, 2\pi]$ that satisfy these equations, keeping in mind the domain restriction $\cos x \neq 0$ (i.e., $x \neq \frac{\pi}{2}, \frac{3\pi}{2}$).
Case 1: $\sin x = \frac{1}{2}$
The general solution for $\sin x = \sin \alpha$ is $x = n\pi + (-1)^n \alpha$, where $\alpha$ is a principal value.
The principal value for $\sin x = \frac{1}{2}$ is $x = \frac{\pi}{6}$ (since $\sin \frac{\pi}{6} = \frac{1}{2}$ and $0 \le \frac{\pi}{6} \le \frac{\pi}{2}$).
For $n=0$: $x = 0 \cdot \pi + (-1)^0 \frac{\pi}{6} = \frac{\pi}{6}$.
For $n=1$: $x = 1 \cdot \pi + (-1)^1 \frac{\pi}{6} = \pi - \frac{\pi}{6} = \frac{5\pi}{6}$.
For $n=2$: $x = 2 \cdot \pi + (-1)^2 \frac{\pi}{6} = 2\pi + \frac{\pi}{6}$ (outside the interval $[0, 2\pi]$).
So, the solutions for $\sin x = \frac{1}{2}$ in $[0, 2\pi]$ are $x = \frac{\pi}{6}$ and $x = \frac{5\pi}{6}$.
Check if $\cos x \neq 0$ for these values:
$\cos \frac{\pi}{6} = \frac{\sqrt{3}}{2} \neq 0$ (valid)
$\cos \frac{5\pi}{6} = -\frac{\sqrt{3}}{2} \neq 0$ (valid)
So, we have two solutions from this case: $x = \frac{\pi}{6}$ and $x = \frac{5\pi}{6}$.
Case 2: $\sin x = -1$
The general solution for $\sin x = -1$ is $x = 2n\pi + \frac{3\pi}{2}$.
For $n=0$: $x = 2(0)\pi + \frac{3\pi}{2} = \frac{3\pi}{2}$.
For $n=1$: $x = 2(1)\pi + \frac{3\pi}{2} = 2\pi + \frac{3\pi}{2}$ (outside the interval $[0, 2\pi]$).
So, the solution for $\sin x = -1$ in $[0, 2\pi]$ is $x = \frac{3\pi}{2}$.
Check if $\cos x \neq 0$ for this value:
$\cos \frac{3\pi}{2} = 0$.
This value $x = \frac{3\pi}{2}$ makes the original equation undefined because it leads to $\cos x = 0$. Therefore, $x = \frac{3\pi}{2}$ is an extraneous solution and must be excluded.
So, we have zero solutions from this case that satisfy the domain restriction.
Combining the solutions from both cases that satisfy the domain restriction, we have $x = \frac{\pi}{6}$ and $x = \frac{5\pi}{6}$.
There are 2 solutions in the interval $[0, 2\pi]$.
Comparing this with the given options:
(A) 0
(B) 1
(C) 2
(D) 3
The number of solutions is 2, which matches option (C).
The final answer is (C).
Question 54. The value of sin $\frac{π}{18}$ + sin $\frac{π}{9}$ + sin $\frac{2π}{9}$ + sin $\frac{5π}{18}$ is given by
(A) sin $\frac{7π}{18}$ + sin $\frac{4π}{9}$
(B) 1
(C) cos $\frac{π}{6}$ + cos $\frac{3π}{7}$
(D) cos $\frac{π}{9}$ + sin $\frac{π}{9}$
Answer:
To Find:
The value of $\sin \frac{\pi}{18} + \sin \frac{\pi}{9} + \sin \frac{2\pi}{9} + \sin \frac{5\pi}{18}$.
Solution:
Let the given expression be $E = \sin \frac{\pi}{18} + \sin \frac{\pi}{9} + \sin \frac{2\pi}{9} + \sin \frac{5\pi}{18}$.
We can group the terms and use the sum-to-product formula: $\sin A + \sin B = 2 \sin \frac{A+B}{2} \cos \frac{A-B}{2}$.
Let's group the terms as $\left(\sin \frac{\pi}{18} + \sin \frac{5\pi}{18}\right) + \left(\sin \frac{\pi}{9} + \sin \frac{2\pi}{9}\right)$.
Consider the first group: $\sin \frac{\pi}{18} + \sin \frac{5\pi}{18}$.
Using the sum-to-product formula with $A = \frac{5\pi}{18}$ and $B = \frac{\pi}{18}$:
$\sin \frac{\pi}{18} + \sin \frac{5\pi}{18} = 2 \sin \left(\frac{\frac{\pi}{18} + \frac{5\pi}{18}}{2}\right) \cos \left(\frac{\frac{5\pi}{18} - \frac{\pi}{18}}{2}\right)$
$\frac{\frac{\pi}{18} + \frac{5\pi}{18}}{2} = \frac{\frac{6\pi}{18}}{2} = \frac{\frac{\pi}{3}}{2} = \frac{\pi}{6}$
$\frac{\frac{5\pi}{18} - \frac{\pi}{18}}{2} = \frac{\frac{4\pi}{18}}{2} = \frac{\frac{2\pi}{9}}{2} = \frac{\pi}{9}$}
So, $\sin \frac{\pi}{18} + \sin \frac{5\pi}{18} = 2 \sin \frac{\pi}{6} \cos \frac{\pi}{9}$.
We know that $\sin \frac{\pi}{6} = \frac{1}{2}$.
$\sin \frac{\pi}{18} + \sin \frac{5\pi}{18} = 2 \left(\frac{1}{2}\right) \cos \frac{\pi}{9} = \cos \frac{\pi}{9}$.
Consider the second group: $\sin \frac{\pi}{9} + \sin \frac{2\pi}{9}$.
Using the sum-to-product formula with $A = \frac{2\pi}{9}$ and $B = \frac{\pi}{9}$:
$\sin \frac{\pi}{9} + \sin \frac{2\pi}{9} = 2 \sin \left(\frac{\frac{\pi}{9} + \frac{2\pi}{9}}{2}\right) \cos \left(\frac{\frac{2\pi}{9} - \frac{\pi}{9}}{2}\right)$
$\frac{\frac{\pi}{9} + \frac{2\pi}{9}}{2} = \frac{\frac{3\pi}{9}}{2} = \frac{\frac{\pi}{3}}{2} = \frac{\pi}{6}$
$\frac{\frac{2\pi}{9} - \frac{\pi}{9}}{2} = \frac{\frac{\pi}{9}}{2} = \frac{\pi}{18}$}
So, $\sin \frac{\pi}{9} + \sin \frac{2\pi}{9} = 2 \sin \frac{\pi}{6} \cos \frac{\pi}{18}$.
We know that $\sin \frac{\pi}{6} = \frac{1}{2}$.
$\sin \frac{\pi}{9} + \sin \frac{2\pi}{9} = 2 \left(\frac{1}{2}\right) \cos \frac{\pi}{18} = \cos \frac{\pi}{18}$.
Add the results from the two groups to find the value of the original expression $E$:
$E = \left(\sin \frac{\pi}{18} + \sin \frac{5\pi}{18}\right) + \left(\sin \frac{\pi}{9} + \sin \frac{2\pi}{9}\right)$
$E = \cos \frac{\pi}{9} + \cos \frac{\pi}{18}$.
Now, let's evaluate the given options to see which one matches our result.
Consider option (A): $\sin \frac{7\pi}{18} + \sin \frac{4\pi}{9}$.
We can use complementary angle identities, $\sin x = \cos (\frac{\pi}{2} - x)$.
$\sin \frac{7\pi}{18} = \sin \left(\frac{\pi}{2} - \frac{\pi}{2} + \frac{7\pi}{18}\right) = \sin \left(\frac{9\pi}{18} - \frac{2\pi}{18}\right) = \sin \left(\frac{\pi}{2} - \frac{2\pi}{18}\right)$. Wait, this is not correct. The complementary angle of $\frac{7\pi}{18}$ is $\frac{\pi}{2} - \frac{7\pi}{18} = \frac{9\pi - 7\pi}{18} = \frac{2\pi}{18} = \frac{\pi}{9}$.
So, $\sin \frac{7\pi}{18} = \cos \left(\frac{\pi}{2} - \frac{7\pi}{18}\right) = \cos \frac{\pi}{9}$.
The complementary angle of $\frac{4\pi}{9}$ is $\frac{\pi}{2} - \frac{4\pi}{9} = \frac{9\pi - 8\pi}{18} = \frac{\pi}{18}$.
So, $\sin \frac{4\pi}{9} = \cos \left(\frac{\pi}{2} - \frac{4\pi}{9}\right) = \cos \frac{\pi}{18}$.
Thus, option (A) is $\sin \frac{7\pi}{18} + \sin \frac{4\pi}{9} = \cos \frac{\pi}{9} + \cos \frac{\pi}{18}$.
This matches the value we calculated for the original expression.
The value of the expression $\sin \frac{\pi}{18} + \sin \frac{\pi}{9} + \sin \frac{2\pi}{9} + \sin \frac{5\pi}{18}$ is equal to $\cos \frac{\pi}{9} + \cos \frac{\pi}{18}$, which is given by option (A).
Comparing this with the given options:
(A) $\sin \frac{7\pi}{18} + \sin \frac{4\pi}{9}$
(B) 1
(C) $\cos \frac{\pi}{6} + \cos \frac{3\pi}{7}$
(D) $\cos \frac{\pi}{9} + \sin \frac{\pi}{9}$
The result matches option (A).
The final answer is (A).
Question 55. If A lies in the second quadrant and 3 tan A + 4 = 0, then the value of 2 cot A – 5 cos A + sin A is equal to
(A) $\frac{-53}{10}$
(B) $\frac{23}{10}$
(C) $\frac{37}{10}$
(D) $\frac{7}{10}$
Answer:
Given:
$3 \tan A + 4 = 0$
A lies in the second quadrant.
To Find:
The value of $2 \cot A – 5 \cos A + \sin A$.
Solution:
From the given equation, $3 \tan A + 4 = 0$, we can find the value of $\tan A$:
$3 \tan A = -4$
$\tan A = -\frac{4}{3}$
Since $\tan A = -\frac{4}{3}$, we can find $\cot A$ using the reciprocal identity:
$\cot A = \frac{1}{\tan A} = \frac{1}{-\frac{4}{3}} = -\frac{3}{4}$
Now, we need to find $\cos A$ and $\sin A$. We know that A lies in the second quadrant.
In the second quadrant:
- $\sin A$ is positive.
- $\cos A$ is negative.
- $\tan A$ and $\cot A$ are negative (which matches our values).
We can use the identity $\sec^2 A = 1 + \tan^2 A$ to find $\sec A$ or $\cos A$.
$\sec^2 A = 1 + \left(-\frac{4}{3}\right)^2$}
$\sec^2 A = 1 + \frac{16}{9}$}
$\sec^2 A = \frac{9}{9} + \frac{16}{9}$}
$\sec^2 A = \frac{25}{9}$}
Taking the square root of both sides:
$\sec A = \pm \sqrt{\frac{25}{9}} = \pm \frac{5}{3}$
Since A is in the second quadrant, $\cos A$ is negative, so $\sec A$ is also negative.
$\sec A = -\frac{5}{3}$
Using the reciprocal identity $\cos A = \frac{1}{\sec A}$:
$\cos A = \frac{1}{-\frac{5}{3}} = -\frac{3}{5}$
Now, we find $\sin A$ using the identity $\tan A = \frac{\sin A}{\cos A}$.
$\sin A = \tan A \cdot \cos A$}
$\sin A = \left(-\frac{4}{3}\right) \cdot \left(-\frac{3}{5}\right)$}
$\sin A = \frac{(-4) \cdot (-3)}{3 \cdot 5} = \frac{12}{15}$}
$\sin A = \frac{4}{5}$
This is consistent with $\sin A$ being positive in the second quadrant.
Now, we evaluate the expression $2 \cot A – 5 \cos A + \sin A$ by substituting the values we found:
$\cot A = -\frac{3}{4}$}
$\cos A = -\frac{3}{5}$}
$\sin A = \frac{4}{5}$}
Expression = $2 \left(-\frac{3}{4}\right) - 5 \left(-\frac{3}{5}\right) + \frac{4}{5}$}
Expression = $\left(\frac{2 \times -3}{4}\right) - \left(\frac{5 \times -3}{5}\right) + \frac{4}{5}$}
Expression = $\frac{-6}{4} - \left(\frac{-15}{5}\right) + \frac{4}{5}$}
Expression = $-\frac{3}{2} - (-3) + \frac{4}{5}$}
Expression = $-\frac{3}{2} + 3 + \frac{4}{5}$}
Find a common denominator for the terms, which is 10.
$-\frac{3}{2} = -\frac{3 \times 5}{2 \times 5} = -\frac{15}{10}$}
$3 = \frac{3 \times 10}{10} = \frac{30}{10}$}
$\frac{4}{5} = \frac{4 \times 2}{5 \times 2} = \frac{8}{10}$}
Expression = $-\frac{15}{10} + \frac{30}{10} + \frac{8}{10}$}
Expression = $\frac{-15 + 30 + 8}{10}$}
Expression = $\frac{15 + 8}{10}$}
Expression = $\frac{23}{10}$}
The value of $2 \cot A – 5 \cos A + \sin A$ is $\frac{23}{10}$.
Comparing this with the given options:
(A) $\frac{-53}{10}$
(B) $\frac{23}{10}$
(C) $\frac{37}{10}$
(D) $\frac{7}{10}$
The calculated value matches option (B).
The final answer is (B).
Question 56. The value of cos2 48° – sin2 12° is
(A) $\frac{\sqrt{5} \;+\; 1}{8}$
(B) $\frac{\sqrt{5} \;-\; 1}{8}$
(C) $\frac{\sqrt{5} \;+\; 1}{5}$
(D) $\frac{\sqrt{5} \;+\; 1}{2\sqrt{2}}$
[Hint: Use cos2 A – sin2 B = cos (A + B) cos (A – B)]
Answer:
To Find:
The value of $\cos^2 48^\circ – \sin^2 12^\circ$.
Solution:
We are given the expression $\cos^2 48^\circ – \sin^2 12^\circ$.
We will use the trigonometric identity provided in the hint:
$\cos^2 A – \sin^2 B = \cos (A + B) \cos (A – B)$
In the given expression, we can identify $A = 48^\circ$ and $B = 12^\circ$.
Calculate the sum of the angles, $A+B$:
$A+B = 48^\circ + 12^\circ = 60^\circ$
Calculate the difference of the angles, $A-B$:
$A-B = 48^\circ - 12^\circ = 36^\circ$
Substitute these values into the identity $\cos^2 A – \sin^2 B = \cos (A + B) \cos (A – B)$:
$\cos^2 48^\circ – \sin^2 12^\circ = \cos (60^\circ) \cos (36^\circ)$
Now, we need to evaluate the values of $\cos 60^\circ$ and $\cos 36^\circ$.
The standard value of $\cos 60^\circ$ is:
$\cos 60^\circ = \frac{1}{2}$
(Standard value)
The value of $\cos 36^\circ$ is a known specific value:
$\cos 36^\circ = \frac{\sqrt{5}+1}{4}$
(Known value)
Substitute these values into the product $\cos 60^\circ \cos 36^\circ$:
$\cos^2 48^\circ – \sin^2 12^\circ = \left(\frac{1}{2}\right) \left(\frac{\sqrt{5}+1}{4}\right)$
Multiply the numerators and the denominators:
$\cos^2 48^\circ – \sin^2 12^\circ = \frac{1 \cdot (\sqrt{5}+1)}{2 \cdot 4}$
$\cos^2 48^\circ – \sin^2 12^\circ = \frac{\sqrt{5}+1}{8}$
The value of the expression $\cos^2 48^\circ – \sin^2 12^\circ$ is $\frac{\sqrt{5}+1}{8}$.
Comparing this result with the given options:
(A) $\frac{\sqrt{5} \;+\; 1}{8}$
(B) $\frac{\sqrt{5} \;-\; 1}{8}$
(C) $\frac{\sqrt{5} \;+\; 1}{5}$
(D) $\frac{\sqrt{5} \;+\; 1}{2\sqrt{2}}$
The calculated value matches option (A).
The final answer is (A).
Question 57. If tan $α$ = $\frac{1}{7}$ , tan $β$ = $\frac{1}{3}$ , then cos 2$α$ is equal to
(A) sin 2$β$
(B) sin 4$β$
(C) sin 3$β$
(D) cos 2$β$
Answer:
Given:
$\tan \alpha = \frac{1}{7}$
$\tan \beta = \frac{1}{3}$
To Find:
The value of $\cos 2\alpha$ and determine which of the given options it is equal to.
Solution:
First, let's calculate the value of $\cos 2\alpha$ using the given value of $\tan \alpha$.
We use the double angle identity for cosine in terms of tangent:
$\cos 2\alpha = \frac{1 - \tan^2 \alpha}{1 + \tan^2 \alpha}$
(Double angle identity)
Substitute the given value $\tan \alpha = \frac{1}{7}$ into the formula:
$\cos 2\alpha = \frac{1 - \left(\frac{1}{7}\right)^2}{1 + \left(\frac{1}{7}\right)^2}$
$\cos 2\alpha = \frac{1 - \frac{1}{49}}{1 + \frac{1}{49}}$
Simplify the numerator and the denominator:
Numerator $= 1 - \frac{1}{49} = \frac{49}{49} - \frac{1}{49} = \frac{49 - 1}{49} = \frac{48}{49}$
Denominator $= 1 + \frac{1}{49} = \frac{49}{49} + \frac{1}{49} = \frac{49 + 1}{49} = \frac{50}{49}$
So, $\cos 2\alpha = \frac{\frac{48}{49}}{\frac{50}{49}}$
$\cos 2\alpha = \frac{48}{49} \times \frac{49}{50}$
$\cos 2\alpha = \frac{48}{50}$
Simplify the fraction by dividing the numerator and denominator by their greatest common divisor, which is 2:
$\cos 2\alpha = \frac{48 \div 2}{50 \div 2} = \frac{24}{25}$}
So, the value of $\cos 2\alpha$ is $\frac{24}{25}$.
Now, let's evaluate the given options using the value $\tan \beta = \frac{1}{3}$.
We will calculate $\sin 2\beta$ and $\cos 2\beta$ first, as they are needed for options (A), (B), and (D).
Using the double angle identity for sine in terms of tangent:
$\sin 2\beta = \frac{2 \tan \beta}{1 + \tan^2 \beta}$
Substitute $\tan \beta = \frac{1}{3}$:
$\sin 2\beta = \frac{2 \left(\frac{1}{3}\right)}{1 + \left(\frac{1}{3}\right)^2} = \frac{\frac{2}{3}}{1 + \frac{1}{9}} = \frac{\frac{2}{3}}{\frac{9+1}{9}} = \frac{\frac{2}{3}}{\frac{10}{9}}$
$\sin 2\beta = \frac{2}{3} \times \frac{9}{10} = \frac{18}{30} = \frac{3}{5}$
Using the double angle identity for cosine in terms of tangent:
$\cos 2\beta = \frac{1 - \tan^2 \beta}{1 + \tan^2 \beta}$
Substitute $\tan \beta = \frac{1}{3}$:
$\cos 2\beta = \frac{1 - \left(\frac{1}{3}\right)^2}{1 + \left(\frac{1}{3}\right)^2} = \frac{1 - \frac{1}{9}}{1 + \frac{1}{9}} = \frac{\frac{9-1}{9}}{\frac{9+1}{9}} = \frac{\frac{8}{9}}{\frac{10}{9}}$
$\cos 2\beta = \frac{8}{9} \times \frac{9}{10} = \frac{8}{10} = \frac{4}{5}$
Now, check the options:
(A) $\sin 2\beta = \frac{3}{5}$. This is not equal to $\frac{24}{25}$.
(D) $\cos 2\beta = \frac{4}{5}$. This is not equal to $\frac{24}{25}$.
(B) $\sin 4\beta$. We use the double angle identity $\sin 2x = 2 \sin x \cos x$, with $x=2\beta$.
$\sin 4\beta = \sin (2 \cdot 2\beta) = 2 \sin 2\beta \cos 2\beta$
Substitute the values we found for $\sin 2\beta$ and $\cos 2\beta$:
$\sin 4\beta = 2 \left(\frac{3}{5}\right) \left(\frac{4}{5}\right)$
$\sin 4\beta = 2 \times \frac{3 \times 4}{5 \times 5}$}
$\sin 4\beta = 2 \times \frac{12}{25}$}
$\sin 4\beta = \frac{24}{25}$}
This value matches the value of $\cos 2\alpha$ we calculated.
We do not need to check option (C) since option (B) matches the value.
Thus, $\cos 2\alpha = \frac{24}{25}$ and $\sin 4\beta = \frac{24}{25}$.
Therefore, $\cos 2\alpha = \sin 4\beta$.}
Comparing this with the given options:
(A) $\sin 2\beta$
(B) $\sin 4\beta$
(C) $\sin 3\beta$
(D) $\cos 2\beta$
The result matches option (B).
The final answer is (B).
Question 58. If tan θ = $\frac{a}{b}$, then b cos 2θ + a sin 2θ is equal to
(A) a
(B) b
(C) $\frac{a}{b}$
(D) None
Answer:
Given:
$\tan \theta = \frac{a}{b}$
To Find:
The value of $b \cos 2\theta + a \sin 2\theta$.
Solution:
We are given $\tan \theta = \frac{a}{b}$.
We need to evaluate the expression $b \cos 2\theta + a \sin 2\theta$.
We can use the double angle identities for $\cos 2\theta$ and $\sin 2\theta$ in terms of $\tan \theta$:
$\cos 2\theta = \frac{1 - \tan^2 \theta}{1 + \tan^2 \theta}$
$\sin 2\theta = \frac{2 \tan \theta}{1 + \tan^2 \theta}$
Substitute the given value $\tan \theta = \frac{a}{b}$ into these identities:
For $\cos 2\theta$:
$\cos 2\theta = \frac{1 - \left(\frac{a}{b}\right)^2}{1 + \left(\frac{a}{b}\right)^2} = \frac{1 - \frac{a^2}{b^2}}{1 + \frac{a^2}{b^2}}$
Simplify the complex fraction by finding a common denominator ($b^2$) in the numerator and denominator:
$\cos 2\theta = \frac{\frac{b^2}{b^2} - \frac{a^2}{b^2}}{\frac{b^2}{b^2} + \frac{a^2}{b^2}} = \frac{\frac{b^2 - a^2}{b^2}}{\frac{b^2 + a^2}{b^2}}$
$\cos 2\theta = \frac{b^2 - a^2}{b^2} \times \frac{b^2}{b^2 + a^2} = \frac{b^2 - a^2}{b^2 + a^2}$
For $\sin 2\theta$:
$\sin 2\theta = \frac{2 \left(\frac{a}{b}\right)}{1 + \left(\frac{a}{b}\right)^2} = \frac{\frac{2a}{b}}{1 + \frac{a^2}{b^2}}$
Simplify the complex fraction:
$\sin 2\theta = \frac{\frac{2a}{b}}{\frac{b^2 + a^2}{b^2}} = \frac{2a}{b} \times \frac{b^2}{b^2 + a^2} = \frac{2ab}{b^2 + a^2}$
Now, substitute these expressions for $\cos 2\theta$ and $\sin 2\theta$ into the given expression $b \cos 2\theta + a \sin 2\theta$:
$b \left(\frac{b^2 - a^2}{b^2 + a^2}\right) + a \left(\frac{2ab}{b^2 + a^2}\right)$
Multiply the terms:
$\frac{b(b^2 - a^2)}{b^2 + a^2} + \frac{a(2ab)}{b^2 + a^2}$
Since the denominators are the same, combine the numerators:
$\frac{b(b^2 - a^2) + a(2ab)}{b^2 + a^2}$
Distribute in the numerator:
$\frac{b^3 - ba^2 + 2a^2b}{b^2 + a^2}$
Combine like terms in the numerator ($-ba^2 + 2a^2b = a^2b$):
$\frac{b^3 + a^2b}{b^2 + a^2}$}
Factor out $b$ from the numerator:
$\frac{b(b^2 + a^2)}{b^2 + a^2}$}
Assuming $b^2 + a^2 \neq 0$ (which is true if $\tan \theta = a/b$ is defined and $a,b$ are not both zero), we can cancel the term $(b^2 + a^2)$:
$b$}
The value of $b \cos 2\theta + a \sin 2\theta$ is $b$.
Comparing this with the given options:
(A) a
(B) b
(C) $\frac{a}{b}$
(D) None
The calculated value matches option (B).
The final answer is (B).
Question 59. If for real values of x, cos θ = x + $\frac{1}{x}$, then
(A) θ is an acute angle
(B) θ is right angle
(C) θ is an obtuse angle
(D) No value of θ is possible
Answer:
Given:
The equation $\cos \theta = x + \frac{1}{x}$ for real values of $x$.
To Determine:
The possible value(s) of $\theta$ or if no such value exists.
Solution:
We are given the equation $\cos \theta = x + \frac{1}{x}$, where $x$ is a real number.
First, let's consider the possible values of $\cos \theta$. For any real angle $\theta$, the value of $\cos \theta$ is always between -1 and 1, inclusive.
So, the range of $\cos \theta$ is $[-1, 1]$.
$-1 \leq \cos \theta \leq 1$
(Range of cosine function)
Next, let's consider the expression on the right side of the equation, $x + \frac{1}{x}$, for real values of $x$. Note that $x$ cannot be 0, as $\frac{1}{x}$ would be undefined.
We can analyze the range of the expression $x + \frac{1}{x}$ by considering two cases for real $x \neq 0$: $x > 0$ and $x < 0$.
Case 1: $x > 0$.
Since $x$ is real and positive, $\frac{1}{x}$ is also real and positive.
We can apply the Arithmetic Mean - Geometric Mean (A.M. - G.M.) inequality for the two positive numbers $x$ and $\frac{1}{x}$. The inequality states that for non-negative numbers $a$ and $b$, $\frac{a+b}{2} \ge \sqrt{ab}$, with equality holding if and only if $a=b$.
Applying the inequality with $a=x$ and $b=\frac{1}{x}$:
$\frac{x + \frac{1}{x}}{2} \ge \sqrt{x \cdot \frac{1}{x}}$
$\frac{x + \frac{1}{x}}{2} \ge \sqrt{1}$
$\frac{x + \frac{1}{x}}{2} \ge 1$
Multiply both sides by 2:
$x + \frac{1}{x} \ge 2$
The equality holds when $x = \frac{1}{x}$, which means $x^2 = 1$. Since we are in the case $x > 0$, this occurs when $x=1$. For $x=1$, $x + \frac{1}{x} = 1 + \frac{1}{1} = 2$.
So, for real $x > 0$, the minimum value of $x + \frac{1}{x}$ is 2, and the range is $[2, \infty)$.
Case 2: $x < 0$.
Since $x$ is real and negative, let $x = -y$, where $y$ is real and positive ($y = -x > 0$).
The expression $x + \frac{1}{x}$ becomes: $(-y) + \frac{1}{(-y)} = -y - \frac{1}{y} = -\left(y + \frac{1}{y}\right)$
From Case 1, for $y > 0$, we know that $y + \frac{1}{y} \ge 2$.
Multiplying the inequality $y + \frac{1}{y} \ge 2$ by -1 and reversing the inequality sign, we get:
$-\left(y + \frac{1}{y}\right) \le -2$}
So, for real $x < 0$, the value of $x + \frac{1}{x}$ is always less than or equal to -2.
$x + \frac{1}{x} \le -2$
The equality holds when $y=1$, which means $x=-1$. For $x=-1$, $x + \frac{1}{x} = -1 + \frac{1}{-1} = -1 - 1 = -2$.
So, for real $x < 0$, the maximum value of $x + \frac{1}{x}$ is -2, and the range is $(-\infty, -2]$.
Combining both cases (for real $x \neq 0$), the range of the expression $x + \frac{1}{x}$ is $(-\infty, -2] \cup [2, \infty)$.
The equation $\cos \theta = x + \frac{1}{x}$ implies that the value of $\cos \theta$ must belong to the set $(-\infty, -2] \cup [2, \infty)$.
However, we know that the actual range of $\cos \theta$ for any real angle $\theta$ is $[-1, 1]$.
For the equation $\cos \theta = x + \frac{1}{x}$ to have a solution for $\theta$ with real $x$, the value $x + \frac{1}{x}$ must be a possible value for $\cos \theta$.
This means the value must be in the intersection of the two ranges: $[-1, 1]$ and $(-\infty, -2] \cup [2, \infty)$.
The intersection of these two sets is empty: $[-1, 1] \cap ((-\infty, -2] \cup [2, \infty)) = \emptyset$.
There is no value that is simultaneously in both ranges.
Therefore, there is no real value of $\theta$ for which $\cos \theta$ can be equal to $x + \frac{1}{x}$ for some real value of $x$.
Thus, no value of $\theta$ is possible.
Comparing this conclusion with the given options:
(A) θ is an acute angle (This implies $\cos \theta > 0$) - Not always possible.
(B) θ is right angle (This implies $\cos \theta = 0$) - Not possible as 0 is not in the range of $x + 1/x$.
(C) θ is an obtuse angle (This implies $\cos \theta < 0$) - Not always possible.
(D) No value of θ is possible
The conclusion matches option (D).
The final answer is (D).
Question 60 to 67 (Fill in the Blanks)
Fill in the blanks in Exercises 60 to 67:
Question 60. The value of $\frac{sin \;50°}{sin \;130°}$ is _________.
Answer:
We are asked to find the value of the expression $\frac{\sin 50^\circ}{\sin 130^\circ}$.
We know the trigonometric identity: $\sin (180^\circ - \theta) = \sin \theta$.
Using this identity, we can rewrite the denominator $\sin 130^\circ$ as:
$\sin 130^\circ = \sin (180^\circ - 50^\circ)$
$\sin 130^\circ = \sin 50^\circ$
[Using $\sin (180^\circ - \theta) = \sin \theta$]
Now, substitute this value back into the given expression:
$\frac{\sin 50^\circ}{\sin 130^\circ} = \frac{\sin 50^\circ}{\sin 50^\circ}$
Since the numerator and the denominator are the same (and not zero), we can simplify the fraction.
$\frac{\sin 50^\circ}{\sin 50^\circ} = 1$
Thus, the value of $\frac{\sin 50^\circ}{\sin 130^\circ}$ is 1.
The final answer is 1.
Question 61. If k = sin $\left( \frac{\pi}{18} \right)$ sin $\left( \frac{5\pi}{18} \right)$ sin $\left( \frac{7\pi}{18} \right)$, then the numerical value of k is ______.
Answer:
We are given the expression $k = \sin \left( \frac{\pi}{18} \right) \sin \left( \frac{5\pi}{18} \right) \sin \left( \frac{7\pi}{18} \right)$.
First, let's convert the angles from radians to degrees.
$\frac{\pi}{18} \text{ radians} = \frac{180^\circ}{18} = 10^\circ$
$\frac{5\pi}{18} \text{ radians} = \frac{5 \times 180^\circ}{18} = 5 \times 10^\circ = 50^\circ$
$\frac{7\pi}{18} \text{ radians} = \frac{7 \times 180^\circ}{18} = 7 \times 10^\circ = 70^\circ$
So, the expression becomes $k = \sin 10^\circ \sin 50^\circ \sin 70^\circ$.
We observe that the angles are $10^\circ$, $60^\circ - 10^\circ = 50^\circ$, and $60^\circ + 10^\circ = 70^\circ$. This matches the form $\sin \theta \sin(60^\circ - \theta) \sin(60^\circ + \theta)$ with $\theta = 10^\circ$.
We use the trigonometric identity: $\sin \theta \sin(60^\circ - \theta) \sin(60^\circ + \theta) = \frac{1}{4} \sin(3\theta)$.
Applying the identity with $\theta = 10^\circ$:
$k = \sin 10^\circ \sin (60^\circ - 10^\circ) \sin (60^\circ + 10^\circ)$
$k = \frac{1}{4} \sin(3 \times 10^\circ)$
$k = \frac{1}{4} \sin 30^\circ$
We know the value of $\sin 30^\circ$, which is $\frac{1}{2}$.
$k = \frac{1}{4} \times \frac{1}{2}$
$k = \frac{1}{8}$
Thus, the numerical value of k is $\frac{1}{8}$.
The final answer is $\frac{1}{8}$.
Question 62. If tan A = $\frac{1 \;−\; cos \;B}{sin \;B}$, then tan 2A = ________.
Answer:
We are given the equation $\tan A = \frac{1 - \cos B}{\sin B}$.
We can use the half-angle identities for $\cos B$ and $\sin B$.
Recall that $1 - \cos \theta = 2 \sin^2 \left( \frac{\theta}{2} \right)$ and $\sin \theta = 2 \sin \left( \frac{\theta}{2} \right) \cos \left( \frac{\theta}{2} \right)$.
Applying these identities to the expression for $\tan A$ with $\theta = B$:
$\tan A = \frac{2 \sin^2 \left( \frac{B}{2} \right)}{2 \sin \left( \frac{B}{2} \right) \cos \left( \frac{B}{2} \right)}$
Assuming $\sin \left( \frac{B}{2} \right) \neq 0$, we can cancel one term of $\sin \left( \frac{B}{2} \right)$ from the numerator and denominator:
$\tan A = \frac{\sin \left( \frac{B}{2} \right)}{\cos \left( \frac{B}{2} \right)}$
$\tan A = \tan \left( \frac{B}{2} \right)$
Now, we need to find $\tan 2A$. We use the double-angle formula for tangent:
$\tan 2A = \frac{2 \tan A}{1 - \tan^2 A}$
Substitute $\tan A = \tan \left( \frac{B}{2} \right)$ into the formula for $\tan 2A$:
$\tan 2A = \frac{2 \tan \left( \frac{B}{2} \right)}{1 - \tan^2 \left( \frac{B}{2} \right)}$
Recall the double-angle formula for tangent in reverse form: $\tan \theta = \frac{2 \tan(\theta/2)}{1 - \tan^2(\theta/2)}$.
Comparing the expression $\frac{2 \tan \left( \frac{B}{2} \right)}{1 - \tan^2 \left( \frac{B}{2} \right)}$ with the formula, we can see that it is equal to $\tan \left( 2 \times \frac{B}{2} \right)$.
$\tan 2A = \tan \left( 2 \times \frac{B}{2} \right) = \tan B$
Thus, if $\tan A = \frac{1 - \cos B}{\sin B}$, then $\tan 2A = \tan B$.
The final answer is tan B.
Question 63. If sin x + cos x = a, then
(i) sin6 x + cos6 x = _______
(ii) | sin x – cos x | = _______.
Answer:
Given:
$\sin x + \cos x = a$
To Find:
(i) $\sin^6 x + \cos^6 x$
(ii) $|\sin x - \cos x|$
Solution:
We are given $\sin x + \cos x = a$.
Squaring both sides of the given equation:
$(\sin x + \cos x)^2 = a^2$
Expanding the left side:
$\sin^2 x + \cos^2 x + 2 \sin x \cos x = a^2$
Using the identity $\sin^2 x + \cos^2 x = 1$:
$1 + 2 \sin x \cos x = a^2$
[Using $\sin^2 x + \cos^2 x = 1$]
Solving for $2 \sin x \cos x$:
$2 \sin x \cos x = a^2 - 1$
Dividing by 2:
$\sin x \cos x = \frac{a^2 - 1}{2}$
(i) Finding $\sin^6 x + \cos^6 x$
We can write $\sin^6 x + \cos^6 x$ as $(\sin^2 x)^3 + (\cos^2 x)^3$.
Using the algebraic identity $p^3 + q^3 = (p+q)^3 - 3pq(p+q)$, with $p = \sin^2 x$ and $q = \cos^2 x$:
$\sin^6 x + \cos^6 x = (\sin^2 x + \cos^2 x)^3 - 3 (\sin^2 x)(\cos^2 x) (\sin^2 x + \cos^2 x)$
Using the identity $\sin^2 x + \cos^2 x = 1$ and $(\sin x \cos x)^2 = \sin^2 x \cos^2 x$:
$\sin^6 x + \cos^6 x = (1)^3 - 3 (\sin x \cos x)^2 (1)$
$\sin^6 x + \cos^6 x = 1 - 3 (\sin x \cos x)^2$
Substitute the value of $\sin x \cos x = \frac{a^2 - 1}{2}$ into this expression:
$\sin^6 x + \cos^6 x = 1 - 3 \left( \frac{a^2 - 1}{2} \right)^2$
$\sin^6 x + \cos^6 x = 1 - 3 \left( \frac{(a^2 - 1)^2}{4} \right)$
$\sin^6 x + \cos^6 x = 1 - \frac{3(a^2 - 1)^2}{4}$
Thus, $\sin^6 x + \cos^6 x = \mathbf{1 - \frac{3}{4}(a^2-1)^2}$.
(ii) Finding $|\sin x - \cos x|$
Let $y = \sin x - \cos x$. We want to find $|y|$.
Consider the square of $y$:
$y^2 = (\sin x - \cos x)^2$
Expanding the right side using $(p-q)^2 = p^2 + q^2 - 2pq$:
$y^2 = \sin^2 x + \cos^2 x - 2 \sin x \cos x$
Using the identity $\sin^2 x + \cos^2 x = 1$:
$y^2 = 1 - 2 \sin x \cos x$
[Using $\sin^2 x + \cos^2 x = 1$]
From the first part, we found $2 \sin x \cos x = a^2 - 1$. Substitute this value:
$y^2 = 1 - (a^2 - 1)$
$y^2 = 1 - a^2 + 1$
$y^2 = 2 - a^2$
Since we need the absolute value, we take the square root of $y^2$:
$|y| = \sqrt{y^2}$
$|\sin x - \cos x| = \sqrt{2 - a^2}$
The value of $|\sin x - \cos x|$ is $\mathbf{\sqrt{2 - a^2}}$.
Question 64. In a triangle ABC with ∠C = 90° the equation whose roots are tan A and tan B is _______.
[Hint: A + B = 90° ⇒ tan A tan B = 1 and tan A + tan B = $\frac{2}{sin \;2A}$]
Answer:
Given:
Triangle ABC with $\angle C = 90^\circ$.
To Find:
The equation whose roots are $\tan A$ and $\tan B$.
Solution:
In any triangle, the sum of the interior angles is $180^\circ$.
$\angle A + \angle B + \angle C = 180^\circ$
Given that $\angle C = 90^\circ$:
$\angle A + \angle B + 90^\circ = 180^\circ$
(Given $\angle C = 90^\circ$)
Subtracting $90^\circ$ from both sides:
$\angle A + \angle B = 90^\circ$
Let the roots of the required quadratic equation be $r_1 = \tan A$ and $r_2 = \tan B$.
A quadratic equation with roots $r_1$ and $r_2$ is given by $t^2 - (r_1 + r_2)t + r_1 r_2 = 0$, where $t$ is the variable.
We need to find the sum of the roots, $r_1 + r_2 = \tan A + \tan B$, and the product of the roots, $r_1 r_2 = \tan A \cdot \tan B$.
Product of Roots:
Product $= \tan A \cdot \tan B$
Since $A + B = 90^\circ$, we have $B = 90^\circ - A$.
Product $= \tan A \cdot \tan (90^\circ - A)$
Using the trigonometric identity $\tan (90^\circ - \theta) = \cot \theta$:
Product $= \tan A \cdot \cot A$
[Using $\tan (90^\circ - \theta) = \cot \theta$]
Using the identity $\tan \theta \cdot \cot \theta = 1$:
Product $= 1$
[Using $\tan \theta \cot \theta = 1$]
Sum of Roots:
Sum $= \tan A + \tan B$
Since $B = 90^\circ - A$, we have $\tan B = \cot A$.
Sum $= \tan A + \cot A$
Rewrite in terms of sine and cosine:
Sum $= \frac{\sin A}{\cos A} + \frac{\cos A}{\sin A}$
Find a common denominator and add the fractions:
Sum $= \frac{\sin^2 A + \cos^2 A}{\sin A \cos A}$
Using the identity $\sin^2 \theta + \cos^2 \theta = 1$:
Sum $= \frac{1}{\sin A \cos A}$
[Using $\sin^2 A + \cos^2 A = 1$]
Multiply the numerator and denominator by 2:
Sum $= \frac{2}{2 \sin A \cos A}$
Using the double angle identity $\sin 2\theta = 2 \sin \theta \cos \theta$:
Sum $= \frac{2}{\sin 2A}$
[Using $\sin 2A = 2 \sin A \cos A$]
Forming the Equation:
The required quadratic equation with roots $\tan A$ and $\tan B$ is:
$t^2 - (\text{Sum of roots})t + (\text{Product of roots}) = 0$
Substitute the values for the sum ($\frac{2}{\sin 2A}$) and product (1) of the roots:
$t^2 - \left(\frac{2}{\sin 2A}\right)t + 1 = 0$
Thus, the equation whose roots are $\tan A$ and $\tan B$ is $t^2 - \left(\frac{2}{\sin 2A}\right)t + 1 = 0$.
Note that since $A+B=90^\circ$, we have $\sin 2A = \sin (2(90^\circ - B)) = \sin (180^\circ - 2B) = \sin 2B$. So the sum can also be written as $\frac{2}{\sin 2B}$.
Question 65. 3 (sin x – cos x)4 + 6 (sin x + cos x)2 + 4 (sin6 x + cos6 x) = _______.
Answer:
We are asked to simplify the expression: $3 (\sin x – \cos x)^4 + 6 (\sin x + \cos x)^2 + 4 (\sin^6 x + \cos^6 x)$.
Let's simplify each part of the expression separately.
Consider the first term: $(\sin x - \cos x)^4$.
We know that $(\sin x - \cos x)^2 = \sin^2 x + \cos^2 x - 2 \sin x \cos x$.
Using the identity $\sin^2 x + \cos^2 x = 1$ and the double angle formula $\sin 2x = 2 \sin x \cos x$:
$(\sin x - \cos x)^2 = 1 - \sin 2x$
[Using $\sin^2 x + \cos^2 x = 1$ and $2 \sin x \cos x = \sin 2x$]
Now, square this result:
$(\sin x - \cos x)^4 = ((\sin x - \cos x)^2)^2 = (1 - \sin 2x)^2$
Expanding the square:
$(1 - \sin 2x)^2 = 1^2 - 2(1)(\sin 2x) + (\sin 2x)^2 = 1 - 2 \sin 2x + \sin^2 2x$
So, $3 (\sin x - \cos x)^4 = 3 (1 - 2 \sin 2x + \sin^2 2x) = 3 - 6 \sin 2x + 3 \sin^2 2x$.
Consider the second term: $(\sin x + \cos x)^2$.
$(\sin x + \cos x)^2 = \sin^2 x + \cos^2 x + 2 \sin x \cos x$.
Using the identity $\sin^2 x + \cos^2 x = 1$ and the double angle formula $\sin 2x = 2 \sin x \cos x$:
$(\sin x + \cos x)^2 = 1 + \sin 2x$
[Using $\sin^2 x + \cos^2 x = 1$ and $2 \sin x \cos x = \sin 2x$]
So, $6 (\sin x + \cos x)^2 = 6 (1 + \sin 2x) = 6 + 6 \sin 2x$.
Consider the third term: $\sin^6 x + \cos^6 x$.
We can write this as $(\sin^2 x)^3 + (\cos^2 x)^3$.
Using the algebraic identity $a^3 + b^3 = (a+b)(a^2 - ab + b^2)$, with $a = \sin^2 x$ and $b = \cos^2 x$:
$\sin^6 x + \cos^6 x = (\sin^2 x + \cos^2 x)((\sin^2 x)^2 - \sin^2 x \cos^2 x + (\cos^2 x)^2)$
Using the identity $\sin^2 x + \cos^2 x = 1$ and $\sin^4 x + \cos^4 x = (\sin^2 x + \cos^2 x)^2 - 2 \sin^2 x \cos^2 x = 1^2 - 2 \sin^2 x \cos^2 x = 1 - 2 \sin^2 x \cos^2 x$:
$\sin^6 x + \cos^6 x = (1)(1 - 2 \sin^2 x \cos^2 x - \sin^2 x \cos^2 x)$
$\sin^6 x + \cos^6 x = 1 - 3 \sin^2 x \cos^2 x$
Using the identity $\sin x \cos x = \frac{1}{2} \sin 2x$, we have $\sin^2 x \cos^2 x = \left(\frac{1}{2} \sin 2x\right)^2 = \frac{1}{4} \sin^2 2x$:
$\sin^6 x + \cos^6 x = 1 - 3 \left(\frac{1}{4} \sin^2 2x\right) = 1 - \frac{3}{4} \sin^2 2x$.
So, $4 (\sin^6 x + \cos^6 x) = 4 \left(1 - \frac{3}{4} \sin^2 2x\right) = 4 - 3 \sin^2 2x$.
Now, substitute the simplified terms back into the original expression:
$3 (\sin x – \cos x)^4 + 6 (\sin x + \cos x)^2 + 4 (\sin^6 x + \cos^6 x)$
$= (3 - 6 \sin 2x + 3 \sin^2 2x) + (6 + 6 \sin 2x) + (4 - 3 \sin^2 2x)$
Combine like terms:
$= (3 + 6 + 4) + (-6 \sin 2x + 6 \sin 2x) + (3 \sin^2 2x - 3 \sin^2 2x)$
$= 13 + 0 + 0$
$= 13$
The value of the expression is 13.
The final answer is 13.
Question 66. Given x > 0, the values of f(x) = – 3 cos $\sqrt{3 + x + x^2}$ lie in the interval _______.
Answer:
We are given the function $f(x) = -3 \cos \sqrt{3 + x + x^2}$ for $x > 0$.
To find the interval in which the values of $f(x)$ lie, we first need to determine the range of the argument of the cosine function, which is $\sqrt{3 + x + x^2}$.
Let's analyze the expression inside the square root: $g(x) = x^2 + x + 3$. This is a quadratic function.
The vertex of the parabola $g(x) = ax^2 + bx + c$ is at $x = -\frac{b}{2a}$.
For $g(x) = x^2 + x + 3$, $a=1$, $b=1$, $c=3$. The vertex is at $x = -\frac{1}{2(1)} = -\frac{1}{2}$.
The minimum value of the quadratic occurs at the vertex:
$g\left(-\frac{1}{2}\right) = \left(-\frac{1}{2}\right)^2 + \left(-\frac{1}{2}\right) + 3 = \frac{1}{4} - \frac{1}{2} + 3 = \frac{1 - 2 + 12}{4} = \frac{11}{4}$.
So, the minimum value of $x^2 + x + 3$ is $\frac{11}{4}$, which occurs at $x = -\frac{1}{2}$.
We are given the domain $x > 0$. Since the vertex $x = -\frac{1}{2}$ is not in this domain, and the parabola opens upwards (as the coefficient of $x^2$ is positive), the function $g(x) = x^2 + x + 3$ is increasing for $x > 0$.
As $x \to 0^+$ (from the right), $g(x) \to 0^2 + 0 + 3 = 3$.
As $x \to \infty$, $g(x) \to \infty$.
Therefore, for $x > 0$, the range of $g(x) = x^2 + x + 3$ is $(3, \infty)$.
Now consider the square root, $\sqrt{g(x)} = \sqrt{x^2 + x + 3}$.
Since the range of $g(x)$ for $x > 0$ is $(3, \infty)$, the range of $\sqrt{g(x)}$ is $(\sqrt{3}, \sqrt{\infty})$, which is $(\sqrt{3}, \infty)$.
Let $\theta = \sqrt{3 + x + x^2}$. For $x > 0$, $\theta \in (\sqrt{3}, \infty)$.
Next, consider the cosine function, $\cos(\theta)$, where $\theta \in (\sqrt{3}, \infty)$.
The value of $\sqrt{3}$ is approximately $1.732$. Since $\sqrt{3} > \frac{\pi}{2}$ (because $\sqrt{3} \approx 1.732$ and $\frac{\pi}{2} \approx 1.571$), the interval $(\sqrt{3}, \infty)$ covers values of $\theta$ that span across multiple periods of the cosine function, starting from a value greater than $\frac{\pi}{2}$.
As $\theta$ ranges from $\sqrt{3}$ to $\infty$, the value of $\cos(\theta)$ oscillates between -1 and 1, covering the entire interval $[-1, 1]$.
So, the range of $\cos \sqrt{3 + x + x^2}$ for $x > 0$ is $[-1, 1]$.
Finally, consider $f(x) = -3 \cos \sqrt{3 + x + x^2}$.
Since the range of $\cos \sqrt{3 + x + x^2}$ is $[-1, 1]$, the range of $-3$ times this value is obtained by multiplying the endpoints of the interval $[-1, 1]$ by $-3$ and reversing the order.
$-3 \times 1 = -3$
$-3 \times -1 = 3$
So, the range of $f(x)$ is $[-3, 3]$.
The values of $f(x)$ lie in the interval $[-3, 3]$.
The final answer is $[-3, 3]$.
Question 67. The maximum distance of a point on the graph of the function y = $\sqrt{3}$ sin x + cos x from x-axis is _______.
Answer:
Given:
The function $y = \sqrt{3} \sin x + \cos x$.
To Find:
The maximum distance of a point on the graph of the function from the x-axis.
Solution:
The distance of a point $(x, y)$ on the graph from the x-axis is given by $|y|$.
We need to find the maximum value of $|y| = |\sqrt{3} \sin x + \cos x|$.
To find the maximum value of the expression $\sqrt{3} \sin x + \cos x$, we can transform it into the form $R \sin(x + \alpha)$ or $R \cos(x - \alpha)$.
Consider the form $R \sin(x + \alpha) = R (\sin x \cos \alpha + \cos x \sin \alpha) = (R \cos \alpha) \sin x + (R \sin \alpha) \cos x$.
Comparing this with $\sqrt{3} \sin x + \cos x$, we have:
$R \cos \alpha = \sqrt{3}$
$R \sin \alpha = 1$
Squaring and adding these two equations:
$(R \cos \alpha)^2 + (R \sin \alpha)^2 = (\sqrt{3})^2 + 1^2$
$R^2 \cos^2 \alpha + R^2 \sin^2 \alpha = 3 + 1$
$R^2 (\cos^2 \alpha + \sin^2 \alpha) = 4$
$R^2 (1) = 4$
[Using $\cos^2 \alpha + \sin^2 \alpha = 1$]
$R^2 = 4 \implies R = 2$ (Since $R$ is the amplitude, $R > 0$).
Now, divide the second equation by the first (assuming $R \cos \alpha \neq 0$):
$\frac{R \sin \alpha}{R \cos \alpha} = \frac{1}{\sqrt{3}}$
$\tan \alpha = \frac{1}{\sqrt{3}}$
Since $R \cos \alpha = \sqrt{3} > 0$ and $R \sin \alpha = 1 > 0$, $\alpha$ is in the first quadrant.
Thus, $\alpha = \frac{\pi}{6}$ (or $30^\circ$).
So, the function can be rewritten as:
$y = 2 \sin \left(x + \frac{\pi}{6}\right)$
The range of the sine function is $[-1, 1]$. That is, $-1 \leq \sin \theta \leq 1$ for any angle $\theta$.
Therefore, the range of $\sin \left(x + \frac{\pi}{6}\right)$ is $[-1, 1]$.
Now, multiply the range by 2 to find the range of $y = 2 \sin \left(x + \frac{\pi}{6}\right)$:
$2 \times (-1) \leq 2 \sin \left(x + \frac{\pi}{6}\right) \leq 2 \times 1$
$-2 \leq y \leq 2$
The range of $y$ is $[-2, 2]$.
The distance of a point $(x, y)$ from the x-axis is $|y|$.
Since $y \in [-2, 2]$, the value of $|y|$ will be between $|-2|$ and $|2|$, which is $2$, and the minimum value is $0$ (when $y=0$).
So, the range of $|y|$ is $[0, 2]$.
The maximum value of $|y|$ is 2.
This is the maximum distance of a point on the graph from the x-axis.
The maximum distance is 2.
The final answer is 2.
Question 68 to 75 (True or False)
In each of the Exercises 68 to 75, state whether the statements is True or False? Also give justification.
Question 68. If tan A = $\frac{1 \;−\; cos \;B}{sin \;B}$ , then tan 2A = tan B
Answer:
The statement is True.
Justification:
We are given that $\tan A = \frac{1 - \cos B}{\sin B}$.
We use the half-angle identities for $\cos B$ and $\sin B$:
$1 - \cos B = 2 \sin^2 \left( \frac{B}{2} \right)$
$\sin B = 2 \sin \left( \frac{B}{2} \right) \cos \left( \frac{B}{2} \right)$
Substitute these into the expression for $\tan A$:
$\tan A = \frac{2 \sin^2 \left( \frac{B}{2} \right)}{2 \sin \left( \frac{B}{2} \right) \cos \left( \frac{B}{2} \right)}$
Assuming $\sin \left( \frac{B}{2} \right) \neq 0$, we can cancel terms:
$\tan A = \frac{\sin \left( \frac{B}{2} \right)}{\cos \left( \frac{B}{2} \right)}$
$\tan A = \tan \left( \frac{B}{2} \right)$
Now, consider the expression $\tan 2A$. Using the double-angle formula for tangent:
$\tan 2A = \frac{2 \tan A}{1 - \tan^2 A}$
Substitute $\tan A = \tan \left( \frac{B}{2} \right)$ into this formula:
$\tan 2A = \frac{2 \tan \left( \frac{B}{2} \right)}{1 - \tan^2 \left( \frac{B}{2} \right)}$
Recognize that the expression $\frac{2 \tan \left( \frac{B}{2} \right)}{1 - \tan^2 \left( \frac{B}{2} \right)}$ is the double-angle formula for $\tan B$, i.e., $\tan B = \frac{2 \tan \left( \frac{B}{2} \right)}{1 - \tan^2 \left( \frac{B}{2} \right)}$.
$\tan 2A = \tan B$
Therefore, if $\tan A = \frac{1 - \cos B}{\sin B}$, then $\tan 2A = \tan B$. The statement is True.
Question 69. The equality sin A + sin 2A + sin 3A = 3 holds for some real value of A.
Answer:
The statement is False.
Justification:
The range of the sine function is $[-1, 1]$. This means that for any real value of $x$, we have $-1 \leq \sin x \leq 1$.
The given equality is $\sin A + \sin 2A + \sin 3A = 3$.
For this equality to hold, the sum of the three terms must be equal to 3.
Since the maximum value of each term is 1 (i.e., $\sin A \leq 1$, $\sin 2A \leq 1$, and $\sin 3A \leq 1$), the only way their sum can be equal to 3 is if each term is simultaneously equal to its maximum value, which is 1.
So, we must have:
$\sin A = 1$
$\sin 2A = 1$
$\sin 3A = 1$
Let's find the values of A for which $\sin A = 1$.
$\sin A = 1 \implies A = \frac{\pi}{2} + 2n\pi$, where $n$ is an integer.
Now, let's check if these values of A also satisfy $\sin 2A = 1$.
Substitute $A = \frac{\pi}{2} + 2n\pi$ into the expression for $2A$:
$2A = 2 \left( \frac{\pi}{2} + 2n\pi \right) = \pi + 4n\pi = (4n+1)\pi$
Now, evaluate $\sin 2A$ for these values of $2A$:
$\sin 2A = \sin ((4n+1)\pi) = \sin (\pi + 4n\pi)$
Since $\sin(\theta + 2k\pi) = \sin \theta$ for any integer $k$, we have:
$\sin (\pi + 4n\pi) = \sin \pi$
The value of $\sin \pi$ is 0.
$\sin 2A = 0$
We see that if $\sin A = 1$, then $\sin 2A = 0$. This contradicts the requirement that $\sin 2A = 1$.
Since the conditions $\sin A = 1$, $\sin 2A = 1$, and $\sin 3A = 1$ cannot be simultaneously satisfied for any real value of A, the equality $\sin A + \sin 2A + \sin 3A = 3$ cannot hold for any real value of A.
Question 70. sin 10° is greater than cos 10°.
Answer:
The statement is False.
Justification:
Consider the angles in the first quadrant ($0^\circ < \theta < 90^\circ$).
For angles $\theta$ such that $0^\circ < \theta < 45^\circ$, the value of $\cos \theta$ is greater than the value of $\sin \theta$.
For angles $\theta$ such that $45^\circ < \theta < 90^\circ$, the value of $\sin \theta$ is greater than the value of $\cos \theta$.
At $\theta = 45^\circ$, $\sin 45^\circ = \cos 45^\circ = \frac{1}{\sqrt{2}}$.
The angle given is $10^\circ$.
Since $10^\circ$ lies in the interval $0^\circ < 10^\circ < 45^\circ$, we must have $\cos 10^\circ > \sin 10^\circ$.
Therefore, $\sin 10^\circ$ is not greater than $\cos 10^\circ$. The statement is False.
Question 71. cos $\frac{2\pi}{15}$ cos $\frac{4\pi}{15}$ cos $\frac{8\pi}{15}$ cos $\frac{16\pi}{15}$ = $\frac{1}{16}$
Answer:
The statement is True.
Justification:
Let the given expression be denoted by P:
$P = \cos \frac{2\pi}{15} \cos \frac{4\pi}{15} \cos \frac{8\pi}{15} \cos \frac{16\pi}{15}$
This is a product of cosine terms where the angles are in a geometric progression with a common ratio of 2. The general formula for such a product is:
$\cos \theta \cos 2\theta \cos 4\theta \cdots \cos (2^{n-1}\theta) = \frac{\sin(2^n \theta)}{2^n \sin \theta}$
In our case, the first angle is $\theta = \frac{2\pi}{15}$, and there are 4 terms, so $n = 4$.
Applying the formula with $\theta = \frac{2\pi}{15}$ and $n = 4$:
$P = \frac{\sin \left(2^4 \cdot \frac{2\pi}{15}\right)}{2^4 \sin \left(\frac{2\pi}{15}\right)}$
Calculate $2^4 = 16$ and $2^4 \cdot \frac{2\pi}{15} = 16 \cdot \frac{2\pi}{15} = \frac{32\pi}{15}$.
$P = \frac{\sin \left(\frac{32\pi}{15}\right)}{16 \sin \left(\frac{2\pi}{15}\right)}$
Now, simplify the numerator $\sin \left(\frac{32\pi}{15}\right)$.
We can write $\frac{32\pi}{15}$ as $\frac{30\pi + 2\pi}{15} = \frac{30\pi}{15} + \frac{2\pi}{15} = 2\pi + \frac{2\pi}{15}$.
Using the property $\sin(2k\pi + \phi) = \sin \phi$ for any integer $k$:
$\sin \left(\frac{32\pi}{15}\right) = \sin \left(2\pi + \frac{2\pi}{15}\right) = \sin \left(\frac{2\pi}{15}\right)$
Substitute this back into the expression for P:
$P = \frac{\sin \left(\frac{2\pi}{15}\right)}{16 \sin \left(\frac{2\pi}{15}\right)}$
Since $\frac{2\pi}{15}$ is not a multiple of $\pi$, $\sin \left(\frac{2\pi}{15}\right) \neq 0$. We can cancel the term $\sin \left(\frac{2\pi}{15}\right)$ from the numerator and denominator.
$P = \frac{\cancel{\sin \left(\frac{2\pi}{15}\right)}}{16 \cancel{\sin \left(\frac{2\pi}{15}\right)}} = \frac{1}{16}$
The value of the left side of the equality is $\frac{1}{16}$, which matches the right side.
Therefore, the statement is True.
Question 72. One value of θ which satisfies the equation sin4 θ – 2sin2 θ – 1 lies between 0 and 2π.
Answer:
The statement is False.
Justification:
We are given the equation $\sin^4 \theta – 2\sin^2 \theta – 1 = 0$.
Let $y = \sin^2 \theta$. Since $-1 \leq \sin \theta \leq 1$, the value of $\sin^2 \theta$ must satisfy $0 \leq \sin^2 \theta \leq 1$. So, $0 \leq y \leq 1$.
Substitute $y = \sin^2 \theta$ into the given equation:
$y^2 - 2y - 1 = 0$
This is a quadratic equation in $y$. We can solve for $y$ using the quadratic formula $y = \frac{-b \pm \sqrt{b^2 - 4ac}}{2a}$, where $a=1$, $b=-2$, and $c=-1$.
$y = \frac{-(-2) \pm \sqrt{(-2)^2 - 4(1)(-1)}}{2(1)}$
$y = \frac{2 \pm \sqrt{4 + 4}}{2}$
$y = \frac{2 \pm \sqrt{8}}{2}$
$y = \frac{2 \pm 2\sqrt{2}}{2}$
$y = 1 \pm \sqrt{2}$
So, the possible values for $y = \sin^2 \theta$ are:
$y_1 = 1 + \sqrt{2}$
$y_2 = 1 - \sqrt{2}$
Now we check if these values are within the valid range for $\sin^2 \theta$, which is $[0, 1]$.
For $y_1 = 1 + \sqrt{2}$:
Since $\sqrt{2} \approx 1.414$, $y_1 \approx 1 + 1.414 = 2.414$.
This value is greater than 1, so $y_1 = 1 + \sqrt{2}$ is not a possible value for $\sin^2 \theta$.
For $y_2 = 1 - \sqrt{2}$:
Since $\sqrt{2} \approx 1.414$, $y_2 \approx 1 - 1.414 = -0.414$.
This value is less than 0, so $y_2 = 1 - \sqrt{2}$ is not a possible value for $\sin^2 \theta$ (as squares must be non-negative).
Since neither of the solutions for $\sin^2 \theta$ are valid real numbers for $\sin^2 \theta$, there is no real value of $\theta$ for which the equation $\sin^4 \theta – 2\sin^2 \theta – 1 = 0$ holds.
Therefore, there is no value of $\theta$ in the interval $(0, 2\pi)$ that satisfies the equation. The statement is False.
Question 73. If cosec x = 1 + cot x then x = 2nπ, 2nπ + $\frac{\pi}{2}$
Answer:
The statement is False.
Justification:
The given equation is $\text{cosec } x = 1 + \cot x$.
Recall that $\text{cosec } x = \frac{1}{\sin x}$ and $\cot x = \frac{\cos x}{\sin x}$.
For $\text{cosec } x$ and $\cot x$ to be defined, the denominator $\sin x$ must not be equal to zero.
$\sin x \neq 0$
The values of $x$ for which $\sin x = 0$ are $x = m\pi$, where $m$ is an integer.
This set includes values like $0, \pm\pi, \pm 2\pi, \pm 3\pi, \dots$.
The statement claims that $x = 2n\pi$ and $x = 2n\pi + \frac{\pi}{2}$ are solutions.
Consider the values $x = 2n\pi$ (where $n$ is an integer). These are a subset of the values where $\sin x = 0$ (specifically, when $m$ is an even integer, $m=2n$).
For $x = 2n\pi$, $\sin(2n\pi) = 0$.
If $\sin x = 0$, then $\text{cosec } x = \frac{1}{\sin x}$ and $\cot x = \frac{\cos x}{\sin x}$ are undefined.
Since the terms in the original equation are undefined for $x = 2n\pi$, these values cannot be solutions to the equation.
The statement includes $x = 2n\pi$ in the solution set. Therefore, the statement is false.
Note: The correct solutions to the equation $\text{cosec } x = 1 + \cot x$ are $x = 2k\pi + \frac{\pi}{2}$ where $k$ is an integer, which can be shown by solving $\sin x + \cos x = 1$ and checking the domain condition $\sin x \neq 0$.
Question 74. If tan θ + tan 2θ + $\sqrt{3}$ tan θ tan 2θ = $\sqrt{3}$ , then θ = $\frac{n\pi}{3}$ + $\frac{\pi}{9}$
Answer:
The statement is True.
Justification:
We are given the equation $\tan \theta + \tan 2\theta + \sqrt{3} \tan \theta \tan 2\theta = \sqrt{3}$.
Rearrange the equation to isolate the terms involving $\sqrt{3}$:
$\tan \theta + \tan 2\theta = \sqrt{3} - \sqrt{3} \tan \theta \tan 2\theta$
Factor out $\sqrt{3}$ from the right side:
$\tan \theta + \tan 2\theta = \sqrt{3} (1 - \tan \theta \tan 2\theta)$
Assuming $1 - \tan \theta \tan 2\theta \neq 0$, we can divide both sides by $(1 - \tan \theta \tan 2\theta)$:
$\frac{\tan \theta + \tan 2\theta}{1 - \tan \theta \tan 2\theta} = \sqrt{3}$
Recognize the left side of the equation as the tangent addition formula: $\tan(A + B) = \frac{\tan A + \tan B}{1 - \tan A \tan B}$.
Here, $A = \theta$ and $B = 2\theta$. So, the left side is $\tan(\theta + 2\theta) = \tan(3\theta)$.
$\tan(3\theta) = \sqrt{3}$
The general solution for the equation $\tan x = \sqrt{3}$ is $x = k\pi + \arctan(\sqrt{3})$, where $k$ is an integer.
Since $\arctan(\sqrt{3}) = \frac{\pi}{3}$, the general solution for $3\theta$ is:
$3\theta = k\pi + \frac{\pi}{3}$
[where $k$ is an integer]
Divide both sides by 3 to solve for $\theta$:
$\theta = \frac{k\pi}{3} + \frac{\pi}{9}$
This general solution derived from the equation matches the form given in the statement $\theta = \frac{n\pi}{3} + \frac{\pi}{9}$, where $n$ is an integer. The use of the variable $n$ instead of $k$ does not change the set of solutions.
We should also consider the assumption $1 - \tan \theta \tan 2\theta \neq 0$. If $1 - \tan \theta \tan 2\theta = 0$, the original equation becomes $\tan \theta + \tan 2\theta + \sqrt{3}(0) = \sqrt{3}$, so $\tan \theta + \tan 2\theta = \sqrt{3}$. If $1 - \tan \theta \tan 2\theta = 0$, then $\tan \theta \tan 2\theta = 1$. If $\tan \theta + \tan 2\theta = \sqrt{3}$ and $\tan \theta \tan 2\theta = 1$, then $\tan \theta$ and $\tan 2\theta$ are the roots of the quadratic equation $t^2 - \sqrt{3}t + 1 = 0$. The discriminant is $(-\sqrt{3})^2 - 4(1)(1) = 3 - 4 = -1 < 0$. Since the discriminant is negative, there are no real values for $\tan \theta$ and $\tan 2\theta$ satisfying both conditions simultaneously. Thus, the case $1 - \tan \theta \tan 2\theta = 0$ does not lead to any real solutions for $\theta$.
Also, the tangents $\tan \theta$ and $\tan 2\theta$ must be defined. The derived solutions $\theta = \frac{(3k+1)\pi}{9}$ do not coincide with values where $\tan \theta$ or $\tan 2\theta$ are undefined (multiples of $\frac{\pi}{2}$).
Since the set of solutions derived from the equation is exactly $\theta = \frac{k\pi}{3} + \frac{\pi}{9}$ for any integer $k$, the statement is True.
Question 75. If tan (π cos θ) = cot (π sin θ), then $\cos \left( \theta - \frac{\pi}{4} \right) = \pm\frac{1}{2\sqrt{2}}$
Answer:
The statement is True.
Justification:
We are given the equation $\tan (\pi \cos \theta) = \cot (\pi \sin \theta)$.
We use the trigonometric identity $\cot x = \tan \left( \frac{\pi}{2} - x \right)$.
Applying this identity to the right side of the equation, with $x = \pi \sin \theta$:
$\tan (\pi \cos \theta) = \tan \left( \frac{\pi}{2} - \pi \sin \theta \right)$
[Using $\cot x = \tan \left( \frac{\pi}{2} - x \right)$]
If $\tan A = \tan B$, then $A = n\pi + B$, where $n$ is an integer.
Here, $A = \pi \cos \theta$ and $B = \frac{\pi}{2} - \pi \sin \theta$.
$\pi \cos \theta = n\pi + \frac{\pi}{2} - \pi \sin \theta$
[where $n$ is an integer]
Divide the entire equation by $\pi$ (assuming $\pi \neq 0$):
$\cos \theta = n + \frac{1}{2} - \sin \theta$
Rearrange the terms to get $\sin \theta + \cos \theta$ on one side:
$\sin \theta + \cos \theta = n + \frac{1}{2}$
We can express $\sin \theta + \cos \theta$ in the form $R \cos(\theta - \alpha)$.
$\sin \theta + \cos \theta = \sqrt{1^2 + 1^2} \left( \frac{1}{\sqrt{2}} \sin \theta + \frac{1}{\sqrt{2}} \cos \theta \right)$
$\sin \theta + \cos \theta = \sqrt{2} \left( \cos \frac{\pi}{4} \sin \theta + \sin \frac{\pi}{4} \cos \theta \right)$
$\sin \theta + \cos \theta = \sqrt{2} \sin \left( \theta + \frac{\pi}{4} \right)$
Alternatively, $\sin \theta + \cos \theta = \sqrt{2} \left( \cos \frac{\pi}{4} \cos \theta + \sin \frac{\pi}{4} \sin \theta \right) = \sqrt{2} \cos \left( \theta - \frac{\pi}{4} \right)$.
So, the equation $\sin \theta + \cos \theta = n + \frac{1}{2}$ becomes:
$\sqrt{2} \cos \left( \theta - \frac{\pi}{4} \right) = n + \frac{1}{2}$
Since $\theta$ is a real number, $\cos \left( \theta - \frac{\pi}{4} \right)$ must be a real number within the range $[-1, 1]$.
$-1 \leq \cos \left( \theta - \frac{\pi}{4} \right) \leq 1$
Multiplying by $\sqrt{2}$:
$-\sqrt{2} \leq \sqrt{2} \cos \left( \theta - \frac{\pi}{4} \right) \leq \sqrt{2}$
Substituting $\sqrt{2} \cos \left( \theta - \frac{\pi}{4} \right) = n + \frac{1}{2}$:
$-\sqrt{2} \leq n + \frac{1}{2} \leq \sqrt{2}$
Subtract $\frac{1}{2}$ from all parts of the inequality:
$-\sqrt{2} - \frac{1}{2} \leq n \leq \sqrt{2} - \frac{1}{2}$
Using the approximation $\sqrt{2} \approx 1.414$:
$-1.414 - 0.5 \leq n \leq 1.414 - 0.5$
$-1.914 \leq n \leq 0.914$
Since $n$ must be an integer, the possible values for $n$ are $-1$ and $0$.
If $n = -1$, then $n + \frac{1}{2} = -1 + \frac{1}{2} = -\frac{1}{2}$.
The equation becomes $\sqrt{2} \cos \left( \theta - \frac{\pi}{4} \right) = -\frac{1}{2}$.
$\cos \left( \theta - \frac{\pi}{4} \right) = -\frac{1}{2\sqrt{2}}$
If $n = 0$, then $n + \frac{1}{2} = 0 + \frac{1}{2} = \frac{1}{2}$.
The equation becomes $\sqrt{2} \cos \left( \theta - \frac{\pi}{4} \right) = \frac{1}{2}$.
$\cos \left( \theta - \frac{\pi}{4} \right) = \frac{1}{2\sqrt{2}}$
Combining the possible values, we have $\cos \left( \theta - \frac{\pi}{4} \right) = \pm \frac{1}{2\sqrt{2}}$.
This matches the conclusion given in the statement.
Therefore, the statement is True.
Question 76 (Match the Following)
Question 76. In the following match each item given under the column C1 to its correct answer given under the column C2 :
$C_1$
(a) $\sin (x + y) \sin (x – y)$
(b) $\cos (x + y) \cos (x – y)$
(c) $\cot \left( \frac{\pi}{4} + \theta \right)$
(d) $\tan \left( \frac{\pi}{4} + \theta \right)$
$C_2$
(i) $\cos^2 x – \sin^2 y$
(ii) $\frac{1 - \tan \theta}{1 + \tan \theta}$
(iii) $\frac{1 + \tan \theta}{1 - \tan \theta}$
(iv) $\sin^2 x – \sin^2 y$
Answer:
The correct matches are as follows:
(a) $\sin (x + y) \sin (x – y)$ matches with (iv) $\sin^2 x – \sin^2 y$.
Justification:
Using the product-to-sum identity $\sin A \sin B = \frac{1}{2}[\cos(A-B) - \cos(A+B)]$.
Let $A = x+y$ and $B = x-y$. Then $A-B = 2y$ and $A+B = 2x$.
$\sin (x + y) \sin (x – y) = \frac{1}{2}[\cos(2y) - \cos(2x)] $
Using the double angle identity $\cos 2\phi = 1 - 2\sin^2 \phi$:
$= \frac{1}{2}[(1 - 2\sin^2 y) - (1 - 2\sin^2 x)] $
$= \frac{1}{2}[1 - 2\sin^2 y - 1 + 2\sin^2 x] $
$= \frac{1}{2}[2\sin^2 x - 2\sin^2 y] = \sin^2 x - \sin^2 y$
(b) $\cos (x + y) \cos (x – y)$ matches with (i) $\cos^2 x – \sin^2 y$.
Justification:
Using the product-to-sum identity $\cos A \cos B = \frac{1}{2}[\cos(A-B) + \cos(A+B)]$.
Let $A = x+y$ and $B = x-y$. Then $A-B = 2y$ and $A+B = 2x$.
$\cos (x + y) \cos (x – y) = \frac{1}{2}[\cos(2y) + \cos(2x)] $
Using the double angle identity $\cos 2y = 1 - 2\sin^2 y$ and $\cos 2x = 2\cos^2 x - 1$:
$= \frac{1}{2}[(1 - 2\sin^2 y) + (2\cos^2 x - 1)] $
$= \frac{1}{2}[1 - 2\sin^2 y + 2\cos^2 x - 1] $
$= \frac{1}{2}[2\cos^2 x - 2\sin^2 y] = \cos^2 x - \sin^2 y$
(c) $\cot \left( \frac{\pi}{4} + \theta \right)$ matches with (ii) $\frac{1 - \tan \theta}{1 + \tan \theta}$.
Justification:
Using the identity $\cot A = \frac{1}{\tan A}$:
$\cot \left( \frac{\pi}{4} + \theta \right) = \frac{1}{\tan \left( \frac{\pi}{4} + \theta \right)}$
Using the tangent addition formula $\tan(A+B) = \frac{\tan A + \tan B}{1 - \tan A \tan B}$ with $A = \frac{\pi}{4}$ and $B = \theta$, and $\tan \frac{\pi}{4} = 1$:
$\tan \left( \frac{\pi}{4} + \theta \right) = \frac{\tan \frac{\pi}{4} + \tan \theta}{1 - \tan \frac{\pi}{4} \tan \theta} = \frac{1 + \tan \theta}{1 - \tan \theta}$
Substitute this back into the cotangent expression:
$\cot \left( \frac{\pi}{4} + \theta \right) = \frac{1}{\frac{1 + \tan \theta}{1 - \tan \theta}} = \frac{1 - \tan \theta}{1 + \tan \theta}$
(d) $\tan \left( \frac{\pi}{4} + \theta \right)$ matches with (iii) $\frac{1 + \tan \theta}{1 - \tan \theta}$.
Justification:
Using the tangent addition formula $\tan(A+B) = \frac{\tan A + \tan B}{1 - \tan A \tan B}$ with $A = \frac{\pi}{4}$ and $B = \theta$, and $\tan \frac{\pi}{4} = 1$:
$\tan \left( \frac{\pi}{4} + \theta \right) = \frac{\tan \frac{\pi}{4} + \tan \theta}{1 - \tan \frac{\pi}{4} \tan \theta} = \frac{1 + \tan \theta}{1 - \tan \theta}$
Summary of matches:
(a) - (iv)
(b) - (i)
(c) - (ii)
(d) - (iii)